You are on page 1of 147

MATHSCOPE.

ORG
Seeking the Unification of Math

Phan c Minh Trng Tn Sang
Nguyn Th Nguyn Khoa L Tun Linh Phm Huy Hong Nguyn Hin Trang





Tuyn tp cc bi ton
HNH HC PHNG

Cc bi ton n tp tuyn sinh lp 10
Cc bi ton n tp Olympiad








Thng 10/2011





MATHSCOPE.ORG
Seeking the Unification of Math

Phan c Minh Trng Tn Sang
Nguyn Th Nguyn Khoa L Tun Linh Phm Huy Hong Nguyn Hin Trang





Tuyn tp cc bi ton
HNH HC PHNG

Cc bi ton n tp tuyn sinh lp 10
Cc bi ton n tp Olympiad








Thng 10/2011
1. Quyn sch c kim duyt v ng bi ban qun tr din n MathScope.org v l
ti sn ca din n MathScope.org. Cm mi hnh thc sao chp v dn cc logo khng
hp l. Cc hnh thc upload file sch ln cc mng x hi, cc trang cng ng, cc din
n khc,. . . u phi ghi r ngun din n MathScope.org.
2. Sch c tng hp phi li nhun. Cm mi hnh thc thu li nhun t vic bn, photo
sch v cc loi hnh khc.
3. Sch c tng hp t ngun ti nguyn ca din n MathScope.org. Do sch c
quyn khng nu tn cc tc gi ca li gii cc bi ton v ngi bin son chnh sa
ni dung v hnh thc din t sao cho hp l.
4. Mi thc mc v bn quyn xin lin h vi ban qun tr din n MathScope.org hoc gi
trc tip ln din n.
5. Nu bn khng ng vi nhng iu khon nu trn, xin vui lng khng s dng sch.
Vic s dng quyn sch chng t bn chp nhn cc iu khon trn.
3
Mc lc
Li ni u 4
Cc thnh vin tham gia bin son 5
Phn mt. Cc kin thc c bn 6
Phn hai. Tuyn tp cc bi ton 9
I. bi . . . . . . . . . . . . . . . . . . . . . . . . . . . . . . . . . . . . . . . . . 9
1. Cc bi ton n tp tuyn sinh lp 10 . . . . . . . . . . . . . . . . . . . . . . 9
2. Cc bi ton n tp Olympiad . . . . . . . . . . . . . . . . . . . . . . . . . . 14
II. Hng dn v gi . . . . . . . . . . . . . . . . . . . . . . . . . . . . . . . . . 21
1. Cc bi ton n tp tuyn sinh lp 10 . . . . . . . . . . . . . . . . . . . . . . 21
2. Cc bi ton n tp Olympiad . . . . . . . . . . . . . . . . . . . . . . . . . . 26
III. Li gii chi tit . . . . . . . . . . . . . . . . . . . . . . . . . . . . . . . . . . . 38
1. Cc bi ton n tp tuyn sinh lp 10 . . . . . . . . . . . . . . . . . . . . . . 38
2. Cc bi ton n tp Olympiad . . . . . . . . . . . . . . . . . . . . . . . . . . 74
4
Li ni u
T bui s khai trong x hi loi ngi, ton hc lun gn lin vi cc lnh vc i sng nh
kin trc, hi ha, khoa hc,. . . V trong hu ht cc lnh vc ca ton hc, hnh hc phng
lun gi v tr ng u v n chnh l nn tng xy dng nn hnh hc khng gian, l c s ca
cc ngnh kin trc, ngh thut v ton hc ng dng. Cng nh lch s pht trin, chng ta
tip xc vi hnh hc phng t rt sm. Cc khi nim v im, ng thng, on thng
c cp n ngay tiu hc. Hnh hc tri di n tn nm cui cp THPT v i theo
n nhng nm i hc, iu ny khng nh vai tr quan trng ca hnh hc ni chung v
hnh hc phng ni ring.
ng thi vi s pht trin ca ton hc, hnh hc phng cng pht trin khng ngng. Lin
tip cc kt qu mi c pht hin v nhng k thut mi c khm ph. Chnh v th, vic
bt kp cc kin thc ca hnh hc phng l cn thit v quan trng. y cng chnh l l
do quyn sch Tuyn tp cc bi ton hnh hc phng ra i. Quyn sch c tng hp t
ti nguyn trn din n MathScope.org v l ti sn ca MathScope.org, tc gi cc bi ton
v li gii, nhm tng hp u l cc thnh vin ca din n MathScope.org vi mong mun
cung cp cho bn hc sinh, sinh vin v thy c gio trn ton quc mt ti liu phong ph v
hnh hc phng, h tr cho qu trnh hc tp v ging dy.
Tuyn tp cc bi ton hnh hc phng khng ch nhm vo i tng d thi Olympic m
cn l ngun ti liu cho cc em hc sinh cp 2 chun b cho k thi tuyn sinh lp 10. Do ,
cc bi ton c chia thnh 2 phn : dnh cho cc em n thi lp 10 v cc bn thi Olympic
ph hp hn vi bn c. Mi bi ton u c nhng hng dn, gi trc khi nu ra li
gii chi tit gip bn c suy lun v tip tc gii quyt bi ton vi nhng gi . Xin
lu rng nhng li nhn xt trong phn hng dn v gi l nhng kin ch quan ca
ngi bin son. Xin cm n ban qun tr v cc thnh vin din n MathScope.org ng
gp, ng h v gip hon thnh quyn sch ny. V xin cm n thy Chu Ngc Hng -
gio vin trng THPT Ninh Hi, Ninh Thun h tr v L
A
T
E
X hon thin quyn sch.
Tuy nhin, chc chn rng cun sch vn cn nhng hn ch nht nh, chng ti rt hoan
nghnh nhng kin ng gp, chia s ca bn c cun sch c hon thin hn. Bn
c c th gp bng cch gi email ring ti hm th alephvn@gmail.com hoc gi trc tip
ln din n MathScope.org (http://forum.mathscope.org/index.php).
Thay mt nhm bin son, ti xin chn thnh cm n s quan tm ca bn c!
H Ni, ngy 31 thng 10 nm 2011
i din nhm bin son
Ch bin
Phan c Minh
5
Cc thnh vin tham gia bin son
Ni dung
Phan c Minh (novae) - HKHTN, HQGHN.
Trng Tn Sang (sang89) - Westminster High School, California, USA.
Nguyn Th Nguyn Khoa (liverpool29) - THCS Nguyn Tri Phng, Thnh ph Hu.
L Tun Linh (conami) - THPT chuyn Lam Sn, Thanh Ha.
Phm Huy Hong (hoangkhtn) - THPT chuyn, HKHTN, HQG H Ni.
Nguyn Hin Trang (tranghieu95) - THPT chuyn Phan Bi Chu, Ngh An
H tr k thut L
A
T
E
X
Chu Ngc Hng (hungchng) - Gio vin trng THPT Ninh Hi, Ninh Thun.
Trnh by ba
V Anh Khoa (anhkhoavo1210) - HKHTN, HQGTPHCM.
Phan c Minh.
6
Phn mt. Cc kin thc c bn
1. nh l Menelaus
Cho tam gic ABC, cc im D, E, F theo th t nm trn cc ng thng BC, CA, AB.
Khi D, E, F thng hng khi v ch khi
FA
FB

DB
DC

EC
EA
= 1
Ch : nh l Menelaus c th m rng cho a gic li n cnh.
2. nh l Ceva
Cho tam gic ABC, cc im D, E, F theo th t nm trn cc ng thng BC, CA, AB.
Khi AD, BE, CF ng quy khi v ch khi
FA
FB

DB
DC

EC
EA
= 1
3. ng thng Euler
Cho tam gic ABC; O, G, H theo th t l tm ng trn ngoi tip, trng tm v trc tm
tam gic. Khi O, G, H thng hng v OH = OG. ng thng i qua O, G, H c gi l
ng thng Euler ca tam gic ABC.
4. ng trn Euler
Vi mi tam gic ABC bt k, 9 im : trung im cc cnh, chn cc ng cao, trung im
cc on thng ni trc tm tam gic vi cc nh cng nm trn mt ng trn, gi l ng
trn Euler ca tam gic ABC. ng trn Euler c bn knh bng mt na bn knh ng
trn ngoi tip tam gic v c tm l trung im on thng ni trc tm v tm ng trn
ngoi tip tam gic.
5. nh l con bm
Cho ng trn (O) v I l trung im ca mt dy cung AB. Qua I dng hai dy cung ty
MN, PQ sao cho MP, NQ ct AB ti E, F theo th t. Khi I l trung im EF.
6. nh l Ptolemy
Vi mi t gic li ABCD ni tip trong mt ng trn, ta u c ng thc
AB CD +AD BC = AC BD
Tng qut : (bt ng thc Ptolemy) Vi mi t gic ABCD bt k, ta c bt ng thc
AB CD +AD BC AC BD
ng thc xy ra khi v ch khi ABCD l t gic li ni tip.
7
7. nh l Stewart
Vi ba im A, B, C thng hng v mt im M bt k, ta c
MA
2
BC +MB
2
CA +MC
2
AB +AB BC CA = 0
Hai h qu quen thuc ca nh l Stewart l cng thc di ng trung tuyn v di
ng phn gic trong : Cho tam gic ABC. t BC = a, CA = b, AB = c; m
a
, l
a
ln lt l
di ng trung tuyn v di ng phn gic trong ng vi nh A ca tam gic. Khi
ta c
m
2
a
=
b
2
+c
2
2

a
2
4
l
2
a
= bc
_
1
a
2
(b +c)
2
_
8. ng thng Simson
Cho tam gic ABC v mt im M nm trn ng trn ngoi tip tam gic. Gi X, Y, Z ln
lt l hnh chiu vung gc ca M trn cc ng thng BC, CA, AB. Khi X, Y, Z thng
hng v ng thng i qua chng c gi l ng thng Simson ca im M i vi tam
gic ABC.
Tng qut : Cho tam gic ABC v mt im M bt k trong mt phng tam gic. Gi X, Y, Z
ln lt l hnh chiu vung gc ca M trn cc ng thng BC, CA, AB. Khi iu kin
cn v M nm trn ng trn ngoi tip tam gic ABC l X, Y, Z thng hng.
9. ng thng Steiner
Cho tam gic ABC v mt im M nm trn ng trn ngoi tip tam gic. Gi X, Y, Z ln
lt l cc im i xng vi M qua BC, CA, AB. Khi X, Y, Z thng hng v ng thng
i qua chng c gi l ng thng Steiner ca im M i vi tam gic ABC. ng thng
Steiner lun i qua trc tm tam gic.
10. im Miquel ca tam gic, t gic ton phn
Cho tam gic ABC v ba im M, N, P tng ng nm trn cc ng thng BC, CA, AB.
Khi cc ng trn ngoi tip cc tam gic ANP, BPM, CMN ng quy ti im Miquel
X ca M, N, P i vi tam gic ABC.
Khi M, N, P thng hng, ta c X im Miquel ca t gic ton phn ABCMNP. Khi X
nm trn ng trn ngoi tip tam gic ABC.
11. ng trn Miquel ca t gic ton phn
Cho t gic ton phn ABCDEF, im Miquel M ca t gic v tm ngoi tip cc tam gic
AEF, CDE, BDF, ABC cng nm trn ng trn Miquel ca t gic.
8
12. nh l Pascal
Cho 6 im A, B, C, D, E, F cng nm trn mt conic bt k. Gi G, H, K theo th t l giao
im ca cc cp ng thng (AB, DE), (BC, EF), (CD, FA). Khi G, H, K thng hng.
13. nh l Pappus
Cho hai ng thng a, b. Trn a ly cc im A, B, C; trn b ly cc im D, E, F. Gi G, H, K
ln lt l giao im ca cc cp ng thng (AE, DB), (AF, CD), (BF, CE). Khi G, H, K
thng hng.
nh l Pappus l trng hp suy bin ca nh l Pascal khi conic suy bin thnh cp ng
thng.
14. Bt ng thc AM - GM
Vi a
1
, a
2
, . . . , a
n
l cc s thc khng m th
a
1
+a
2
+ +a
n
n

n

a
1
a
2
a
n
ng thc xy ra khi v ch khi a
1
= a
2
= = a
n
.
15. Bt ng thc Cauchy - Schwarz
Vi a
1
, a
2
, . . . , a
n
v b
1
, b
2
, . . . , b
n
l cc s thc th
_
a
2
1
+a
2
2
+ +a
2
n
_ _
b
2
1
+b
2
2
+ +b
2
n
_
(a
1
b
1
+a
2
b
2
+ +a
n
b
n
)
2
ng thc xy ra khi v ch khi
a
1
b
1
=
a
2
b
2
= =
a
n
b
n
. Trong quy c nu mu bng 0 th t
bng 0 v ngc li.
16. Bt ng thc Nesbitt
Vi a, b, c l cc s thc dng th
a
b +c
+
b
c +a
+
c
a +b

3
2
ng thc xy ra khi v ch khi a = b = c.
9
Phn hai. Tuyn tp cc bi ton
I. bi
1. Cc bi ton n tp tuyn sinh lp 10
Bi 1.1. Tam gic ABC vung ti A c BC = 2AB. Ly D, E nm trn AC, AB sao cho

ABD =
1
3

ABC v

ACE =
1
3

ACB. F l giao im ca BD, CE. H, K l im i xng ca


F qua AC, BC.
(a) Chng minh H, D, K thng hng.
(b) Chng minh tam gic DEF cn.
Bi 1.2. ng trn (O) ni tip tam gic ABC(AB > AC) tip xc vi AB, AC ti P, Q. Gi
R, S ln lt l trung im BC, AC. Giao im ca PQ, RS l K. Chng minh rng B, O, K
thng hng.
Bi 1.3. Cho tam gic ABC nhn nhn H lm trc tm. Chng minh rng, ta c bt ng
thc :
HA +HB +HC <
2
3
(AB +BC +CA)
Bi 1.4. Gi AB l mt dy cung c nh ca ng trn (O). P l im di ng trn dy
cung AB nhng khng trng vi hai u mt. V ng trn (C) i qua A, P tip xc trong
vi (O) v ng trn (D) i qua B, P tip xc trong vi (O). Ly N l giao im th 2 ca
(C), (D).
(a) Chng minh rng ANB CPD. T hy ch ra N di ng trn ng no.
(b) Chng minh rng NP lun i qua mt im c nh.
Bi 1.5. Cho tam gic ABC c

BAC = 120

v cc ng phn gic AA

, BB

, CC

. Tnh

.
Bi 1.6. Cho hnh vung ABCD c hai ng cho ct nhau ti E. Mt ng thng i qua
A ct cnh BC M v ct ng thng CD N. Gi K l giao im ca EM v BN. Chng
minh rng CK BN.
Bi 1.7. Cho ABC c

BAC = 90

(AB < AC). ng trn (O; r) ng knh AB v ng


trn (P; R) ng knh AC ct nhau D v A.
(a) Gi M l im chnh gia cung nh DC, AM ct (O) ti N, ct BC ti E. Chng minh
ABE cn v cc im O, N, P thng hng.
(b) Dng ng knh NQ ca (O). Chng minh Q, D, M thng hng.
(c) Gi K l trung im MN. Chng minh PK OK.
Bi 1.8. Tam gic ABC nhn c 3 ng cao AA
1
, BB
1
, CC
1
ct nhau ti trc tm H. Gi
H
a
, H
b
, H
c
ln lt l trc tm ca cc tam gic AB
1
C
1
, BC
1
A
1
, CA
1
B
1
, hy chng minh rng
10
A
1
B
1
C
1
= H
a
H
b
H
c
.
Bi 1.9. Cho dy cung AB c nh trn (O) v

AOB = 120

. M l mt im di ng trn
cung ln AB, ng trn ni tip tam gic MAB tip xc vi MA, MB ti E, F. Chng minh
rng EF lun tip xc vi mt ng trn c nh.
Bi 1.10. Cho ng trn (O) v ng thng d nm ngoi ng trn. Gi S l hnh chiu
vung gc ca O ln d. V cc ct tuyn SAB, SEF. AF, BE ln lt ct d ti C, D. Chng
minh S l trung im ca CD.
Bi 1.11. Cho tam gic ABC vung ti A. K ng cao AH v ng phn gic BE ca
tam gic ABC (H BC, E AC). ng thng qua A vung gc vi BE ct BC, BE ln
lt ti M, N.
(a) Chng minh t gic ANHB ni tip mt ng trn. Gi ng trn l (O).
(b) ng thng CN ct (O) ti T (T ,= N). Chng minh rng : CH BC = CN CT.
(c) Gi I l giao im ca ON v AH. Chng minh rng :
1
4HI
2
=
1
AB
2
+
1
AC
2
.
Bi 1.12. Cho tam gic ABC ni tip ng trn (O; R) c ng cao AD. Gi E l hnh
chiu ca B trn AO, K l trung im ca BC, I l tm ng trn ngoi tip t gic ABDE.
Chng minh rng IK l ng trung trc ca DE.
Bi 1.13. Cho tam gic ABC nhn ni tip ng trn (O). Cc ng cao AD, BE, CF ct
nhau ti H.
(a) K ng knh AA

ca (O), I l trung im ca BC. Chng minh rng ba im H, I, A

thng hng.
(b) Gi G l trng tm tam gic ABC. Chng minh rng S
AHG
= 2S
AOG
.
Bi 1.14. Cho M l mt im nm bn trong hnh bnh hnh ABCD. Khi , hy chng minh
bt ng thc
MA MC +MB MD AC BC
Bi 1.15. Cho ng trn (O; R), ng knh BC. A l im di ng trn na ng trn (A ,=
B, C). Trn na ng trn kia ly I l im chnh gia cung BC. Dng AH BC ti H. Gi
(O
1
; R
1
); (O
2
; R
2
); (O
3
; R
3
) ln lt l cc ng trn ni tip cc tam gic ABH, ACH, ABC.
(a) Chng minh AI O
1
O
2
.
(b) HO
1
ct AB ti E, HO
2
ct AC ti F. Chng minh O
1
O
2
H ABC.
(c) Tm v tr im A R
1
+R
2
+R
3
ln nht.
Bi 1.16. Cho na ng trn tm O ng knh AB = 2R. C l mt im trn na ng
trn (C ,= A, B). Dng CH AB ti H. E, F ln lt l hnh chiu ca H trn CA, CB.
(a) Chng minh EF song song vi tip tuyn ti C ca (O).
(b) Chng minh t gic ABFE ni tip.
11
(c) Tm v tr im C chu vi v din tch tam gic ABC ln nht.
(d) Chng minh khi C di ng, tm I ca ng trn ni tip OCH di chuyn trn ng
c nh.
Bi 1.17. Cho hnh vung ABCD c nh, cnh a. E l im di chuyn trn cnh CD. ng
thng AE v BC ct nhau ti F. ng thng vung gc vi AE ti A ct ng thng CD
ti K.
(a) Chng minh AF(CK CF) = BD FK.
(b) Chng minh rng trung im I ca KF di ng trn mt ng thng c nh khi E di
ng trn CD.
(c) Ch ra v tr ca E di EK ngn nht.
Bi 1.18. Cho tam gic ABC u. Gi D l im di ng trn cnh BC. Gi (I
1
; R
1
); (I
2
; R
2
); (I
3
; R
3
)
ln lt l cc ng trn ni tip ca cc tam gic ABD, ACD, ABC v (I
3
; R) l ng trn
ngoi tip tam gic ABC. Tia AD ct (I
3
; R) ti E.
(a) Chng minh
1
ED
=
1
EB
+
1
EC
.
(b) Tm v tr ca E
1
ED
+
1
EB
+
1
EC
nh nht. Chng minh khi y S
ABEC
ln nht.
(c) Tm v tr im D R
1
+R
2
ln nht.
Bi 1.19. Cho (O; R) v mt im M nm ngoi ng trn. T M dng hai tip tuyn
MA, MB i vi (O; R). Gi E l trung im ca BM; H l giao im ca OM vi AB. on
thng AE ct (O; R) ti C.
(a) Chng minh t gic HCEB ni tip.
(b) Chng minh EMC EAM.
(c) MC ct (O) ti D. Tnh DB theo R bit OM = 3R.
(d) OB ct (O) ti T v ct AD ti S. MT giao SA ti N. Chng minh N l trung im
AS.
Bi 1.20. Cho hnh vung ABCD cnh a. E l im di ng trn cnh AD (E ,= A). Tia
phn gic ca

EBA,

EBC ct DA, DC ti M, N.
(a) Chng minh BE MN.
(b) Tm v tr im E S
DMN
ln nht.
12
Bi 1.21. Cho ABC. Mt ng trn (O) qua A v B ct AC v BC D v E. M l giao
im th hai ca cc ng trn ngoi tip cc tam gic ABC v DEC. Chng minh rng

OMC = 90

.
Bi 1.22. Cho hnh thoi ABCD c

ABC = 60

. Mt ng thng qua D khng ct hnh thoi


nhng ct cc ng thng AB, BC ln lt ti E, F. Gi M l giao im ca AF v CE.
Chng minh rng AD tip xc vi ng trn ngoi tip tam gic MDF.
Bi 1.23. Cho ng trn (O) v dy AD. Gi I l im i xng vi A qua D. K tip tuyn
IB vi ng trn (O). Tip tuyn vi ng trn (O) ti A ct IB K. Gi C l giao im
th hai ca KD vi ng trn (O). Chng minh rng BC song song vi AI.
Bi 1.24. Cho ABC ni tip ng trn tm O v ngoi tip ng trn tm I . AI, BI, CI
ct (O) ln lt ti D, E, F. DE ct CF ti M, DF ct BE ti N.
(a) Chng minh rng MN | BC.
(b) Gi Q l tm ng trn ngoi tip DMN, P l giao im ca AD v EF . Chng
minh cc im M, N, P, Q cng nm trn mt ng trn.
Bi 1.25. Cho ABC c nh, M l im di ng trn cnh BC. Dng ng knh BE ca
ng trn ngoi tip ABM v ng knh CF ca ng trn ngoi tip ACM. Gi N
l trung im EF. Chng minh rng khi M di ng trn BC th N di ng trn mt ng
thng c nh.
Bi 1.26. Cho tam gic ABC c

BAC = 135

, AB = a, AC = b. im M nm trn cnh BC
sao cho

BAM = 45

. Tnh di AM theo a, b.
Bi 1.27. Cho hnh vung ABCD, ly im M nm trong hnh vung sao cho

MAB =

MBA = 15

. Hi tam gic MCD l tam gic g? Ti sao?


Bi 1.28. Cho t gic ABCD ni tip (O; R) sao cho tia BA v tia CD ct nhau ti I, cc
tia DA v CB ct nhau K (I, K nm ngoi (O)). Phn gic ca gc

BIC ct AD, BC ln
lt ti Q, N. Phn gic ca gc

AKB ct AB, AC ln lt ti M, P.
(a) Chng minh t gic MNPQ l hnh thoi.
(b) Chng minh IK
2
= ID IC +KB KC.
(b) Gi F l trung im ca AB, J l hnh chiu ca F trn OB, L l trung im ca FJ.
Chng minh AJ OL.
Bi 1.29. Cho t gic ABCD ni tip (O) c hai ng cho AC, BD ct nhau ti M. ng
vung gc vi OM ti M ct AB, BC, CD, DA ln lt ti M
1
, M
2
, M
3
, M
4
. Chng minh
M
1
M
4
= M
2
M
3
.
Bi 1.30. Cho t gic li ABCD vi E, F l trung im ca BD v AC. Chng minh rng
AB
2
+CD
2
+BC
2
+DA
2
= 4EF
2
+AC
2
+BD
2
Bi 1.31. Trn (O; R) ly hai im B, C c nh sao cho BC =

3R. A l mt im trn cung


ln BC (A ,= B; C).
13
(a) Chng minh khi A di ng, phn gic

BAC lun i qua mt im c nh I.


(b) Gi E, F ln lt l hnh chiu ca I trn cc ng thng AB, AC. Chng minh BE =
CF.
(c) Chng minh khi A di ng th EF lun i qua mt im c nh.
(d) Tm v tr dim A S
AEIF
ln nht. Tnh gi tr ln nht theo R.
Bi 1.32. Cho (O; R) v im A c nh vi OA > R. Dng ct tuyn AMN ca (O) khng
qua tm (AM < AN). Chng minh rng
(a) ng trn ngoi tip OMN lun i qua mt im c nh H (H khng trng O) khi
ct tuyn di ng.
(b) Tip tuyn ti M v N ca (O) ct nhau ti T. Chng minh T di ng trn mt ng
thng c nh khi ct tuyn AMN di ng.
Bi 1.33. Cho ABC c

BAC = 60

, AC = b, AB = c (b > c). ng knh EF ca ng


trn ngoi tip tam gic ABC vung gc vi BC ti M. I v J l chn ng vung gc h
t E xung AB; AC; H v K l chn ng vung gc h t F xung AB; AC.
(a) Chng minh IJ HK.
(b) Tnh bn knh ng trn ngoi tip tam gic ABC theo b v c.
(c) Tnh AH +AK theo b v c.
Bi 1.34. Cho tam gic ABC. Mt im D di ng trn cnh BC. Gi P, Q tng ng l tm
ng trn ni tip ca cc tam gic ABD, ACD. Chng minh rng khi D di ng th ng
trn ng knh PQ lun i qua mt im c nh.
Bi 1.35. Cho tam gic ABC c phn gic AD v trung tuyn AM. ng trn ngoi tip
tam gic ADM ct AB ti E v AC ti F. Gi L l trung im EF. Xc nh v tr tng i
ca hai ng thng ML v AD.
Bi 1.36. Cho BC l dy cung ca (O; R). t BC = aR. im A trn cung BC ln, k cc
ng knh CI, BK. t S =
AB +AC
AI +AK
. Chng minh rng S =
2 +

4 a
2
a
. T tm gi
tr nh nht ca S.
Bi 1.37. Cho tam gic ABC ni tip (O, R) c

BAC 90

. Cc ng trn (A; R
1
), (B; R
2
),
(C; R
3
) i mt tip xc ngoi vi nhau.
Chng minh rng
S
ABC
=
BC R
2
1
+AC R
2
2
+AB R
2
3
+ 2R
1
R
2
R
3
4R
Bi 1.38. Cho hnh thoi ABCD c cnh l 1. Trn cnh BC ly M, CD ly N sao cho chu vi
CMN bng 2 v 2

NAM =

DAB. Tnh cc gc ca hnh thoi.


Bi 1.39. V pha ngoi ca tam gic ABC dng cc hnh vung BCMN, ACPQ c tm O
v O

.
14
(a) Chng minh rng khi c nh hai im A, B v cho C thay i th ng thng NQ lun
i qua mt im c nh.
(b) Gi I l trung im ca AB. Chng minh IOO

l tam gic vung cn.


Bi 1.40. Cho hai ng trn (O; R) v (O

; R

) ngoi nhau bit OO

= d > R + R

. Mt
tip tuyn chung trong ca hai ng trn tip xc vi (O) ti E v tip xc vi (O

) ti F.
ng thng OO

ct (O) ti A, B v ct (O

) ti C, D (B, C nm gia A, D). AE ct CF ti


M, BE ct DF ti N. Gi giao im ca MN vi AD l I. Tnh di OI.
Bi 1.41. Cho tam gic ABC c din tch S
0
. Trn cc cnh BC, CA, AB ly cc im M, N, P
sao cho
MB
MC
= k
1
,
NC
NA
= k
2
,
PA
PB
= k
3
(k
1
, k
2
, k
3
< 1).
Hy tnh din tch tam gic to bi cc on thng AM, BN, CP.
2. Cc bi ton n tp Olympiad
Bi 2.1. (APMO 2000) Cho tam gic ABC vi trung tuyn AM v phn gic AN. ng
thng vung gc vi AN ti N ct AB, AM ln lt ti P, Q. ng thng vung gc vi AB
ti P ct ng thng AN ti O. Chng minh rng OQ vung gc vi BC.
Bi 2.2. (D tuyn IMO 1994) Tam gic ABC khng cn ti A c D, E, F l cc tip im
ca ng trn ni tip ln BC, CA, AB. X l im bn trong tam gic ABC sao cho ng
trn ni tip tam gic XBC tip xc vi BC ti D, v tip xc vi XB, XC ti Y, Z. Chng
minh rng E, F, Y, Z ng vin.
Bi 2.3. Dng hnh vung DEFG ni tip tam gic ABC sao cho D, E BC; F AC; G
AB. Gi d
A
l trc ng phng ca hai ng trn (ABD), (ACE). Ta nh ngha cc ng
thng d
B
, d
C
tng t. Chng minh rng cc ng thng d
A
, d
B
, d
C
ng quy.
Bi 2.4. Cho tam gic ABC vi trng tm G. Mt ng thng d i qua G ct BC, CA, AB
ln lt ti M, N, P. Chng minh rng, ta c ng thc :
1
GM
+
1
GN
+
1
GP
= 0
Bi 2.5. Cho t gic ABCD ni tip ng trn (O) c cc cnh i khng song song v cc
ng cho ct nhau ti E. F l giao im ca AD vi BC. M, N ln lt l trung im ca
AB, CD. Chng minh rng EF l tip tuyn ca ng trn ngoi tip tam gic EMN.
Bi 2.6. Cho tam gic ABC vi ng trn ni tip (I) v E, F l cc tip im ca (I) vi
CA, AB. Ly K bt k thuc on EF, gi H, L l giao im ca BK, CK vi AC, AB tng
ng. Chng minh rng HL tip xc vi (I).
Bi 2.7. Gi BH, BD ln lt l ng cao v phn gic ca tam gic ABC. N, L, M ln
lt l trung im ca BH, BD, AC. Ly K l giao im ca MN v BD. Chng minh rng,
AL, AK l hai ng ng gic trong gc

BAC.
Bi 2.8. Cho tam gic ABC vung ti A. Trn cc tia AB, AC ly E, F tng ng sao cho
BE = BC = CF. Chng minh rng vi mi im M nm trn ng trn ng knh BC, ta
u c
MA +MB +MC EF
15
Bi 2.9. Cho tam gic ABC c BC = a, CA = b, AB = c v I l tm ng trn ni tip tam
gic ABC. Chng minh rng
IA +IB +IC

ab +bc +ca
Bi 2.10. T im A nm ngoi ng trn (O), k hai tip tuyn AB, AC n (O). Gi E, F
l trung im ca AB, AC. Ly D l mt im bt k trn EF, v cc tip DP, DQ ti ng
trn. PQ ct BC, EF ln lt ti N, M. Chng minh rng, ON | AM.
Bi 2.11. Cho tam gic ABC cn ti A ni tip ng trn (O). Trn cnh y BC, ly im
M (M khc B, C). V ng trn tm D qua M tip xc vi AB ti B v ng trn tm E
qua M tip xc vi AC ti C. Gi N l giao im th hai ca hai ng trn ny.
(a) Chng minh rng tng bn knh ca hai ng trn (D), (E) l khng i khi M di ng
trn BC.
(b) Tm tp hp trung im I ca DE.
Bi 2.12. Cho M l im di ng trn ng trn (O, r) c hai ng knh c nh AB, CD
vung gc vi nhau. Gi I l hnh chiu ca M ln CD v P l giao im ca OM, AI. Tm
tp hp cc im P.
Bi 2.13. Cho tam gic u ABCv mt im M bt k trong mt phng tam gic. Gi
x, y, z l khong cch t M n cc nh A, B, C v p, q, r l khong cch t M n cc cnh
AB, BC, CA. Chng minh rng :
p
2
+q
2
+r
2

1
4
(x
2
+y
2
+z
2
)
Bi 2.14. Cho a gic u A
1
A
2
A
3
A
4
A
5
A
6
A
7
v im M bt k trong mt phng. Chng minh
rng
MA
1
+MA
3
+MA
5
+M
7
MA
2
+MA
4
+MA
6
Bi 2.15. Tam gic ABC khng cn ni tip (O) c A
1
, B
1
, C
1
l trung im ca BC, CA, AB.
Gi A
2
l mt im trn tia OA
1
sao cho 2 tam gic OAA
1
v OA
2
A ng dng. Cc im
B
2
, C
2
nh ngha tng t. Chng minh rng AA
2
, BB
2
, CC
2
ng quy.
Bi 2.16. Cho tam gic ABC vi M l trung im BC. V ng trn (O) ty qua A v
ct cc on AB, AC, AM ln lt ti B
1
, C
1
, M
1
. Chng minh rng,
AB
1
AB +AC
1
AC = 2AM
1
AM
Bi 2.17. Cho tam gic ABC ni tip ng trn bn knh R.Gi q l chu vi tam gic c cc
nh l tm cc ng trn bng tip tam gic ABC. Chng minh rng :
q 6

3R
Bi 2.18. Cho tam gic ABC c : BC = a; CA = b; AB = c; v r v R theo th t l bn
knh ng trn ni tip v ngoi tip tam gic ABC. Chng minh rng
r
R
+
(a b)
2
+ (b c)
2
+ (c a)
2
16R
2

1
2
16
Bi 2.19. Cho tam gic ABC. Cc ng phn gic BE, CF ct nhau ti I. AI ct EF ti
M. ng thng qua M song song vi BC theo th t ct AB, AC ti N, P. Chng minh rng
MB +MC < 3NP
Bi 2.20. Cho tam gic ABC nhn vi ng cao CF v CB > CA. Gi O, H ln lt l tm
ngoi tip v trc tm ca tam gic ABC. ng thng qua F vung gc vi OF ct AC ti
P. Chng minh rng

FHP =

BAC.
Bi 2.21. Cho ng trn (O; R) v mt im P c nh bn trong ng trn. AB, CD l 2
dy cung di ng ca (O) nhng lun i qua P v lun vung gc vi nhau.
(a) Chng minh rng PA
2
+PB
2
+PC
2
+PD
2
khng i.
(b) Gi I l trung im BC. Hi I di ng trn ng no?
Bi 2.22. Cho tam gic ABC v im M bt k nm trong tam gic . Chng minh rng :
MA +MB +MC + minMA, MB, MC < AB +BC +CA
Bi 2.23. Tam gic cn ABC ni tip (O) c AB = AC v AQ l ng knh ca (O). Ly
M, N, P ln lt trn cnh AB, BC, CA sao cho AMNP l hnh bnh hnh. Chng minh rng
NQ MP.
Bi 2.24. Cho t gic ABCD c M, N ln lt l trung im AB, CD v O l giao im ca
2 ng cho. Gi H, K l trc tm ca tam gic OAB, OCD. Hy chng minh MN HK.
Bi 2.25. Cho t gic ABCD ni tip (O) c hai ng cho ct nhau ti I. Gi M, N ln lt
l trung im ca AB, CD. P, Q l chn ng cao k t I ca tam gic IAD, IBC. Chng
minh rng, PQ MN.
Bi 2.26. Cho tam gic ABC v tam gic DBC c tm ni tip ln lt l H, K. Chng minh
rng AD HK.
Bi 2.27. Cho K l im nm trong tam gic ABC. Mt ng thng qua K ct hai cnh
AB, AC theo th t M, N. Chng minh rng :
S
ABC
8
_
S
BMK
S
CNK
Bi 2.28. Cho tam gic ABC nhn v M l mt im thuc min trong tam gic. Gi A
1
, B
1
, C
1
ln lt l giao im ca MA, MB, MC vi cc cnh tam gic ABC. Ly A
2
, B
2
, C
2
l cc
im i xng vi M qua trung im ca B
1
C
1
, C
1
A
1
, A
1
B
1
. Chng minh rng AA
2
, BB
2
, CC
2
ng quy.
Bi 2.29. Cho tam gic ABC ni tip (O; R) c M thuc cung BC khng cha A. Tm v tr
ca M P = 2010 MB + 2011 MC t gi tr ln nht.
Bi 2.30. Cho tam gic ABC. Cc im D, E, F nm trn cc cnh BC, CA, AB sao cho
AD, BE, CF ng quy ti O. Qua O k ng thng song song vi BC ct DE, DF theo th
t ti H v K. Chng minh O l trung im HK.
Bi 2.31. Cho tam gic ABC. M l mt im bt k trn mt phng v khng nm trn
17
tam gic ABC. Cc ng thng AM, BM, CM ln lt ct cc ng thng BC, CA, AB ti
D, E, F. Gi H, K ln lt l giao im ca cc cp ng thng BM vi FD; CM vi ED.
Chng minh cc ng thng AD, BK, CH ng quy.
Bi 2.32. Cho t gic li ABCD. Chng minh :
minAB, BC, CD, DA

AC
2
+BD
2
2
maxAB, BC, CD, DA
Bi 2.33. Cho ng trn (O; R) v hai im A, B c nh i xng vi nhau qua O. Gi M
l im chy trn (O). ng thng MA, MB ct (O) ti P, Q tng ng. Chng minh rng
gi tr biu thc
MA
AP
+
MB
BQ
khng i khi M di chuyn trn (O).
Bi 2.34. Cho (O) v dy AB. im M di chuyn trn cung ln AB. Cc ng cao AE, BF
ca ABM ct nhau ti H. K (H; HM) ct MA, MB C v D. Chng minh ng thng
k t H vung gc vi CD lun i qua mt im c nh khi M di chuyn trn cung ln AB.
Bi 2.35. Cho tam gic ABC ni tip ng trn (O). G l trng tm tam gic. AG, BG, CG
ln lt ct (O) ti A
1
, B
1
, C
1
. Chng minh rng :
GA
1
+GB
1
+GC
1
GA +GB +GC
Bi 2.36. Cho ABC v D, E, F ln lt l hnh chiu ca A, B, C xung ba cnh tng ng.
ng thng qua D song song vi EF ct AB, AC ti P, Q. Bit EF BC = R. Chng minh
rng ng trn ngoi tip PQR i qua trung im BC.
Bi 2.37. Cho t gic li ABCD ni tip ng trn (O). Cho AB = a, CD = b,

AIB = ,
trong I l giao im ca hai ng cho AC v BD. Tnh bn knh ng trn (O) theo
a, b v .
Bi 2.38. Cho ABC c trc tm H. ng trn qua B, C ct AB, AC ti D, E. Gi F l
trc tm ADE v I l giao im ca BE v CD. Chng minh rng I, H, F thng hng.
Bi 2.39. Cho ABC khng cn, ngoi tip ng trn (I). Tip im ca (I) trn BC, CA, AB
ln lt l D, E, F. DE ct AB P. Mt ng thng qua C ct AB, FE ln lt N, M.
PM ct AC Q. Chng minh rng IN vung gc vi FQ.
Bi 2.40. Cho t gic ABCD. Gi I, J theo th t l trung im ca AC, BD. Chng minh
rng :
AC +BD + 2IJ < AB +BC +CD +DA
Bi 2.41. Cho ABC ni tip ng trn (O). E thuc cung BC khng cha A v khng
trng B, C. AE ct tip tuyn ti B, C ca (O) ti M, N. Gi giao im ca CM v BN l
F. Chng minh rng EF lun i qua mt im c nh khi E di chuyn trn cung BC khng
cha A.
Bi 2.42. Cho t gic ABCD ni tip tha mn AB CD = AD BC. ng trn (C) qua
A, B v tip xc vi BC, ng trn (C

) qua A, D v tip xc CD. Chng minh rng giao


im khc A ca (C) v (C

) l trung im BD.
Bi 2.43. Cho tam gic nhn ABC, gi H l trc tm ca tam gic. Tm iu kin cn v
18
i vi cc gc ca tam gic 9 im : chn cc ng cao ca tam gic, trung im cc cnh
ca tam gic, trung im cc on thng HA, HB, HC l nh ca mt a gic u.
Bi 2.44. Cho tam gic ABC. ng trn (I) ni tip tam gic ABC v tip xc vi
BC, AC, AB ln lt ti D, E, F. Chng minh rng ID, EF v trung tuyn AM (M BC)
ng quy.
Bi 2.45. Cho hai on thng AB v A

bng nhau. Php quay tm M bin A thnh A

,
bin B thnh B

. Php quay tm N bin A thnh B

, bin B thnh A

. Gi S l trung im
ca AB. Chng minh rng SM vung gc vi SN.
Bi 2.46. Cho tam gic ABC, M l im nm trong tam gic. AM, BM, CM ct BC, CA, AB
theo th t D, E, F. Gi H, I, K theo th t l hnh chiu ca M trn BC, CA, AB . K hiu
P(HIK) l chu vi tam gic HIK. Hy chng minh :
P(DEF) P(HIK)
Bi 2.47. Tam gic ABC nhn ni tip (O), ng cao AH ct (O) ti A

. OA

ct BC ti
A

. Xc nh tng t cho B

, C

. Chng minh AA

, BB

, CC

ng quy.
Bi 2.48. Cho ng trn (O) v mt ng thng d c nh. Gi H l hnh chiu ca ca O
trn d. Ly M c nh thuc ng trn. A, B thay i trn d sao cho H l trung im AB.
Gi s AM, BM ct (O) ti P, Q. Chng minh PQ lun i qua mt im c nh.
Bi 2.49. Cho ng trn tm I ni tip tam gic ABC tip xc vi BC, AB, AC ti D, E, F.
Qua E v ng song song vi BC ct AD, DF M, N. Chng minh rng M l trung im
ca EN.
Bi 2.50. Cho tam gic ABC c AB = c, BC = a, AC = b v I l tm ng trrn ni tip.
Hai im B

, C

ln lt nm trn hai cnh AB, AC sao cho B

, C

, I thng hng. Chng minh


rng
S
ABC

a +b +c
2

bc

_
S
AB

C
S
ABC

Bi 2.51. Cho t gic ABCD ni tip. E, F, G, H ln lt l tm ng trn ni tip cc tam


gic ABC, BCD, CDA, DAB. Chng minh rng t gic EFGH ni tip.
Bi 2.52. Cho hnh vung ABCD. I ty thuc AB, DI ct BC ti E, CI ct AE ti F.
Chng minh rng BF DE.
Bi 2.53. Cho tam gic ABC khng vung ni tip ng trn (O), trc tm H. d l ng
thng bt k qua H. Gi d
a
,d
b
, d
c
ln lt l cc ng thng i xng vi d qua BC, CA, AB.
Chng minh rng d
a
, d
b
, d
c
ng quy ti mt im trn (O).
Bi 2.54. Cho hnh thang ABCD (AB | CD). AC ct CD ti O. Bit khong cch t O n
AD v BC bng nhau, hy chng minh rng ABCD l hnh thang cn.
Bi 2.55. Cho tam gic ABC cn ti A. ng trn tip xc AB, AC, ct BC ti K. AK
ct ti im th hai l M. P, Q l im i xng ca K qua B, C. Chng minh rng ng
trn ngoi tip tam gic MPQ tip xc vi .
Bi 2.56. Cho tam gic ABC vung ti A c

B = 20

, phn gic trong BI. im H nm trn


19
cnh AB sao cho

ACH = 30

. Hy tnh s o

CHI.
Bi 2.57. Cho tam gic ABC ngoi tip (I). Gi D, E, F ln lt l im i xng vi I qua
BC, CA, AB. Chng minh rng AD, BE, CF ng quy.
Bi 2.58. Cho tam gic ABC cn ti A ni tip (O). im M l trung im ca AC. BM ct
li (O) ti im th hai l Q. Chng minh rng 2AQ BQ.
Bi 2.59. Cho ABC tha mn AB +BC = 3CA. ng trn ni tip (I) tip xc AB, BC
ti D, E. Gi K, L tng ng i xng vi D, E qua I. Chng minh rng t gic ACKL ni
tip.
Bi 2.60. Cho tam gic ABC ngoi tip (I). (I) tip xc BC, CA, AB ln lt ti D, E, F.
Chng minh rng tm ng trn ngoi tip cc tam gic AID, BIE, CIH thng hng.
Bi 2.61. Cho tam gic ABC ni tip (O). M, N ln lt l im chnh gia cung AB khng
cha C v cung AC khng cha B. D l trung im MN. G l mt im bt k trn cung BC
khng cha A. Gi I, J, K ln lt l tm ni tip cc tam gic ABC, ABG, ACG. Ly P l
giao im th hai ca (GJK) vi (ABC). Chng minh rng P DI.
Bi 2.62. Cho n gic u A
1
A
2
. . . A
n
(n 4) tha mn iu kin
1
A
1
A
2
=
1
A
1
A
3
+
1
A
1
A
4
Hy tm n.
Bi 2.63. Gi AA
1
, BB
1
, CC
1
tng ng l cc ng phn gic trong ca tam gic ABC.
AA
1
, BB
1
, CC
1
ct ng trn ngoi tip tam gic ti A
2
, B
2
, C
2
theo th t. Chng minh
rng :
AA
1
AA
2
+
BB
1
BB
2
+
CC
1
CC
2

9
4
Bi 2.64. Cho tam gic ABC, ng thng d ct cc ng thng BC, CA, AB ln lt ti
D, E, F. Gi O
1
, O
2
, O
3
ln lt l tm ng trn ngoi tip cc tam gic AEF, BDF, CDE.
Chng minh rng trc tm tam gic O
1
O
2
O
3
nm trn d.
Bi 2.65. Cho t gic ABCD, AC ct BD ti O. Gi M, N, P, Q ln lt l hnh chiu ca
O trn AB, BC, CD, DA. Bit rng OM = OP, ON = OQ. Chng minh rng ABCD l hnh
bnh hnh.
Bi 2.66. Cho tam gic ABC, phn gic trong AD(D BC). Gi M, N l cc im thuc
tia AB, AC sao cho

MDA =

ABC,

NDA =

ACB. Cc ng thng AD, MN ct nhau ti P.


Chng minh rng :
AD
3
= AB AC AP
Bi 2.67. Trn mt phng cho 2000 ng thng phn bit, i mt ct nhau. Chng minh
rng tn ti t nht 2 ng thng m gc ca chng khng ln hn
180
2000
().
Bi 2.68. Cho t gic ABCD ni tip (O) c AB = AD. M, N nm trn cc cnh BC, CD
sao cho MN = BM +DN. AM, AN ct (O) ti P, Q.
Chng minh rng trc tm tam gic APQ nm trn MN.
Bi 2.69. Cho t gic ABCD. Hai ng cho AC, BD ct nhau ti O. Gi r
1
, r
2
, r
3
, r
4
ln
20
lt l bn knh cc ng trn ni tip cc tam gic AEB, BEC, CED, DEA.
Chng minh rng
1
r
1
+
1
r
3
=
1
r
2
+
1
r
4
l iu kin cn v t gic ABCD ngoi tip c mt ng trn.
Bi 2.70. Cho tam gic ABC c M l trung im ca BC v H l trc tm tam gic. ng
thng vung gc vi HM ti H ct AB, AC ti D, E. Chng minh rng H l trung im ca
DE.
Bi 2.71. Cho on thng AB = a c nh. im M di ng trn AB (M khc A, B). Trong
cng mt na mt phng b l ng thng AB dng hinh vung AMCD v MBEF. Hai
ng thng AF, BC ct nhau N.
Tm v tr im M sao cho on MN c di ln nht.
Bi 2.72. Cho tam gic ABC nhn khng cn, ni tip (O). Cc ng cao AA
0
, BB
0
, CC
0
ng quy ti H. Cc im A
1
, A
2
thuc (O) sao cho ng trn ngoi tip cc tam gic
A
1
B
0
C
0
, A
2
B
0
C
0
tip xc trong vi (O) ti A
1
, A
2
. B
1
, B
2
, C
1
, C
2
xc nh tng t.
Chng minh rng B
1
B
2
, C
1
C
2
, A
1
A
2
ng quy ti mt im trn OH.
Bi 2.73. Cho ng trn (I) ni tip tam gic ABC tip xc BC, CA, AB ti A
1
, B
1
, C
1
.
Cc ng thng IA
1
, IB
1
, IC
1
tng ng ct cc on thng B
1
C
1
, C
1
A
1
, A
1
B
1
ti A
2
, B
2
, C
2
.
Chng minh cc ng thng AA
2
, BB
2
, CC
2
ng quy.
Bi 2.74. Cho tam gic ABC cn ti A. Trn tia i ca tia CA ly im E. Giao im ca
BE v phn gic gc

BAC l D. Mt ng thng qua D song song AB ct BC F. AF ct


BE ti M. Chng minh rng M l trung im BE.
Bi 2.75. Cho t gic li ABCD sao cho AB ko song song vi CD v im X bn trong t
gic tha

ADX =

BCX < 90

DAX =

CBX < 90

. Gi Y l giao im ng trung trc


ca AB v CD. Chng minh rng

AY B = 2

ADX.
Bi 2.76. Cho t gic li ABCD ni tip trong (O). AD ct BC ti E, AC ct BD ti F.M, N
l trung im AB, CD. Chng minh rng :
2MN
EF
=

AB
CD

CD
AB

Bi 2.77. Cho t gic ABCD ni tip c mt ng trn. Chng minh rng :


AC
BD
=
DA AB +BC CD
AB BC +CD DA
Bi 2.78. Cho tam gic nhn ABC ni tip (O; R).Gi R
1
, R
2
, R
3
tng ng l bn knh ng
trn ngoi tip cc tam gic OBC, OCA, OAB. Chng minh rng :
R
1
+R
2
+R
3
3R
21
II. Hng dn v gi
1. Cc bi ton n tp tuyn sinh lp 10
Bi 1.1.
(a) Ta c

FHD = 20

, vic cn li ch l kim tra

FHK = 20

.
(b) Gi I l giao im ca HK, BC. Ln lt chng minh cc kt qu sau

DFI = 120

BEFI ni tip

EFI = 120

FIE = 20

DIF
DFI = EFI
Kt qu cui chng t tam gic EFD cn ti F.
Bi 1.2.
Vi ch rng SK = SQ, s dng cc bin i di on thng ch ra rng RK = RB.
Bi 1.3.
Qua H dng cc ng thng song song vi cc cnh tam gic v cc giao im i vi cc
cnh cn li. Hy ch cc hnh bnh hnh to c v s dng bt ng thc tam gic, ta s
c iu cn chng minh.
Bi 1.4.
(a) T hai tam gic ng dng ANB, CPD suy ra

ANB khng i. T rt ra c qu
tch im N.
(b) im c nh cn tm chnh l giao im tip tuyn ti A, B ca O.
Bi 1.5.
Hy chng minh rng B

l tm bng tip trong gc B ca tam gic AA

B v C

l tm bng
tip trong gc C ca tam gic AA

C t suy ra

= 90

.
Bi 1.6.
Gi S l giao im ca EM, CD. p dng nh l Menelaus cho hai tam gic ACN, BCN v
nh l Thales rt ra :
BC
2
NC
2
=
KB
KN
ng thc ny chng t tam gic vung BCN nhn K lm chn ng cao k t C.
Bi 1.7.
(a) Bng tnh cht ca tip tuyn v cc php bin i gc, hy chng minh

BAE =

BEA.
T suy ra N l trung im AE v O, N, P thng hng.
(b) Hy chng minh

MDN = 90

.
(c) Chng minh t gic OKPA ni tip.
Bi 1.8.
Hy chng minh A
1
B
1
H
a
H
b
l hnh bnh hnh nh b sau : Vi tam gic XY Z, trc tm
Q th QX = Y Z cot X.
22
Bi 1.9.
Gi N l trung im ca AB. ng trn c nh cn tm l
_
N,
AB
2
_
.
Bi 1.10.
chng minh kt qu ca bi ton, ta s ch ra rng OS l phn gic ca gc

COD bng
cch s dng cc tam gic ng dng v t gic ni tip.
Bi 1.11.
Hai (a) v (b) u l nhng kt qu n gin v quen thuc.
Vi (c), ta s chng minh AH = 2HI, sau p dng h thc lng trong tam gic vung
ABC.
Bi 1.12.
Bng cch bin i gc da vo cc t gic ni tip, hy chng minh rng IK l phn gic
trong ca gc DIE.
Bi 1.13.
(a) Hy chng minh BHCA

l hnh bnh hnh.


(b) Thc cht y l kt qu quen thuc v ng thng Euler : H, O, G thng hng v
HG = 2OG.
Bi 1.14.
Dng thm hnh bnh hnh ABMT. T hy p dng bt ng thc Ptolemy cho t gic
AMDT vi ch cc on thng bng nhau suy ra iu cn chng minh.
Bi 1.15.
(a) Hy chng minh (O
3
) l trc tm ca AO
1
O
2
.
(b) Da vo cc tam gic ng dng, ta suy ra ng thc
O
1
H
O
2
H
=
BH
AH
=
AB
AC
T suy ra O
1
HO
2
BAC.
(c) S dng kt qu sau
_

_
R
3
=
AB +AC BC
2
R
2
=
AH +CH AC
2
R
1
=
AH +BH AB
2
Bi 1.16.
(a) C 2 cch chng minh c bn nht cho kt qu ny:
V tip tuyn Cx ca O. Hy chng minh rng tip tuyn ny song song vi EF.
V ng knh CC

, gi giao im ca CC

, EF l Q. Hy chng minh BFQC

ni tip
suy ra kt qu.
(b) Suy ra trc tip t (a).
(c) Nhn xt CA
2
+CB
2
khng i nh gi chu vi v din tch ABC. Ngoi ra, cn mt
23
cch n gin hn nh gi din tch nh vo tnh cht : di ng trung tuyn tam
gic khng nh hn di ng cao xut pht cng mt nh.
(d) Khi C di ng trn cung AB th I lun di ng trn cung cha gc 135

dng trn on
OA hoc OB nm trn na mt phng b AB cha C (tr hai im A v B).
Bi 1.17.
(a) Trn tia CD ly im T sao cho AT = AC. Hy chng minh CK CF = CT.
(b) I BD c nh.
(c) p dng ng thc EK =
AE
2
DE
suy ra on EK ngn nht khi E C.
Bi 1.18.
(a) Chng minh tun t cc ng thc sau:
EA = EB +EC

1
ED
=
EA
EB EC
(b) p dng ng thc chng minh (a).
(c) Gi di cc cnh tam gic u ABC l a. Hy chng minh rng:
R
1
+R
2
=
(3a 2AD)R
3
a
Bi 1.19.
(d) Gi I l giao im ca AT, BM. Khi , chng minh tun t :
M l trung im BI.

SN
MB
=
TN
TM
=
AN
MI
Bi 1.20.
(a) Dng MI
1
BE ti I
1
. Hy chng minh M, I
1
, N thng hng.
(b) T (a). hy chng minh AM + CN = MN v suy ra gi tr ln nht ca S
DMN
t
c khi E D.
Bi 1.21.
Gi I, K ln lt l tm ca cc ng trn (CDE), (ABC). Dng ng knh CP ca (I).
Chng minh tun t cc kt qu sau:
PM CM
PO CM
M, O, P thng hng
Bi 1.22.
Chng minh tun t cc kt qu sau y:
FCD DAE
24
ACF EAC
ACM AFC
AM AF = AD
2
Bi 1.23.
Ch rng ADBC l t gic iu ha, hy tm cc ng thc v t s di on thng
c BDI BCA. T suy ra iu cn chng minh.
Bi 1.24.
(a) Hy chng minh INDM ni tip.
(b) Chng minh PN | AB, PM | AC. T suy ra t gic PNQM ni tip v c tng 2 gc
i l 180

.
Bi 1.25.
Gi H l trung im BC, N di ng trn ng thng vung gc vi AH ti A c nh.
Bi 1.26.
Ly N trn BC sao cho

BAM = 90

. p dng cng thc ng phn gic tnh di AN


theo AM, b; AM theo AN, a. T rt ra quan h gia AM vi a, b.
Bi 1.27.
Dng tam gic AME u (E nm trong tam gic ADM). T suy ra DM = DA = DC.
p s : MCD u.
Bi 1.28.
(a) Gi H l giao im ca KP v IN. Hy chng minh t gic MNPQ c hai ng cho
vung gc vi nhau ti trung im ca mi ng suy ra iu phi chng minh.
(b) Gi E l giao im ca ng trn ngoi tip tam gic ABK vi IK. Chng minh tun
t cc ng thc sau:
ID IC = IE IK
KB KC = KE IK
(c) Gi R l giao im ca AJ, OL. K AS BO (S BO). Ln lt chng minh:
J l trung im BS
OLF AJB
AFRO ni tip
AJ OL
Bi 1.29.
Bi ton ny l h qu trc tip ca nh l con bm. Hy chng minh rng M ng thi l
trung im ca cc on thng M
1
M
3
v M
2
M
4
Bi 1.30.
25
p dng cng thc di ng trung tuyn cho cc tam gic ACE, ABD, BCD.
Bi 1.31.
(c) Gi M l trung im BC th EF lun i qua M c nh.
(d) S
AEIF
max S
ABC
max.
Bi 1.32.
(a) ng trn ngoi tip OMN lun i qua im H AO c nh.
(b) T lun di ng trn ng thng vung gc vi OA ti H c nh.
Bi 1.33.
(a) Hy chng minh cc kt qu
AEIJ
AE | HK
(b) R =
_
b
2
+c
2
bc
3
(c) rng BHF = CKF .
p s : IH +IK = b +c.
Bi 1.34.
im c nh cn tm chnh l tip im ca ng trn ni tip tam gic ABC vi BC.
c c kt qu ny, ta cn s dng b sau :
B . Cho hai ng trn (O
1
), (O
2
) khng ct nhau, hai tip tuyn chung trong d
1
, d
2
ct
tip tuyn chung ngoi d ti A, B.Gi C, D ln lt l tip im ca (O
1
), (O
2
) vi d. Khi ,
AC = BD.
Bi 1.35.
Nu ABC cn ti A th ML AD.
Nu AB ,= AC, hy chng minh BE = CF. T suy ra ML | AD.
Bi 1.36.
p dng nh l Ptolemy cho t gic ni tip AIBK. Sau , da vo a 2, hy chng minh
rng:
S =
2 +

4 a
2
a
1
Bi 1.37.
t p =
a +b +c
2
, suy ra R
1
= p a, R
2
= p b, R
3
= p c. ng thc cn chng minh tng
ng vi :
a(p a)
2
+b(p b)
2
+c(p c)
2
+ 2(p a)(p b)(p c) = abc
chng minh ng thc ny, c th dng phng php khai trin rt gn hoc dng phng
php a thc. Phn chng minh dnh cho bn c.
Bi 1.38.
Dng v pha b AD khng cha C tam gic ADG sao cho ADG = ABM. Hy chng
minh rng N, D, G thng hng suy ra rng ABCD l hnh vung.
26
Bi 1.39.
(a) Gi L l trung im ca QN. Hy chng minh ALB vung cn suy ra L c nh.
(b) Chng minh OI, O

I vung gc v bng nhau.


Bi 1.40.
im mu cht ca bi ton l chng minh MNAD. T suy ra BIN CIM.
p s : OI =
d
2
+R
2
R
2
2d
.
Bi 1.41.
Chng minh ng thc
S
BFC
=
k
2
1 +k
2
+k
2
k
3
S
0
p s :
S = S
0

(k
1
k
2
k
3
1)
2
(k
1
k
2
+k
1
+ 1)(k
2
k
3
+k
2
+ 1)(k
3
k
1
+k
3
+ 1)
2. Cc bi ton n tp Olympiad
Bi 2.1.
Da vo nhng quan h vung gc c gi thit v quan h vung gc cn chng minh, ta c
th suy ngh theo cc hng sau :
a vo h trc ta : Tt nhin v 2 trc ta phi vung gc vi nhau, do tm
ta nn t P hoc N. Tuy nhin, do N l chn ng phn gic trong ca tam
gic ABC nn vic t tm ti N s thun tin hn.
Da vo tng trc tm : Ta c OA QN, hy tm cch dng tm K sao cho Q l
trc tm ca tam gic AOK. T cch dng im K, gii bi ton ngc chng minh
rng Q chnh l trc tm ca tam gic AOK theo cch dng .
S dng vector : S dng vector l mt phng php c s la chn phong ph. Tt
nhin ng thc cn chng minh phi l

OQ

BC = 0. Cc vector

OQ,

BC c th biu
din thnh rt nhiu tng ca cc vector khc nhau. y va l im mnh cng chnh
l im yu ca vector, ta phi tm nhng cp vector thch hp c th tnh ton. D
nhin

BC nn c gi nguyn,

OQ c th tch thnh tng ca 2 vector

OP,

PQ v 2
vector ny u c th tnh c module theo di cc cnh v cc gc ca 2 vector ny
hp vi BC cng c th xc nh theo cc gc ca tam gic ABC.
Bi 2.2.
Hy chng minh rng EF, Y Z, BC ng quy suy ra kt qu.
Bi 2.3.
Hy biu din t s
MB
MC
qua cc yu t lin quan n tam gic ABC nh tnh cht ca phng
tch. Sau s dng nh l Ceva cho tam gic ABC suy ra iu phi chng minh.
Bi 2.4.
Chiu M, N, P theo phng song song vi BC ln ng trung tuyn xut pht t A ca tam
27
gic ABC a h thc cn tnh ton ln ng trung tuyn .
Bi 2.5.
chng minh SE l tip tuyn ca (EMN) m tm ng trn ny cha xc nh, ta c 2
hng c bn sau y :
Chng minh h thc v gc : Quy v chng minh

FEM =

ENM. Hy dng cc hnh


bnh hnh AEBL, CEDK, tn dng cc tam gic ng dng rt ra ng thc v gc
trn.
Chng minh h thc v cnh : Gi s MN ct FE ti P (d thy rng P cng chnh l
trung im ca EF), ta cn chng minh PE
2
= PM PN. Gi giao im ca AB, CD
l S, hy s dng cc nh l v hng im iu ha chng minh ng thc trn. Phn
cn li xin dnh cho bn c.
Bi 2.6.
Thc cht y l bi ton o ca b quen thuc ca t gic ngoi tip ng trn : Cc
ng cho v cc ng thng ni cc tip im ca ng trn ni tip mt t gic ngoi
tip ln cc cnh i ca t gic ng quy ti mt im.
Bi 2.7.
Hy chng minh ng thc sau :
KD
KB

LD
LB
=
AD
2
AB
2
ng thc trn chng t AK, AL l hai ng ng gic trong gc BAC. Hy s dng nh
l Menelaus v ch ti cc trung im tnh ton, rt ra ng thc trn.
Bi 2.8.
Hy ch n 2 ng thc sau :
a MA = b MB +c MC
a
2
= MB
2
+MC
2
S dng 2 ng thc trn v bt ng thc Cauchy - Schwarz, ta suy ra iu cn chng minh.
Bi 2.9.
Hy ch b :
IA =
_
bc(b +c a)
a +b +c
T , ta c th a bi ton v bt ng thc i s n gin hn.
Bi 2.10.
tng chnh ca bi ton l chng minh AM, ON cng vung gc vi AD. Sau y l 2
hng cn ch tip cn kt qu ny :
Cc v i cc.
Phng tch ca mt im vi ng trn (O) v vi ng trn im tm A.
28
Bi 2.11.
(a) Gi K l giao im ca BD, CE. Hy s dng nh l Thales chng minh rng R
(D)
+
R
(E)
= BK = CK.
(b) d on trc qu tch ca I, ta chn 3 v tr M khc nhau. T cho ta gi thuyt
I di ng trn ng thng c nh song song vi BC. Cng chnh t y cho ta tng h
ng thng vung gc IH xung BC. H vung gc tng t cho D, E xung BC, bng mt
s bc tnh ton, ta s thy c di on IH khng i, t suy ra qu tch im I.
Bi 2.12.
Cu hnh ng trn vi 2 ng knh c nh vung gc vi nhau lm ta lin tng ngay
n h trc ta . Nu chn A(r, 0), B(r, 0), C(0, r), D(0, r) th qu tch ca im P s l
ng cong c phng trnh y
2
= 2xr +r
2
.
Bi 2.13.
Gi A

, B

, C

ln lt l hnh chiu vung gc ca M ln cc ng thng BC, CA, AB theo


th t. Ta chng minh cc bt ng thc, ng thc sau suy ra iu cn chng minh :
p
2
+q
2
+r
2

1
3
_
B

C
2
+C

A
2
+A

B
2
_
B

C
2
+C

A
2
+A

B
2
=
3
4
_
x
2
+y
2
+z
2
_
Bi 2.14.
p dng nh l Ptolemy cho cc t gic :
MA
1
A
2
A
3
MA
5
A
6
A
7
MA
2
A
4
A
6
A
1
A
3
A
4
A
5
Kt hp vi mt s bin i hp l, ta s c ngay bt ng thc cn chng minh.
Bi 2.15.
Trc tin, hy chng minh rng A
2
chnh l giao im ca hai tip tuyn k t B, C ca (O)
v tng t i vi B
2
, C
2
. Ta a v bi ton quen thuc v c th lm theo hai cch :
Ta c th thy ngay AA
2
, BB
2
, CC
2
chnh l cc ng i trung ca tam gic ABC nn
chng ng quy ti im Lemoine ca tam gic ABC.
p dng nh l Ceva. Tht vy, do (O) tr thnh ng trn ni tip tam gic A
2
B
2
C
2
nn A, B, C tr thnh tip im ca ng trn ni tip trn cc cnh tam gic
A
2
B
2
C
2
. T , ta c th p dng nh l Ceva cho tam gic A
2
B
2
C
2
chng minh
AA
2
, BB
2
, CC
2
ng quy.
Bi 2.16.
Ta s a AB
1
AB, AC
1
AC, AM
1
AM thnh cc biu thc cha AB, BC, CA, T
B/(O)
, T
C/(O)
,
T
M/(O)
. T bin i ng thc cn chng minh v mt ng thc ng theo cng thc trung
29
tuyn.
Bi 2.17.
Hy chng minh hai b sau y :
Tam gic XY Z ni tip ng trn bn knh R th :
XY +Y Z +ZX 3

3R
Nu I
a
, I
b
, I
c
l cc tm bng tip ca tam gic ABC th bn knh ng trn ngoi tip
tam gic I
a
I
b
I
c
bng 2 ln bn knh ng trn ngoi tip tam gic ABC.
Bi 2.18.
im mu cht ca bi ton l bt ng thc sau y :
R
2
2Rr = OI
2
DM
2
=
(b c)
2
4
Trong D l tip im ca ng trn ni tip tam gic ABC vi BC v M l trung im
ca BC.
Bi 2.19.
Bi ton da trn b sau y :
B . Gi H, I, K l hnh chiu ca im M (c nh ngha trong bi) ln BC, CA, AB
th MH = MI +MK.
Phn cn li l s dng bt ng thc tam gic khai thc b ny. Ta s thu c bt
ng thc cn chng minh.
Bi 2.20.
Ly K i xng vi H qua AB. ng thng PF ct (O), BK ti M, N, Q. Hy s dng nh
l con bm cho tam gic ABC chng minh PKQH l hnh bnh hnh.
Bi 2.21.
(a) y l mt kt qu rt quen thuc :
PA
2
+PB
2
+PC
2
+PD
2
= 4R
2
Mt cch nhanh nht l v ng knh AK ca (O) v ch BCDK l hnh thang cn suy
ra kt qu.
(b) Gi M l trung im ca OP. Trc ht hy chng minh rng IO
2
+ IP
2
khng i,
t y suy ra I di ng trn
_
M,
2R
2
OP
2
4
_
c nh.
Bi 2.22.
Hy chng minh v s dng kt qu sau : Vi im M bt k nm trong t gic ABCD, ta
lun c :
MC +MD < DA +AB +BC
Tr li bi ton, hy gi trung im cc cnh BC, CA, AB khai thc kt qu trn.
Bi 2.23.
chng minh QN MP, ta c hai hng sau :
30
Gi K l im i xng ca N qua MP. Ta s chng minh K (O). T suy ra
N, K, Q thng hng. Vi ch rng AK | MP. Ta s c iu cn chng minh.
S dng vector : Phn tch

QN thnh tng ca

QB,

QC;

MP thnh tng ca

MA,

MN
v ch cc ng vung gc vi nhau. cho tin cho vic bin i, nn t k =
NC
BC
.
Bi 2.24.
Trc ht, c nhn xt rng 2

MN =

AC +

BD. T nhn xt ny, nu gi x, y ln lt l


di hnh chiu ca HK ln AC, BD; ta ch cn chng minh x AC = y BD.
Bi 2.25.
Ta c hai hng gii quyt :
Gi K l trung im AC, hy chng minh rng KMN IQP suy ra kt qu.
S dng vector : Trc ht, c nhn xt rng 2

MN =

AC +

BD. T nhn xt ny,
nu gi x, y ln lt l di hnh chiu ca PQ ln AC, BD; ta ch cn chng minh
x AC = y BD. V ng thc ny c th chng minh da vo tnh cht phng tch ca
im I vi (O).
Bi 2.26.
Hy ch n hai b sau :
B 1 : Cho tam gic ABC v mt im M nm trong tam gic y. Khi MB+MC <
AB +AC.
B 2 : Nu tam gic ABC ngoi tip (I) th :
a

IA +b

IB +c

IC =

0
T hai b trn, hy bin i HK suy ra kt qu.
Bi 2.27.
Bt ng thc u bi tng ng vi :
S
2
ABC
S
BMK
S
CNK
8
Ta thy rng t s din tch tam gic ABC v tam gic BMK hoc tam gic CNK khng th
ngay trc tip chuyn thnh t s cc on thng v chng khng c chung nh cng khng c
chung cnh y. Do , ta s tm tam gic khc c quan h "gn gi" hn vi c 2 tam gic
ABC, CNK. Tng t, ta cng s chn tam gic c quan h "gn gi" hn vi tam gic ABC
v tam gic CNK. y chnh l mu cht ca bi ton. Tam gic cn tm l tam gic MAN.
Phn chng minh c th cn li xin dnh cho bn c.
Bi 2.28.
C hai hng gii quyt :
S dng tnh cht ca trng tm : Gi S l im i xng ca M qua trung im P ca
BC. Hy chng minh G cng l trng tm ca tam gic AMS. y chnh l cha kha
ca bi ton.
31
S dng nh l Ceva : p dng trc tip nh l Ceva dng sin cho tam gic ABC vi
ch MB
1
A
2
C
1
, MC
1
B
2
A
1
, MA
1
C
2
B
2
l cc hnh bnh hnh c cc cp cnh v gc
bng nhau.
Bi 2.29.
Ly im T trn cung BC khng cha im M ca (O) sao cho 2010 TB = 2011 TB. Sau
p dng nh l Ptolemy cho t gic TBMC.
Bi 2.30.
y l mt kt qu rt p v c rt nhiu li gii. Xin nu ra hai hng gii :
S dng tnh cht ca hng im iu ha.
Qua A k ng thng song song BC ct DE, DF ti M, N. p dng nh l Thales v
Ceva chng minh A l trung im MN.
Bi 2.31.
S dng nh l Ceva bng cch chng minh ln lt cc ng thc :

MH
BH
=
MD FA
AD FB

CK
MK
=
CE AD
EA MD

AF
BF

CE
AE

BD
CD
= 1
Bi 2.32.
Ta cn n b quan trng sau y : (vi cc k hiu nh gi thit)
AB
2
+BC
2
+CD
2
+DA
2
= AC
2
+BD
2
+ 4IJ
2
Hy dng hnh bnh hnh thch hp nhm to ra cc on thng bng nhau, kt hp vi mt
s bin i hp l thu c kt qu.
Bi 2.33.
Thng thng khi gp tng ca cc phn thc, mt cch t nhin ta s c gng a chng v
dng c chung mu. Ta c th lm c iu y trong bi ton ny vi ch :
MA AP = T
A/(O)
= T
B/(O)
= MB BQ
Bi 2.34.
ng thng ny lun i qua im O

i xng vi O qua AB c nh.


Bi 2.35.
Ta c th d dng nhn ra quan h:
GA GA
1
= GB GB
1
= GC GC
1
=
Ngoi ra, cn ch ng thc :
=
GA
2
+GB
2
+GC
2
3
32
Khi , bt ng thc hnh hc tr thnh bt ng thc i s tm thng.
Bi 2.36.
Ta cn chng minh P, Q, R, M ng vin, iu ny tng ng vi DP DQ = DR DM ()
Hy chng minh rng RB RC = RD RM (). Ch P, Q, B, C ng vin dng tnh
cht ca phng tch, t c th dng () chng minh ().
Bi 2.37.
Bin i t ng thc :
cos = cos

AOB
2
cos

COD
2
sin

AOB
2
sin

COD
2
p s :
R =

a
2
+b
2
+ 2ab cos
2 sin
Bi 2.38.
Cch quen thuc v ngn gn nht l s dng phng tch : Hy chng minh rng F, H, I u
nm trn trc ng phng ca ng trn ng knh CE v ng trn ng knh BD.
Bi 2.39.
Gi giao im ca FQ vi (I) khng trng vi F l T. Gi s TDED = M

. S dng nh
l Pascal suy ra M M

.
T N k tip tuyn tip xc vi (I) ti T

. Hy chng minh T T

suy ra FQ l ng i
cc ca N i vi (I). T y ta c kt qu cn chng minh.
Bi 2.40.
Ch n b : Trong mt t gic li, tng di hai ng cho nh hn chu vi v ln hn
tng di hai cnh i ca t gic.
Bi 2.41.
Gi K l giao im tip tuyn ti B, C ca (O). Ly Q l giao im AK, BC. Khi , c th
dng cc-i cc hoc t s kp chng t EF lun i Q c nh.
Bi 2.42.
p dng nh l Ptolemy chng minh hai kt qu sau, t suy ra ngay iu phi chng
minh :
ng trn ngoi tip ABE tip xc vi CB.
ng trn ngoi tip ADE tip xc vi CD.
Bi 2.43.
Gi M, N, P, X, Y, Z l trung im cc on BC, CA, AB, HA, HB, HC v D, E, F l chn
ng cao h t A, B, C ca tam gic ABC theo th t. Ta xt 3 trng hp sau y:
Trng hp 1 : C t nht 2 trong 3 b (M, D); (N, E); (P, F) trng nhau.
Trng hp 2 : C ng mt b trong (M, D); (N, E); (P, F) trng nhau.
Trng hp 3 : Khng b no trong cc b trn trng nhau.
33
Trng hp 1 cho ta ABC u; trng hp 2 cho ta

A = 45

,

B =

C = 67, 5

; trong khi
trng hp 3 li khng th xy ra.
Bi 2.44.
Gi N l giao im ca ID v EF, ta s chng minh AN i qua trung im BC. Hy dng
thm ng thng qua N vung gc vi ID v cc giao im ca n vi AB, AC. Ch cc
t gic ni tip v p dng nh l Thales chng minh.
Bn c c th tham kho thm cch 1 ca bi 2.73.
Bi 2.45.
Gi S l trung im AB v S

l trung im A

. Hy s dng tnh cht ca php quay


chng minh S, S

, M, N ng vin, t suy ra iu phi chng minh.


Bi 2.46.
Ch hai b sau :
B 1 : Cho im M nm trong gc

xOy. A, B theo th t l cc im khc O thuc tia
Ox, Oy; H, K theo th t l hnh chiu ca M trn Ox, Oy. Khi , ta c
P(MAB) 2HK
B 2 : Cho tam gic ABC, M l im nm trong tam gic. AM, BM, CM ct BC, CA, AB
theo th t D, E, F. Ta c
AM
AD
+
BM
BE
+
CM
CF
= 2
Bi 2.47.
Bng cc bin i gc v p dng nh l sin, hy biu din t s
BA

CA

theo cc gc B, C. T
p dng nh l Ceva suy ra iu cn chng minh.
Bi 2.48.
Nu M, O, H thng hng th PQ lun song song vi (d). Ta xt trng hp M, O, H khng
thng hng v (d) khng ct (O). Khi , ng thng PQ lun i qua giao im ca (d) v
(OHR) c nh.
Bi 2.49.
Qua A dng ng thng (d) song song vi BC v ct DF ti P. Vi ch rng AP = AF =
AE, hy p dng nh l Thales suy ra M l trung im EN.
Bi 2.50.
Bi ton ny c th gii quyt theo hai cch sau :
Hy chng minh ng thc
bAB
(a +b +c)AB

+
cAC
(a +b +c)AC

= 1
T ng thc trn v mt s nh gi, bin i thch hp, ta c iu cn chng minh.
D thy rng, bt ng thc cn chng minh tng ng vi :
4b
2
c
2
(a +b +c)
2
AB

AC

34
Ch cn ch rng :
IA
2
cos
2
A
2
AB

AC

v
IA
2
cos
2
A
2
=
4b
2
c
2
(a +b +c)
2
T suy ra iu cn chng minh.
Bi 2.51.
Hy chng minh cc t gic CDFG, CFEB, AHEB, AHGD ni tip suy ra cc gc ca t
gic EFGH u vung.
Bi 2.52.
Cho BF, AC ln lt ct DE ti T, K (KITE) = 1.
Gi giao im ca ng trn ngoi tip ABCD vi DE l N. AN ct BC ti G. Ln lt
chng minh cc kt qu sau :
(CBEG) = 1
(KINE) = 1
N T
Bi 2.53.
Hy ch rng
S
BC
S
AB
= R
[B,2(BA,BC)]
Trong S
d
l php i xng trc d, R
O,
l php quay tm O, gc quay .
Bi 2.54.
S dng b hnh thang : gi H = ADBC, khi HO i qua trung im ca AB v CD.
Bi 2.55.
Gi D, E l tip im ca vi AB, AC. Mu cht ca bi ton l s dng cc tnh cht ca
t gic iu ha, hng im iu ha chng minh M, D, P thng hng v M, E, Q thng hng.
Bi v DE | BC nn nu v tip tuyn ti M ca th cng l tip tuyn ca (MPQ).
Bi 2.56.
p s :

CHI = 20

.
Xin nu hai hng tip cn bi ton :
S dng hnh hc thun ty : K phn gic CK ca gc

HCB, gi L l hnh chiu ca


K ln BC. Ch rng tam gic KBC cn ti K v HI | CK suy ra kt qu.
S dng cng c lng gic : t =

CHI. Sau p dng nh l hm s sin cho tam


gic CHI v sau mt s php bin i hp l, ta s thu c phng trnh theo sau
y :
cos(30

+) = 2 cos 20

sin
Cng vic cn li ch l chng minh phng trnh trn c nghim duy nht = 20

.
35
Bi 2.57.
Thc cht bi ton ny l mt trng hp ring ca nh l Kariya : Cho tam gic ABC nhn
(I) l ng trn ni tip. V pha ngoi tam gic ly cc im M, N, P sao cho IM = IN = IP
v IM, IN, IP tng ng vung gc BC, CA, AB. Khi ta c AM, BN, CP ng quy. M
nh l Kariya cng l mt trng hp ring ca nh l Kiepert v nh l Jacobi.
chng minh bi ton, hy p dng nh l sin v nh l Ceva dng sin cho tam gic ABC.
Bi 2.58.
Hy biu din di cc on thng AQ, BM, MQqua a, b vi AB = AC = a, BC = b (2a > b).
T a bt ng thc cn chng minh v mt bt ng thc i s n gin.
Bi 2.59.
Gi G l giao im ca CK, AB; F l giao im ca AL, BC; M l giao im ca AL, CK.
Mt s kt qu cn ch suy ra kt lun ca bi ton :
AGC cn ti A.
M (I).
Bi 2.60.
Gi A
1
, B
1
, C
1
ln lt l trung im EF, DE, DF. Khi , hy xt php nghch o tm I
phng tch k = r
2
(vi r l bn knh ng trn ni tip tam gic ABC) v ch DA
1
, BE
1
,
CF
1
ng quy, ta s c iu phi chng minh.
Ngoi ra, ta c th s dng nh l Menelaus. Tuy nhin, ta khng th s dng nh l Meneleus
chng minh 3 tm ngoi tip y thng hng mt cch trc tip. Th nhng, ch cn
rng nu gi A
2
, B
2
, C
2
l chn ng phn gic ngoi tam gic ABC th tm ngoi tip cc
tam gic AID, BIE, CIF chnh l trung im ca IA
2
, IB
2
, IC
2
. Bng nh l Menelaus, d
thy rng A
2
, B
2
, C
2
thng hng, t suy ra iu phi chng minh.
Bi 2.61.
Gi P

l giao im ca DI vi (O) (P

thuc cung BC khng cha A). Khi , hy chng


minh rng:
PM
PN
=
AM
AN
=
P

M
P

N
ng thc ny chng t AMPN, AMP

N u l t gic iu ha. V iu ny cng chng t


P P

.
Bi 2.62.
t x =

n
_
0 x

4
_
. S dng nh l hm s sin c c phng trnh
1
sin x
=
1
sin 2x
+
1
sin 3x
Cng vic cn li ch l gii phng trnh trn
_
0,

4
_
.
p s : Bi ton c nghim duy nht n = 7.
Bi 2.63.
36
Hy tnh ton cc t s trong bi theo di cc cnh tam gic a bt ng thc cn
chng minh v mt bt ng thc i s.
Bi 2.64.
Gi M l im Miquel ca t gic ton phn BCEFAD. Hy chng minh rng d l ng
thng Steiner ca M i vi (O
1
O
2
O
3
) suy ra iu cn chng minh (ch ng trn Miquel
ca t gic ton phn v tnh cht ca ng thng Steiner)
Bi 2.65.
S dng phn chng chng minh : B qua trng hp tn ti mt cp cnh i song song,
xt trng hp c hai cp cnh i u song song. Khi , gi E l giao im ca AD, BC; F
l giao im ca AB, CD. Hy chng minh rng, nu ABCD khng l hnh bnh hnh th
FO | EO, iu ny hin nhin v l.
Bi 2.66.
ng thc cn chng minh c suy ra t 4 ng thc sau :
AD
2
= AN AC
AD
2
= AM AB
AM AD = AP AC
AN AD = AP AB
Bi 2.67.
Hy tnh tin cc ng thng cho v mt im v ch rng gc ca chng vn c bo
ton. p dng nguyn l Dirichlet ta s c iu cn chng minh.
Bi 2.68.
Ly im H trn on MN sao cho MH = BM, NH = DN. Hy chng minh H i xng vi
B qua AP, i xng vi D qua AQ. T suy ra AHPQ, QHAP c iu cn chng
minh.
Bi 2.69.
t AB = a, BC = b, CD = c, DA = d, OA = x, OB = y, OC = z, OD = t. Hy tm cch loi
b cc i lng x, y, z, t trong ng thc c gi thit. Ta cn bin i tng ng ch
cui s l a +c = b +d. Khi , p dng nh l Pithot, ta s c ABCD ngoi tip.
Bi 2.70.
C hai cch tip cn bi ton :
Cch 1 : Ch hai cp tam gic ng dng ADH CHM v AHE BMH. Sau
hy s dng cc cp t l v cnh ca hai cp ng dng chng t HE = HD.
Cch 2 : S dng tnh cht ca t s kp, hy chng minh kt qu tng qut :
MB
MC
=
HD
HE
Bi 2.71.
C hai hng tip cn bi ton :
37
Cch 1 : Chng minh NM l phn gic

ANB t suy ra MN
BC
2
(phn gic
nh hn trung tuyn).
Cch 2 : Hy chng minh:
1
MN
=
1
DM
+
1
ME
Nhn xt rng DM +ME khng i nh gi MN.
Bi 2.72.
Gi X
A
l giao im ca BC, B
0
C
0
, nh ngha tng t cho X
B
, X
C
. Hy chng minh rng
X
A
, X
B
, X
C
l cc ca A
1
A
2
, B
1
B
2
, C
1
C
2
. T suy ra rng kt lun ca bi ton tng ng
vi X
A
, X
B
, X
C
thng hng v ng thng i qua chng vung gc vi OH.
Bi 2.73.
AA
2
, BB
2
, CC
2
chnh l cc ng trung tuyn ca tam gic ABC.
Ngoi ra, ta cng c th s dng nh l Ceva dng sin chng t AA
2
, BB
2
, CC
2
ng quy.
Bi 2.74.
Bi ton c th c gii quyt theo hai cch sau :
Cch 1 : p dng nh l Menelaus cho BCE vi cc im A, F, M (sau khi tnh
cc t s mt cch thch hp).
Cch 2 : Gi H l trung im BC. Hy chng minh rng MH | CE.
Bi 2.75.
S dng b sau : Cho hai ng trn (O
1
) v (O
2
) ct nhau ti X, Z. Ly A l mt im
bt k nm trn (O
1
). Dng tia ZB i xng tia ZA qua ZX vi B thuc (O
2
). Gi O l tm
ngoi tip ABZ. Khi ta c OO
1
= OO
2
.
Bi 2.76.
Gi P l trung im EF. Ly U l im i xng ca F qua N, V l trung im EU. Hy
chng minh cc kt qu sau :
EBF EDU, PAB V CD

PM
AB
=
V N
CD
=
PF
CD

2PN
EF
=
CD
AB
Bi 2.77.
Cch nhanh nht l s dng h thc lin quan gia cc cnh, din tch v bn knh ngoi tip
tam gic.
Tuy nhin, i vi cc bn cha bit ti h thc lng trong tam gic th c th lm theo cch
k dy DE, CF song song vi AC, BD tng ng ri p dng nh l Ptolemy cho cc t gic
ni tip ABCE, ACDF suy ra kt qu.
Bi 2.78.
p dng nh l hm s sin v bt ng thc quen thuc cos A + cos B + cos C
3
2
.
38
III. Li gii chi tit
1. Cc bi ton n tp tuyn sinh lp 10
Bi 1.1 Tam gic ABC vung ti A c BC = 2AB. Ly D, E nm trn AC, AB sao cho

ABD =
1
3

ABC v

ACE =
1
3

ACB. F l giao im ca BD, CE. H, K l im i xng ca


F qua AC, BC.
(a) Chng minh H, D, K thng hng.
(b) Chng minh tam gic DEF cn.
Li gii
I
H
K
V
T
F
A
C
B
D
E
(a) Gi T = FH AC, V = FK BC. T gii thit c th suy ra tam gic ABC l na tam
gic u nn vic tnh cc gc l tm thng. Ta c,

FHD =

HFD =

ABD = 20

.
Mc khc,

FHK =

FTV (do TV | HK) =

ACE (do CTFV ni tip) = 20

FHD
Suy ra H, F, K thng hng.
(b) HK ct BC ti I. Ta ln lt tnh cc gc :

DFI = 180

DIF

IDF = 180

20

40

= 120

BEC = 90

+ 10

= 100

BIF = 80
0
nn BEFI ni tip.
Suy ra
_
_
_

EFI = 180

ABC = 120

DFI

FIE = 20

DIF
Do , DFI = EFI FD = FE. Do , tam gic DEF cn ti F.
Bi 1.2 ng trn (O) ni tip tam gic ABC(AB > AC) tip xc vi AB, AC ti P, Q.
Gi R, S ln lt l trung im BC, AC. Giao im ca PQ, RS l K. Chng minh rng
B, O, K thng hng.
Li gii
39
K
R
S
Q
P
O
A
B
C
Trc tin, ta s chng minh rng RB = RK. Gi a = BC, b = CA, c = AB, ch rng
SK = SQ do tam gic SQK c 2 gc y bng nhau. Khi :
RK = RS SK
=
c
2
SQ =
c
2
(CS CQ)
=
c
2

_
1
2
b
a +b c
2
_
=
c
2

1
2
b +
a +b c
2
=
1
2
a = BR
V vy, tam gic BRK cn ti R, suy ra

RBK =

RKB =

KBA (RK | AB).


Do K thuc ng phn gic gc

ABC hay B, O, K thng hng.


Bi 1.3 Cho tam gic ABC nhn nhn H lm trc tm. Chng minh rng, ta c bt ng
thc :
HA +HB +HC <
2
3
(AB +BC +CA)
Li gii
L
I
Y
E K
F
H
A
B
C
40
Qua H v cc ng thng song song vi BC, CA, AB ct cc cnh tam gic ABC ti
E, K, Y, I, F, L sao cho FK | AC, IE | AB, LY | BC v E, K BC; I, Y AC; F, L AB.
Khi , hin nhin cc ng thng LY, FK, IE ln lt vung gc vi HA, HB, HC.
Tam gic AHL vung ti H nn HA < AL. Tng t, ta cng c HC < CE. p dng bt
ng thc tam gic, ta thu c :
HB < HL +LB = LB +BE
Du ng thc trn do HLBE l hnh bnh hnh. T , ta thu c :
HA +HB +HC < AL +LB +BE +EC = AB +BC
Xy dng hai bt ng thc tng t ri cng theo v, ta c ngay iu cn chng minh.
Bi 1.4 Gi AB l mt dy cung c nh ca ng trn (O). P l im di ng trn dy
cung AB nhng khng trng vi hai u mt. V ng trn (C) i qua A, P tip xc trong
vi (O) v ng trn (D) i qua B, P tip xc trong vi (O). Ly N l giao im th 2 ca
(C), (D).
(a) Chng minh rng ANB CPD. T hy ch ra N di ng trn ng no.
(b) Chng minh rng NP lun i qua mt im c nh.
Li gii
K
N D
C
O
A B
P
(a) Nhn xt rng
_

PAN =
1
2

PCN =

PCD

PBN =
1
2

PDN =

PDC
.
T y suy ra ANB CPD
Do

ANB =

CPD. Mc khc, do OCPD l hnh bnh hnh nn

CPD =

AOB = nn

ANB = khng i.
Vy N di chuyn trn cung cha gc dng trn on thng AB.
41
(b) Gi K l giao im ca tip tuyn ti A, B ca (O). Khi K thuc trc ng phng
ca (C), (D) nn NP lun qua K c nh. Ta c th chng minh kt qu ny ph hp vi
kin thc lp 9 nh sau :
Gi P
1
l giao im ca KN vi (C) v P
2
l giao im ca KN vi (D). Khi :
KP
1
KN = KA
2
= KB
2
= KP
2
KN
T y suy ra P
1
P
2
P.
Bi 1.5 Cho tam gic ABC c

BAC = 120

v cc ng phn gic AA

, BB

, CC

. Tnh

.
Li gii
B'
A'
C'
A
B
C
Gi Ax l tia i ca tia AB. Khi ,

CAx = 60

nn AC l phn gic ngoi nh A ca tam


gic AA

B. Mc khc, BB

l phn gic trong ca tam gic ny nn B

chnh l tm bng tip


trong gc B ca tam gic AA

B.
Suy ra A

l phn gic

AA

C.
Chng minh tng t, ta c A

l phn gic AA

B. V vy

= 90

.
Bi 1.6 Cho hnh vung ABCD c hai ng cho ct nhau ti E. Mt ng thng i
qua A ct cnh BC M v ct ng thng CD N. Gi K l giao im ca EM v BN.
Chng minh rng CK BN.
Li gii
S
K
N
E
D
C
A B
M
42
B qua trng hp n gin EM | CD. Ko di EM ct CD ti S. p dng nh l Menelaus
cho tam gic ACN vi ct tuyn (EMS) v tam gic BCN vi ct tuyn (MKS) :
MA
MN

SN
SC

EC
EA
= 1
MC
MB

KB
KN

SN
SC
= 1
T y suy ra :
MA
MN
=
MC
MB

KB
KN
p dng nh l Thales, ta thy rng :
MA
MN
=
MB
MC
=
AB
CN
=
BC
CN
Do ,
BC
2
NC
2
=
KB
KN
Gi K

l cn ng cao k t C ca tam gic BCN th ta c kt qu quen thuc :


BC
2
NC
2
=
K

B
K

N
K v K

chia trong on BN theo cng mt t s nn trng nhau.


iu ny chng t CK BN.
Bi 1.7 Cho ABC c

BAC = 90

(AB < AC). ng trn (O; r) ng knh AB v


ng trn (P; R) ng knh AC ct nhau D v A.
(a) Gi M l im chnh gia cung nh DC, AM ct (O) ti N, ct BC ti E. Chng minh
ABE cn v cc im O, N, P thng hng.
(b) Dng ng knh NQ ca (O). Chng minh Q, D, M thng hng.
(c) Gi K l trung im MN. Chng minh PK OK.
Li gii
43
K
Q
E
N
M
D
O
P
A
B
C
(a) Vi ch rng AB l tip tuyn ti A ca (P), ta c

BAE =

BAD +

DEA =

ACD +

CAE
=

BEA
Suy ra tam gic ABE cn ti B.
Do N va l chn ng cao va l trung im AE.
T y suy ra P, N, O thng hng.
(b) T gi thit suy ra

NDQ = 90

Mt khc :

DNM +

DMN =

DBA +

DCA = 90

Suy ra

MDN = 90

. V vy Q, D, M thng hng.
(c) Ta c K l trung im MN nn KN = KD. Li c ON = OD nn KO l ng trung
trc ca ND hay KO | MD. Do

OKA =

DMA =

DCA =

OPA
V vy t gic OKPA ni tip. Suy ra

OKP = 90

hay OKPK.
Bi 1.8 Tam gic ABC nhn c 3 ng cao AA
1
, BB
1
, CC
1
ct nhau ti trc tm H. Gi
H
a
, H
b
, H
c
ln lt l trc tm ca cc tam gic AB
1
C
1
, BC
1
A
1
, CA
1
B
1
, hy chng minh
rng A
1
B
1
C
1
= H
a
H
b
H
c
.
Li gii
44
H
c
H
b
H
a
C
1
A
1
B
1
H
A
B
C
Trc ht xin pht biu v khng chng minh mt b quen thuc : Vi tam gic XY Z c
trc tm Q th QX = Y Z cot X.
p dng b trn, suy ra :
B
1
H
a
= AC
1
cot

AB
1
C
1
= AC
1
cot

ABC
(do

AB
1
C
1
=

ABC, B
1
C
1
BC ni tip)
A
1
H
b
= BC
1

BA
1
C
1
= BC
1
cot

BAC
(do

BA
1
C
1
=

BAC, ACA
1
C
1
ni tip)
M
AC
1
BC
1
=
AC
1
CC
1

CC
1
BC
1
=
cot

BAC
cot

ABC
nn B
1
H
a
= A
1
H
b
. Hn na, B
1
H
a
| A
1
H
b
(cng vung
gc vi AB). Suy ra A
1
B
1
H
a
H
b
l hnh bnh hnh.
T c c H
a
H
b
= A
1
B
1
. Lm tng t vi hai cnh cn li, ta c hai tam gic H
a
H
b
H
c
v A
1
B
1
C
1
bng nhau theo trng hp cnh-cnh-cnh.
Bi 1.9 Cho dy cung AB c nh trn (O) v

AOB = 120

. M l mt im di ng trn
cung ln AB, ng trn ni tip tam gic MAB tip xc vi MA, MB ti E, F. Chng
minh rng EF lun tip xc vi mt ng trn c nh.
Li gii
K
T
H
F
E
N
A
B
M
45
Gi N l trung im BC v H, K, T ln lt l hnh chiu ca A, B, N ln EF. Theo nh l
v ng trung bnh hnh thang th :
NT =
AH +BK
2
T gi thuyt bi suy ra

AMB = 60

nn tam gic MEF u. T y ta c AFH, BFK


u l na tam gic u. Do , AH =
1
2
AE, BK =
1
2
BF. Suy ra,
NT =
AE +BF
4
Nhng r rng AE +BF = AB nn NT =
AB
4
khng i v NT EF.
Vy EF lun tip xc vi ng trn
_
N;
AB
4
_
c nh.
Bi 1.10 Cho ng trn (O) v ng thng d nm ngoi ng trn. Gi S l hnh chiu
vung gc ca O ln d. V cc ct tuyn SAB, SEF. AF, BE ln lt ct d ti C, D. Chng
minh S l trung im ca CD.
Li gii
d
H
K
C D
E
A
O
S
B
F
T O h cc ng vung gc xung AF, BE vi H, K l chn cc ng vung gc . Khi
, H, K ln lt l trung im AF, BE. V hai tam gic SAF, SEB ng dng v SH, SK l
trung tuyn ca cc tam gic nn SAH SEK.
Suy ra

SHA =

SKE.
M OHSC, OKSD ni tip nn

SHA =

SOC,

SKE =

SOD. Do ,

SOC =

SOD. Tam gic


COD c OS va l ng cao va l phn gic nn cn ti O. V th, OS cng chnh l trung
tuyn tc S l trung im CD.
46
Bi 1.11 Cho tam gic ABC vung ti A. K ng cao AH v ng phn gic BE ca
tam gic ABC (H BC, E AC). ng thng qua A vung gc vi BE ct BC, BE ln
lt ti M, N.
(a) Chng minh t gic ANHB ni tip mt ng trn. Gi ng trn l (O).
(b) ng thng CN ct (O) ti T (T ,= N). Chng minh rng : CH BC = CN CT.
(c) Gi I l giao im ca ON v AH. Chng minh rng :
1
4HI
2
=
1
AB
2
+
1
AC
2
.
Li gii
I
T
O
M
N
H
E
B
A
C
(a) Ta c

ANB =

AHB = 90

nn t gic ANHB ni tip ng trn


_
O;
AB
2
_
.
(b) CH BC = CN CT = T
M/(O)
.
(c) Xt tam gic ABM c BN va l ng cao, va l ng phn gic trong. Do tam
gic ABM cn ti B. Suy ra N l trung im AM.
Li c AB l mt ng knh ca (O) nn O l trung im AB. V vy I l trung im AH
hay AH = 2HI. T ta c
1
4HI
2
=
1
AH
2
Vy ta cn chng minh
1
AH
2
=
1
AB
2
+
1
AC
2
M ng thc ny hin nhin ng theo h thc lng trong tam gic vung ABC. Ta c iu
cn chng minh.
Bi 1.12 Cho tam gic ABC ni tip ng trn (O; R) c ng cao AD. Gi E l hnh
chiu ca B trn AO, K l trung im ca BC, I l tm ng trn ngoi tip t gic ABDE.
Chng minh rng IK l ng trung trc ca DE.
Li gii
47
I
K
E
M
D
O
C
B
A
T gic BDEA ni tip ng trn ng knh AB nn tm I ca ng trn ngoi tip t
gic ny l trung im AB.
Ta c I v K l trung im AB, AC nn OI AB v OK BC. Suy ra ng gic BIOEK
ni tip ng trn ng knh OB.
V vy m

EIK =

EBK =

EBD =
1
2

EID hay IK l phn gic ca

DIE.
Li c ID = IE nn tam gic IDE cn ti I. Do IK l trung trc ca DE.
Bi 1.13 Cho tam gic ABC nhn ni tip ng trn (O). Cc ng cao AD, BE, CF
ct nhau ti H.
(a) K ng knh AA

ca (O), I l trung im ca BC. Chng minh rng ba im H, I, A

thng hng.
(b) Gi G l trng tm tam gic ABC. Chng minh rng S
AHG
= 2S
AOG
.
Li gii
D
F
E
G
I
A' K
H
O
C
B
A
(a) Ta c BA

| CH (cng vung gc vi AB) v CA

| BH(cng vung gc vi AC) nn t


gic BHCA

l hnh bnh hnh, do HA

v BC ct nhau ti trung im mi ng hay I


48
ng thi l trung im A

H. Vy H, I, A

thng hng.
(b) Ta c H, G, O thng hng v HG = 2GO (ng thng Euler trong tam gic ABC) nn
S
AHG
= 2S
AGO
.
Bi 1.14 Cho M l mt im nm bn trong hnh bnh hnh ABCD. Khi , hy chng
minh bt ng thc
MA MC +MB MD AC BC
Li gii.
T
C
A B
D
M
Dng hnh bnh hnh ABMT. Khi MT song song v bng AB, suy ra MT cng song song
v bng vi CD nn MCDT cng l hnh bnh hnh.
p dng bt ng thc Ptolemy cho t gic AMDT :
MT AD MA DT +MD AT
Ch cn thay MT = AB, AD = BC, DT = MC, AT = MD, ta c ngay iu cn chng minh.
Bi 1.15 Cho ng trn (O; R), ng knh BC. A l im di ng trn na ng trn
(A ,= B, C). Trn na ng trn kia ly I l im chnh gia cung BC. Dng AH
BC ti H. Gi (O
1
; R
1
); (O
2
; R
2
); (O
3
; R
3
) ln lt l cc ng trn ni tip cc tam gic
ABH, ACH, ABC.
(a) Chng minh AI O
1
O
2
.
(b) HO
1
ct AB ti E, HO
2
ct AC ti F. Chng minh O
1
O
2
H ABC.
(c) Tm v tr im A R
1
+R
2
+R
3
ln nht.
Li gii
49
S
P
F
E
O
2
O
1
O
3
H
I
O
B C
A
(a) Gi S, P ln lt l giao im ca O
1
O
3
vi AO
2
v O
2
O
3
vi AO
2
.
Ta c B, O
1
, O
3
thng hng nn

ABS +

BAS =

BAH + 2

ABS = 90

. Suy ra O
1
S AO
2
.
Tng t, ta c O
2
P AO
1
.
Do O
3
l trc tm tam gic AO
1
O
2
hay AI O
1
O
2
.
(b) Ta c BO
1
H AO
2
H nn
O
1
H
O
2
H
=
BH
AH
=
AB
AC
Suy ra O
1
HO
2
BAC.
(c) Theo mt kt qu quen thuc ta c :
_

_
R
3
=
AB +AC BC
2
R
2
=
AH +CH AC
2
R
1
=
AH +BH AB
2
V vy R
1
+R
2
+R
3
= AH R.
Do R
1
+R
2
+R
3
ln nht A l im chnh gia cung BC.
Bi 1.16 Cho na ng trn tm O ng knh AB = 2R. C l mt im trn na ng
trn (C ,= A, B). Dng CH AB ti H. E, F ln lt l hnh chiu ca H trn CA, CB.
(a) Chng minh EF song song vi tip tuyn ti C ca (O).
(b) Chng minh t gic ABFE ni tip.
(c) Tm v tr im C chu vi v din tch tam gic ABC ln nht.
(d) Chng minh khi C di ng, tm I ca ng trn ni tip OCH di chuyn trn ng
c nh.
Li gii
50
x
C'
I
F
E
H
B
O
A
C
(a) Gi tip tuyn ca (O) ti C l Cx.
Ta c

xCA =

CBA = 90

HCB =

CHF.
Mt khc t gic CEHF l hnh ch nht nn ta c

CHF =

CEF

xCA =

CEF.
Suy ra Cx | EF.
(b) Theo chng minh cu (a) ta c

CEF =

CBA nn t gic AEFB ni tip.


(c) Ta c (CA +CB)
2
2(CA
2
+CB
2
) = 2AB
2
= 8R
2
. Suy ra CA +CB 2

2R.
V vy
CA +CB +AB
_
2

2 + 2
_
R
Li c
S
ABC
=
1
2
CA CB
1
8
(CA +CB)
2

1
8
8R
2
= R
2
Trong c hai trng hp, du = xy ra C l im chnh gia cung AB.
Vy khi C nm chnh gia cung AB th chu vi v din tch tam gic ABC ln nht.
(d) Khng mt tnh tng qut, gi s CA CB.
Ta s chng minh

AIO = 135

.
Tht vy. K ng knh CC

ca (O). Ta c

ACH =

CB nn CI ng thi l phn gic

ACB.
Suy ra

ACI =

IHO = 45

nn t gic AHIC ni tip.


V vy

AIO =

AIH +

HIO
=

ACH + 90

HCI
= 90

ACI = 135

Do I lun nm trn cung cha gc 135

dng trn on OA v thuc na mt phng b


AB cha C.
Tng t vi CA CB ta c I lun thuc cung cha gc 135

dng trn on OB v nm
trn na mt phng b AB cha C.
Tm li khi C di ng trn cung AB th I lun di ng trn cung cha gc 135

dng trn
51
on OA hoc OB nm trn na mt phng b AB cha C (tr hai im A v B).
Ch . Cu (c) ca bi ton ny c mt cch gii khc c th p dng cho trng hp tam
gic ABC khng vung :
Bi 1.ton. Cho ng trn (O; R) c dy BC c nh, tm gi tr ln nht ca AB+AC vi
A l im di ng trn mt cung BC ca (O).
Li gii
Trn tia i ca tia AB ly im M sao cho AC = AM. Suy ra AMC cn ti A
Do

AMC =

ACM =

BAC
2
nn M di chuyn trn cung cha gc

BAC
2
dng trn AB v
nm trn cng mt na mt phng b BC vi A.
Suy ra AB +AC ln nht AM ln nht BCCM.
Khi A l im chnh gia cung BC ca (O).
Bi 1.17 Cho hnh vung ABCD c nh, cnh a. E l im di chuyn trn cnh CD.
ng thng AE v BC ct nhau ti F. ng thng vung gc vi AE ti A ct ng
thng CD ti K.
(a) Chng minh AF(CK CF) = BD FK.
(b) Chng minh rng trung im I ca KF di ng trn mt ng thng c nh khi E
di ng trn CD.
(c) Ch ra v tr ca E di EK ngn nht.
Li gii
T
H
I
K
F
A B
D
C
E
(a) Ta c

KAD = 90

DAF = 90

AFB =

FAB
Suy ra ABF = ADK. Do AK = AF hay FAK vung cn ti A.
Trn tia CD ly im T sao cho AT = AC th ATK = ACF.
Do KT = CF CK CF = CT.
52
V vy
AF(CK CF) = AF CT
=
1

2
KF

2AC
= BD KF
(b) Tam gic AKF vung cn ti A c I l trung im KF nn AIKF.
Suy ra t gic ADIK ni tip. Do

IAD =

IKD,

AID =

AKD.
V vy

IAD +

AID =

AKF = 45

ADB nn I lun nm trn ng thng BD.


(c) p dng h thc lng trong tam gic vung ta c
DE EK = AE
2
EK =
AE
2
DE

AC
2
CD
=
2a
2
a
= 2a
Du = xy ra khi v ch khi E trng vi C.
Bi 1.18 Cho tam gic ABC u. Gi D l im di ng trn cnh BC.
Gi (I
1
; R
1
); (I
2
; R
2
); (I
3
; R
3
) ln lt l cc ng trn ni tip ca cc tam gic
ABD, ACD, ABC v (I
3
; R) l ng trn ngoi tip tam gic ABC. Tia AD ct (I
3
; R)
ti E.
(a) Chng minh
1
ED
=
1
EB
+
1
EC
.
(b) Tm v tr ca E
1
ED
+
1
EB
+
1
EC
nh nht. Chng minh khi y S
ABEC
ln nht.
(c) Tm v tr im D R
1
+R
2
ln nht.
Li gii
F
M
E
K H
I
2
I
1
I
3
A
B
C
D
(a) Ta chng minh EA = EB +EC
Tht vy. Trn tia i ca tia EB ly im F sao cho EF = EC. Khi ECF u nn suy
53
ra BCF = ACE.
V vy m EA = FB = EB +EF = EB +EC.
T kt qu trn ta suy ra
1
EB
+
1
EC
=
EA
EB EC
Ta cn chng minh
1
ED
=
EA
EB.EC
hay ED EA = EB EC, iu ny ng do EDC
EBA.
(b) T chng minh cu (a) ta c
1
EB
+
1
EC
+
1
ED
=
2EA
EB AC

8EA
(EB +EC)
2
=
8
EA

4
R
Du = xy ra EB = EC hay E l im chnh gia cung nh BC.
Khi E l trung im cung nh BC ca (ABC) th khong cch gia E v BC ln nht, hay
tam gic BEC c din tch ln nht. Khi din tch t gic ABEC t gi tr ln nht.
(c) Gi di cnh tam gic ABC l a.
K I
1
H, I
2
K vung gc vi BC (H, K BC), gi M l trung im BC.
Ta c
HK = DH +DK =
AD +BD AB
2
+
AD +CD AC
2
=
2AD a
2
Theo nh l Thales th
R
1
R
3
=
BH
BM
=
2BH
a
v
R
2
R
3
=
CK
CM
=
2CK
a
T ta c cc ng thc sau
R
1
+R
2
R
3
=
2(a HK)
a
=
3a 2AD
a
R
1
+R
2
=
(3a 2AD)R
3
a
T ng thc cui cng suy ra R
1
+ R
2
ln nht khi AD b nht. iu xy ra khi v ch
khi D l trung im BC.
54
Bi 1.19 Cho (O; R) v mt im M nm ngoi ng trn. T M dng hai tip tuyn
MA, MB i vi (O; R). Gi E l trung im ca BM; H l giao im ca OM vi AB.
on thng AE ct (O; R) ti C.
(a) Chng minh t gic HCEB ni tip.
(b) Chng minh EMC EAM.
(c) MC ct (O) ti D. Tnh DB theo R bit OM = 3R.
(d) OB ct (O) ti T v ct AD ti S. MT giao SA ti N. Chng minh N l trung im
AS.
Li gii
I
N S
T
D
C
E
H
B
A
O
M
(a) Ta c EH | AM nn

HEA =

EAM =

CBA. Suy ra t gic HCEB ni tip.


(b) Ta c
EM
2
= EB
2
= EC EA
Suy ra
EM
EC
=
EA
EM
Mt khc, hai tam gic EMC v EAM c gc E chung. Suy ra EMC EAM.
(c) T cu (b), ta c

ADM =

MAC =

EMD
Do AD | MB. Suy ra OBAD hay BA = BD.
Ta c OB
2
= OH OM, suy ra
OH =
OB
2
OM
=
R
2
3R
=
R
3
Do
BD = AB = 2HB = 2

OB
2
OH
2
= 2
_
R
2

R
2
9
=
4

2R
3
55
(d) Gi I l giao im ca BM vi AT.
Ta c BAI vung ti A m AM = MB. Suy ra M l trung im ca IB. M MB | AD,
suy ra
SN
MB
=
TN
TM
=
AN
MI
V vy AN = NS.
Bi 1.20 Cho hnh vung ABCD cnh a. E l im di ng trn cnh AD (E ,= A). Tia
phn gic ca

EBA,

EBC ct DA, DC ti M, N.
(a) Chng minh BE MN.
(b) Tm v tr im E S
DMN
ln nht.
Li gii
I
N
M
A B
D C
E
(a) Gi I l im i xng vi A qua BM. Khi I BE v BI = a.
Tng t, nu gi I

l im i xng vi C qua BN th I

BE v BI

= a. Suy ra I I

.
V vy m I I

BIM =

BIN = 90

. Do I MN v MN vung gc vi BE ti I.
(b) T cu (a), ta suy ra AM +CN = MN. T suy ra
2

DM DN DM +DN
= 2a (AM +CN)
= 2a MN
= 2a

DM
2
+DN
2
2a

2DM DN
Do
2
_
2S
DMN
2a
_
4S
DMN
V vy
S
DMN

_
a
3 + 2

2
_
2
Du = xy ra DM = DN E D.
56
Vy din tch tam gic DMN c gi tr ln nht bng
_
a
3 + 2

2
_
2
khi E D.
Bi 1.21 Cho ABC. Mt ng trn (O) qua A v B ct AC v BC D v E. M l
giao im th hai ca cc ng trn ngoi tip cc tam gic ABC v DEC. Chng minh
rng

OMC = 90

.
Li gii
(i) Cch 1.
Q
L
P
K
I
M
E
D
O
B
C
A
Gi I, K ln lt l tm ng trn ngoi tip cc tam gic CDE v ABC.
K ng knh CP ca (I) ct AB ti L. Suy ra PM CM. (1)
Ta c

ACL +

CAB =

DEP +

DEC = 90

Do CL AB hay PC | OK.
Ta c OI DE (tnh cht ng ni tm ca 2 ng trn ct nhau) v CK DE (k ng
knh CQ ca (K), chng minh tng t CL AB)
Suy ra CIOK l hnh bnh hnh. M I l trung im CP nn PIKO cng l hnh bnh hnh.
Do PO | IK.
M IK CM (tnh cht ng ni tm ca 2 ng trn ct nhau) nn OP CM (2)
T (1) v (2) suy ra O, M, P thng hng, do

COM = 90

.
(ii) Cch 2.
57
y
x
F
K
I
M
E
D
C
B
O
A
Gi I, K l tm ng trn ngoi tip tam gic CDE, ABC. Dng tip tuyn Cx, Cy ca cc
ng trn (ABC) v (CDE).
Ta c :

xCA =

CBA =

CDE
Suy ra Cx | DE. Do DECK hay CK | OI.
Tng t, ta c CI | OK nn CIOK l hnh bnh hnh.
Gi F l trung im OC th F cng l trung im IK. M IK l ng trung trc ca CM
nn FM = FC =
OC
2
Suy ra tam gic COM vung ti M.
Bi 1.22 Cho hnh thoi ABCD c

ABC = 60

. Mt ng thng qua D khng ct hnh


thoi nhng ct cc ng thng AB, BC ln lt ti E, F. Gi M l giao im ca AF v
CE. Chng minh rng AD tip xc vi ng trn ngoi tip tam gic MDF.
Li gii
M
E
D
B
A
C
F
58
T gi thit ta c AC = AD = CD. Hai tam gic FCD v DAE ng dng, suy ra
CF
AD
=
CD
AE
Do
CF AE = AD CD = AC
2
Tng ng vi
AC
CF
=
AE
AC
Li c

ACF =

EAC, suy ra ACF EAC.


T ta c

ACM =

CFM.
V vy ACM AFC. Suy ra
AD
2
= AC
2
= AM AF
Vy AD tip xc vi ng trn ngoi tip tam gic DMF.
Bi 1.23 Cho ng trn (O) v dy AD. Gi I l im i xng vi A qua D. K tip
tuyn IB vi ng trn (O). Tip tuyn vi ng trn (O) ti A ct IB K. Gi C l giao
im th hai ca KD vi ng trn (O). Chng minh rng BC song song vi AI.
Li gii
C
K
B
I
A
D
Ta thy rng ADBC l t gic iu ha.
T , theo mt b quen thuc, ta c AD BC = AC BD.
Li c AD = DI, suy ra
DB
DI
=
CB
CA
Ch rng

BDI =

BCA, ta suy ra BDI BCA.


V vy

KBC =

BAC =

BID hay BC | AI.


59
Bi 1.24 Cho ABC ni tip ng trn tm O v ngoi tip ng trn tm I . AI, BI, CI
ct (O) ln lt ti D, E, F. DE ct CF ti M, DF ct BE ti N.
(a) Chng minh rng MN | BC.
(b) Gi Q l tm ng trn ngoi tip DMN, P l giao im ca AD v EF . Chng
minh cc im M, N, P, Q cng nm trn mt ng trn.
Li gii
P
Q
N
M
F
E
D
I
O
A
B C
(a) Ta c

NIM +

NDM = 90

BAC
2
+

ACI +

ABI
= 90

BAC +

ABC +

ACB
2
= 180

Suy ra t gic INDM ni tip.


Do

INM =

IDM =

ABE =

CBE
V vy MN | BC.
(b) Tng t cu (a), ta c PN | AB, PM | AC. Suy ra

NPM =

BAC.
Li c

NQM = 2

NDM =

ABC +

ACB
Do

NPM +

NQM =

BAC +

ABC +

ACB = 180

Vy t gic MPNQ ni tip.


Bi 1.25 Cho ABC c nh, M l im di ng trn cnh BC. Dng ng knh BE ca
ng trn ngoi tip ABM v ng knh CF ca ng trn ngoi tip ACM. Gi N
l trung im EF. Chng minh rng khi M di ng trn BC th N di ng trn mt ng
thng c nh.
60
Li gii
H
N
F
E
A
B C
M
Gi s AB < AC, gi H l trung im BC, M nm gia H v C
Ta c

AEB =

AMB =

AFC v

BAE =

CAF nn suy ra

AME =

ABE =

AMF
Do M, E, F thng hng. T ta c

ABC =

AEF,

ACB =

AFE
Suy ra ABC AEF. M H, N ln lt l trung im BC, EF nn AHC ANF.
Do

HAN =

HAC +

CAN +

NAF
=

CAF = 90

Vy N lun nm trn ng thng i qua A vung gc vi AH.


Bi 1.26 Cho tam gic ABC c

BAC = 135

, AB = a, AC = b. im M nm trn cnh
BC sao cho

BAM = 45

. Tnh di AM theo a, b.
Li gii
Ly N trn BC sao cho

BAM = 90

. p dng cng thc di ng phn gic, ta c


AN =

2 AM b
AM +b
, AM =

2 AN a
AN +a
Suy ra
AN AM +b AN =

2 AM b, AM AN +a AM =

2 AN a
Tr theo v hai ng thc trn, ta c
b AN a AM =

2 (b AM a AN)
Tng ng vi
AN =
AM
_
a +b

2
_
a

2 +b
61
Do
b

2
AM +b
=
a +b

2
a

2 +b
Vy
AM =
ab
a +b

Bi 1.27 Cho hnh vung ABCD, ly im M nm trong hnh vung sao cho

MAB =

MBA = 15

. Hi tam gic MCD l tam gic g? Ti sao?


Li gii
E
M
C D
A B
Dng tam gic u AME (E nm trong tam gic ADM). Suy ra

DAE =

MAB = 15

.
Do DEA = AMB. V vy

DEA =

AMB = 150

. Suy ra

DEM = 360

DEA

AEM = 150

T suy ra DEM = DEA hay DM = DA = DC.


Tng t ta c CM = CD.
Vy ABC l tam gic u.
Bi 1.28 Cho t gic ABCD ni tip (O; R) sao cho tia BA v tia CD ct nhau ti I, cc
tia DA v CB ct nhau K (I, K nm ngoi (O)). Phn gic ca gc

BIC ct AD, BC ln
lt ti Q, N. Phn gic ca gc

AKB ct AB, AC ln lt ti M, P.
(a) Chng minh t gic MNPQ l hnh thoi.
(b) Chng minh IK
2
= ID IC +KB KC.
(b) Gi F l trung im ca AB, J l hnh chiu ca F trn OB, L l trung im ca FJ.
Chng minh AJ OL.
Li gii
62
E
S
R
H
L
J
F
P
M
N
Q
K
I
O
A
D
B
C
(a) Gi H l giao im ca KP v IN.
Ta c

IKH +

KIH =

AKI +

AIK +
1
2

DKC +
1
2

BIC
= 180

BAD +
1
2

DKC +
1
2

BIC
Li c

DKC =

BAD

ABK =

BAD

ADC

BIC =

BAD

ADI =

BAD

ABC
Suy ra

IKH +

KIH = 180

1
2
(

ADC +

ABC) = 90

Do

KHI = 90

.
V vy cc tam gic MIP v QKN cn do c ng cao ng thi l ng phn gic.
Suy ra t gic MNPQ c hai ng cho vung gc vi nhau ti trung im H ca mi ng
cho nn t gic MNPQ l hnh thoi.
(b) Gi E l giao im ca ng trn ngoi tip tam gic ABK vi IK.
Bng mt s bin i gc n gin, ta suy ra c t gic IEAD ni tip.
Suy ra
ID IC = IA IB = IE IK
v KB KC = KA KD = KE IK
Cng theo v hai ng thc trn, ta c
ID IC +KB KC = IK(IE +KE)
= IK
2
63
(c) Gi R l giao im ca AJ v OL. K AS BO(S BO) th J l trung im BS
Ta c t gic AFSO ni tip nn

BAS =

FOJ. Do FJO BSA.


Suy ra

BAJ =

FOL hay t gic AFRO ni tip.


V vy

ARO =

AFO = 90

.
Bi 1.29 Cho t gic ABCD ni tip (O) c hai ng cho AC, BD ct nhau ti M.
ng vung gc vi OM ti M ct AB, BC, CD, DA ln lt ti M
1
, M
2
, M
3
, M
4
. Chng
minh M
1
M
4
= M
2
M
3
.
Li gii
K
H
M
3
M
1
M
2
M
4
M
O
C D
A
B
Khng mt tnh tng qut, gi s cc im c v tr tng i nh hnh v trn. Cc trng
hp khc chng minh hon ton tng t.
K OH v OK ln lt vung gc vi AD, BC.
T gic OMHM
4
ni tip nn

M
4
OM =

AHM. Tng t, ta c

M
2
OM =

M
2
KM.
Mt khc AMD BMC nn AHM BKM.
Suy ra

AHM =

M
2
KM hay

M
4
OM =

M
2
OM.
V vy M
2
OM
4
cn ti O. Do M l trung im M
2
M
4
.
Chng minh tng t, ta suy ra M l trung im M
1
M
3
.
T suy ra iu cn chng minh.
Nhn xt. Bi ton trn l mt h qu trc tip ca nh l con bm. nh l con bm tng
qut c pht biu nh sau :
T gic ABCD ni tip ng trn (O). P l giao im ca AC v BD. Mt ng thng qua
P ct (O) ti E, F; ct AB, CD theo th t ti G, H; ct BC, AD theo th t ti I, J. Khi
:
1
PE

1
PF
=
1
PG

1
PH
=
1
PI

1
PJ
Bi 1.30 Cho t gic li ABCD vi E, F l trung im ca BD v AC. Chng minh rng
AB
2
+CD
2
+BC
2
+DA
2
= 4EF
2
+AC
2
+BD
2
64
Li gii
E
F
A
D C
B
p dng cng thc ng trung tuyn, ta c :
4EF
2
= 2AE
2
+ 2CE
2
AC
2
= AB
2
+AD
2

BD
2
2
+BC
2
+CD
2

BD
2
2
AC
2
= AB
2
+AD
2
+BC
2
+CD
2
BD
2
AC
2
T suy ra
4EF
2
+BD
2
+AC
2
= AB
2
+AD
2
+BC
2
+CD
2
Ta c ng thc cn chng minh.
Bi 1.31 Trn (O; R) ly hai im B, C c nh sao cho BC =

3R. A l mt im trn
cung ln BC (A ,= B; C).
(a) Chng minh khi A di ng, phn gic

BAC lun i qua mt im c nh I.


(b) Gi E, F ln lt l hnh chiu ca I trn cc ng thng AB, AC. Chng minh
BE = CF.
(c) Chng minh khi A di ng th EF lun i qua mt im c nh.
(d) Tm v tr dim A S
AEIF
ln nht. Tnh gi tr ln nht theo R.
Li gii
65
M
F
E
I
B
O
C
A
(a) Phn gic

BAC lun i qua im I l im chnh gia cung BC nh c nh.


(b) V I l trung im cung nh BC nn IB = IC.
Li c

IEB =

IFC = 90

IBE =

ICF nn EIB = FIC.


Suy ra BE = CF.
(c) Gi M l trung im BC th IMBC. Suy ra E, F, M thng hng (ng thng Simson)
nn EF lun i qua M c nh.
(d) T EIB = FIC, ta suy ra
S
AEIF
= S
ABIC
= S
ABC
+S
BIC
V I c nh nn S
AEIF
ln nht khi v ch khi S
ABC
ln nht. iu ch xy ra khi A l
trung im cung ln BC ca (O).
Khi th
S
AEIF
=
4
3
S
ABC
=
4
3

BC
2

3
4
=
4
3

_
R

3
_
2

3
4
= R
2

3
Vy din tch t gic AEIF ln nht bng R
2

3 khi A l trung im cung ln BC ca (O).


Bi 1.32 Cho (O; R) v im A c nh vi OA > R. Dng ct tuyn AMN ca (O) khng
qua tm (AM < AN). Chng minh rng
(a) ng trn ngoi tip OMN lun i qua mt im c nh H (H khng trng O)
khi ct tuyn di ng.
(b) Tip tuyn ti M v N ca (O) ct nhau ti T. Chng minh T di ng trn mt ng
thng c nh khi ct tuyn AMN di ng.
Li gii
66
H
T
M
A
O
N
(a) Gi H l giao im ca ng trn ngoi tip tam gic OMN vi AO; AB l mt tip
tuyn ca (O) i qua A (B l tip im).
Khi ta c AH AO = AB
2
= AM AN = AO
2
R
2
khng i.
M AO v A c nh nn H c nh.
Vy (OMN) lun i qua H c nh.
(b) D thy OT l ng knh ca ng trn ngoi tip ng gic OHMTN nn

OHT = 90

,
tc l T i ng trn ng thng vung gc vi OA ti H l ng c nh.
Bi 1.33 Cho ABC c

BAC = 60

, AC = b, AB = c (b > c). ng knh EF ca ng


trn ngoi tip tam gic ABC vung gc vi BC ti M. I v J l chn ng vung gc h
t E xung AB; AC; H v K l chn ng vung gc h t F xung AB; AC.
(a) Chng minh IJ HK.
(b) Tnh bn knh ng trn ngoi tip tam gic ABC theo b v c.
(c) Tnh AH +AK theo b v c.
Li gii
I
J
K
H
M
F
E
O
B
C
A
67
(a) Ta thy HK i qua M (ng thng Simson)
Gi L l giao im ca AE v IJ, ta c

IAE =

ECB =

EBC =

JAE
Do AIE = AJE. Suy ra AEIJ.
Mt khc, ta c

EAC =

EFC =

AKH
Suy ra AE | HK. V vy IJHK.
(b) Do

BAC = 60

nn BC
2
= b
2
+c
2
bc.
Mt khc, ta c BC = R

3. T suy ra
R =
_
b
2
+c
2
bc
3
(c) Ta c BHF = CKF, suy ra BH = CK. Do
AH +AK = b +BH +c CK = b +c
Vy ta c ng thc cn chng minh.
Bi 1.34 Cho tam gic ABC. Mt im D di ng trn cnh BC. Gi P, Q tng ng l
tm ng trn ni tip ca cc tam gic ABD, ACD. Chng minh rng khi D di ng th
ng trn ng knh PQ lun i qua mt im c nh.
Li gii
E V U
Q
P
A
B C
D
Ta c b sau (phn chng minh xin dnh cho bn c) :
B : Hai ng trn (O
1
) v (O
2
) khng ct nhau, hai tip tuyn chung trong d
1
, d
2
ct
mt tip tuyn chung ngoi d ti A v B.Gi C, D ln lt l tip im ca d trn (O
1
) v (O
2
)
th AC = BD.
Tr li bi ton : Ta s chng minh im c nh l tip im F ca ng trn ni tip ABC
vi BC.
68
K tip tuyn chung trong ca (P) v (Q) khc AD ct BC ti E. Gi U, V l tip im ca
(P) v (Q) vi BC.
p dng b ta c :
BE = BU +UE = BU +DV
Li c BU =
BD +BA AC
2
v DV =
DA +DC AC
2
.
Suy ra BE =
BC +BA AC
2
hay E F.
T b ta cng c EV = UD, do FV = UD v FU = DV .
Ta c PDU DQV , suy ra
DU DV = PU QV
Tng ng vi
FU FV = PU QV
V vy PUF FV Q. T suy ra

PFQ = 90

. M

PDQ = 90

nn F thuc ng trn
ngoi tip tam gic PDQ.
Bi 1.35 Cho tam gic ABC c phn gic AD v trung tuyn AM. ng trn ngoi tip
tam gic ADM ct AB ti E v AC ti F. Gi L l trung im EF. Xc nh v tr tng
i ca hai ng thng ML v AD.
Li gii
L'
M'
F'
C'
L
F
E
M D
A
B
C
Xt trng hp tam gic ABC cn ti A th ng trn ngoi tip tam gic ADM tr thnh
ng trn ng knh AM v tip xc vi BC ti M.
Suy ra hai ng thng ML v AD trng nhau.
Xt trng hp tam gic ABC khng cn ti A
Gi giao im ca ng thng ML vi AB, AC ln lt l P, Q.
Ta c
BE
BM
=
BD
BA
=
CD
CA
=
CF
CM
Suy ra
BE = CF
69
Gi C

, F

l cc im i xng vi B, E qua AD; M

, L

l trung im BC

, EF

.
D thy rng M

, L

nm trn AD v CC

= FF

. Mt khc, MM

v LL

l cc ng trung
bnh trong cc tam gic BCC

v EFF

nn ta c MM

v LL

cng song song vi AC v c


di bng nhau. Suy ra MM

L l hnh bnh hnh.


Do ML | M

hay ML | AD.
Tm li nu tam gic ABC cn ti A th ML trng vi AD, cn nu tam gic ABC khng
cn ti A th ML song song vi AD.
Bi 1.36 Cho BC l dy cung ca (O; R). t BC = aR. im A trn cung BC ln, k
cc ng knh CI, BK. t S =
AB +AC
AI +AK
. Chng minh rng S =
2 +

4 a
2
a
. T tm
gi tr nh nht ca S.
Li gii
K
I
O
B C
A
T gi thit ta suy ra BCKI l hnh ch nht.
Suy ra IK = BC = aR v BI = CK = R

4 a
2
p dng nh l Ptolemy cho t gic AIBK ta c
AI BK +BI AK = IK AB
Tng ng vi
AI 2R +AK R

4 a
2
= AB aR
Suy ra
2AI +AK

4 a
2
= AB a
Tng t, ta c
2AK +AI

4 a
2
= AC a
Do
AB +AC
AI +AK
=
2 +

4 a
2
a
V a 2 nn S =
2 +

4 a
2
a
1.
Du = xy ra a = 2.
70
Vy S t gi tr nh nht l 1 khi v ch khi BC l ng knh ng trn (O).
Bi 1.37 Cho tam gic ABC ni tip (O, R) c

BAC 90

. Cc ng trn (A; R
1
),
(B; R
2
), (C; R
3
) i mt tip xc ngoi vi nhau.
Chng minh rng
S
ABC
=
BC R
2
1
+AC R
2
2
+AB R
2
3
+ 2R
1
R
2
R
3
4R
Li gii
t BC = a, CA = b, AB = c, p =
a +b +c
2
.
D thy rng R
1
= p a, R
2
= p b, R
3
= p c.
Ta cn chng minh
a(p a)
2
+b(p b)
2
+c(p c)
2
+ 2(p a)(p b)(p c) = abc
t E(a, b, c) = a(p a)
2
+b(p b)
2
+c(p c)
2
+ 2(p a)(p b)(p c).
Ta c
E(0, b, c) = b
_
c b
2
_
2
+c
_
b c
2
_
2
+ 2
b +c
2

c b
2

b c
2
=
_
b c
2
__
b +c 2
b +c
2
_
= 0
Tng t, ta c E(0, b, c) = E(a, 0, c) = E(a, b, 0) = 0.
Suy ra E = kabc, trong k l hng s thc.
Cho a = b = c, ta thy rng k = 1.
V vy E(a, b, c) = abc.
Bi 1.38 Cho hnh thoi ABCD c cnh l 1. Trn cnh BC ly M, CD ly N sao cho chu
vi CMN bng 2 v 2

NAM =

DAB. Tnh cc gc ca hnh thoi.


Li gii
N
M
C
D
B
A
G
71
Dng v pha na mt phng b AD khng cha C tam gic ADG sao cho ADG = ABM.
Suy ra

ADG =

ABM v BM = DG.
V MC +NC +MN = 2 nn
MN = 2 NC MC
= DN +MB
= DN +DG
Mt khc, do 2

MAN =

DAB nn AGN = AMN.


Do MN = NG hay NG = ND +DG. Suy ra N, D, G thng hng.
V vy ABCD l hnh thoi tng hai gc i din bng 180

nn ABCD l hnh vung.


Bi 1.39 V pha ngoi ca tam gic ABC dng cc hnh vung BCMN, ACPQ c tm O
v O

.
(a) Chng minh rng khi c nh hai im A, B v cho C thay i th ng thng NQ
lun i qua mt im c nh.
(b) Gi I l trung im ca AB. Chng minh IOO

l tam gic vung cn.


Li gii
F
L
I
O
O'
N
M
Q
P
C
A
B
(a) Gi L l trung im NQ, ta s chng minh L l im c nh.
Tht vy.
Ta c O

L = CO = OB, OL = CO

= O

A v

LO

A = 90

CO

L = 90

COL =

LOB
72
Suy ra LO

A = BOL. Do LA = LB v

LA =

LBO. V vy m

ALB = 360

LA

OLB

OLO

= 360

LBO

OLB

OCO

= 360

180

+
_
90

COL
_

OCO

= 90

T suy ra AIB vung cn ti L.


Mt khc, d thy rng L v C thuc cng mt na mt phng b AB. Suy ra L c nh.
(b) Ta c PCB = ACM. Suy ra PB = AM v t gic AFCP ni tip vi F l giao im
ca AM v PB
Do

PFA =

PCA = 90

.
V vy AM BP.
Li c PB = 2O

I, PB | O

I v AM = 2OI.AM | OI. Suy ra OI v O

I vung gc v bng
nhau.
Vy OIO

vung cn ti I.
Bi 1.40 Cho hai ng trn (O; R) v (O

; R

) ngoi nhau bit OO

= d > R +R

. Mt
tip tuyn chung trong ca hai ng trn tip xc vi (O) ti E v tip xc vi (O

) ti F.
ng thng OO

ct (O) ti A, B v ct (O

) ti C, D (B, C nm gia A, D). AE ct CF


ti M, BE ct DF ti N. Gi giao im ca MN vi AD l I. Tnh di OI.
Li gii
I
N
M
F
E
D A
O B O' C
T gi thit ta c BC = d R R

. Do
IB +IC = d R R

(1)
Ta thy t gic EMFN l hnh ch nht, do :

IMF =

FEN =

EAB =

IDF
73
Suy ra t gic MIFD ni tip. Do

MID =

MFD = 90

hay MN AD.
V vy BIN CIM.
p dng h thc lng trong tam gic vung ta c :
IB
2
IC
2
=
BN
2
CM
2
=
IB DB
IC AC
Suy ra
IB
IC
=
BD
AC
=
d R +R

d +R R

(2)
T (1) v (2) suy ra IB =
(d R)
2
R
2
2d
OI = OB +BI =
d
2
+R
2
R
2
2d
.
Vy OI =
d
2
+R
2
R
2
2d
.
Bi 1.41 Cho tam gic ABC c din tch S
0
. Trn cc cnh BC, CA, AB ly cc im
M, N, P sao cho
MB
MC
= k
1
,
NC
NA
= k
2
,
PA
PB
= k
3
(k
1
, k
2
, k
3
< 1).
Hy tnh din tch tam gic to bi cc on thng AM, BN, CP.
Li gii
F
E
I
A
B
C
N
M
P
Gi EIF l tam gic to bi 3 on thng AM, BN, CP. Ta c :
S
BCN
S
0
=
CN
CA
=
k
2
k
2
+ 1
v
S
BCF
S
BCN
=
BF
BN
Suy ra
S
BCF
= S
0

BF
BN

k
2
k
2
+ 1
(1)
p dng nh l Menelaus cho tam gic ABN v ct tuyn PCF, ta c
FB
FN

CN
CA

PA
PB
= 1
Suy ra
BF
FN
=
1 +k
2
k
2
k
3
74
Tng ng vi
BF
BN
=
1 +k
2
1 +k
2
+k
2
k
3
(2)
T (1) v (2), ta suy ra
S
BFC
=
k
2
1 +k
2
+k
2
k
3
S
0
Chng minh tng t :
S
ACI
=
k
3
1 +k
3
+k
1
k
3
S
0
, S
AEB
=
k
1
1 +k
1
+k
1
k
2
S
0
T ta c din tch tam gic to bi cc on thng AM, BN, CP l :
S = S
0
_
1
_
k
1
1 +k
1
+k
1
k
2
+
k
2
1 +k
2
+k
2
k
3
+
k
3
1 +k
3
+k
1
k
3
__
= S
0

(k
1
k
2
k
3
1)
2
(k
1
k
2
+k
1
+ 1)(k
2
k
3
+k
2
+ 1)(k
3
k
1
+k
3
+ 1)

2. Cc bi ton n tp Olympiad
Bi 2.1 (APMO 2000) Cho tam gic ABC vi trung tuyn AM v phn gic AN. ng
thng vung gc vi AN ti N ct AB, AM ln lt ti P, Q. ng thng vung gc vi
AB ti P ct ng thng AN ti O. Chng minh rng OQ vung gc vi BC.
Li gii
(i) Cch 1. S dng phng php ta .
A
O
P
Q
N
M
B
C
Ta chn N l gc ta v trc honh, trc tung nm trn NA, NP tng ng.
Gi y = ax + b vi a, b R

l phng trnh ng thng AB. Khi , phng trnh ng


thng AC c dng y = ax b.
Gi s phng trnh ng thng BC l y = cx vi c R

.
T c th d dng suy ra ta ca im B, C l :
B
_
b
c a
,
bc
c a
_
, C
_

b
c +a
,
bc
c +a
_
75
Khi , trung im M ca BC c ta : M
_
ab
c
2
a
2
,
abc
c
2
a
2
_
.
T phng trnh ng thng AB, PO, ta tnh c :
A
_

b
a
, 0
_
, O(ab, 0)
Do , ta c th vit phng trnh ng thng AM ri suy ra ta im Q l
_
0,
ab
c
_
.
T y, ta thy rng h s gc ca ng thng OQ l
1
c
, trong khi h s gc ca BC l c.
Suy ra OQ BC.
(ii) Cch 2. S dng hnh hc x nh.
A
H
K
O
P
Q
N
M
B
C
ng thng PQ ct AC ti H v ng thng qua A song song vi BC ti K.
ng thng BC | AK v ct cc ng thng AP, AM, AQ ti B, M, C tha mn M l trung
im BC. Do , (AP, AH, AQ, AK) = 1 hay (PHQK) = 1. NA va l phn gic va l
ng cao tam gic APH nn N l trung im PH. Theo h thc Newton :
NQ NK = NP
2
= AN NO
T y d thy rng Q l trc tm ca tam gic AOK.
OQ AK hay OQ BC (do BC | QK)
(iii) Cch 3. Ta s s dng vector chng t rng :

OQ

BC = 0
Trc tin, ta s tnh on PQ. t

BAM = ,

CAM = v gi H l giao im ca PQ v
AC. Khng mt tnh tng qut, gi s rng .
Tam gic APH c AN va l ng cao va l phn gic nn cn ti A. Do :
QP
sin
=
QH
sin
=
2PN
sin + sin
Suy ra
PQ =
2PN sin
sin + sin
=
2PN
1 +
sin
sin
=
2PN AC
AB +AC
76
Ta bin i :

OQ

BC =

BC

OP +

PQ

BC
=

AC

OP +

PQ

BC
= AC OP sin A +
2PN AC BC
AB +AC
cos
B C
2
Do , ta cn chng minh
2PN AC BC
AB +AC
cos
B C
2
= AC OP sin A
Tng ng vi
2BC
AB +AC
cos
A
2
cos
B C
2
= sin A
Hay
2 cos
A
2
cos
B C
2
= sin B + sin C
ng thc ny l hin nhin v ta c
2 cos
A
2
cos
B C
2
= 2 sin
B +C
2
cos
B C
2
= sin B + sin C
Bi ton c chng minh.
Bi 2.2 (D tuyn IMO 1994) Tam gic ABC khng cn ti A c D, E, F l cc tip im
ca ng trn ni tip ln BC, CA, AB. X l im bn trong tam gic ABC sao cho ng
trn ni tip tam gic XBC tip xc vi BC ti D, v tip xc vi XB, XC ti Y, Z. Chng
minh rng E, F, Y, Z ng vin.
Li gii
S
E
Z
Y
X
D
F
A
B
C
Trc tin, ta s chng minh EF, Y Z, BC ng quy ti 1 im.
Tht vy, gi S = EF BC, S

= Y Z BC.
Do AD, BE, CF ng quy nn (SDBC) = 1.
Tng t, XD, BZ, CY ng quy, suy ra (S

DBC) = 1. T S S

hay EF, Y Z, BC ng
77
quy ti S.
Do SD l tip tuyn ca (DEF) nn SD
2
= SE SF. Mc khc, SD cng l tip tuyn ca
(DY Z), suy ra SY SZ = SD
2
= SE SF.
ng thc ny chng t E, F, Y, Z ng vin.
Bi 2.3 Dng hnh vung DEFG ni tip tam gic ABC sao cho D, E BC; F AC; G
AB. Gi d
A
l trc ng phng ca hai ng trn (ABD), (ACE). Ta nh ngha cc ng
thng d
B
, d
C
tng t. Chng minh rng cc ng thng d
A
, d
B
, d
C
ng quy.
Li gii
M
A
E
G
D
F
B
C
Gi M l giao im ca d
A
vi ng thng BC. R rng M thuc on thng DE, bn
c t kim tra iu ny. Hn na, do M thuc trc ng phng ca (ABD), (ACE) nn
MD MB = ME MC. T suy ra :
MB
MC
=
ME
MD
=
BE
CD
=
BD +GD
CE +EF
=
cot B + 1
cot C + 1
Suy ra

MB
MC
=

cot B + 1
cot C + 1
= 1
Theo nh l Ceva cho tam gic ABC, ta c ngay iu cn chng minh.
Bi 2.4 Cho tam gic ABC vi trng tm G. Mt ng thng d i qua G ct BC, CA, AB
ln lt ti M, N, P. Chng minh rng, ta c ng thc :
1
GM
+
1
GN
+
1
GP
= 0
Li gii
78
D
P'
N'
N
P
G
M'
A
B
C
M
Gi M

, N

, P

ln lt l hnh chiu ca M, N, P theo phng song song vi BC ln AG. Khi


M

l trung im BC v ng thc cn chng minh tng ng vi :


1
GM

+
1
GN

+
1
GP

= 0
Gi D l giao im ca MN vi ng thng qua A song song vi BC. Do M

l trung im
BC nn A(GDNP) = 1. T y suy ra (GAN

) = 1. S dng h thc Descartes cho


hng im ny, ta c
1
GN

+
1
GP

=
2
GA
=
1
GM

V vy
1
GM

+
1
GN

+
1
GP

= 0
ng thc c chng minh.
Bi 2.5 Cho t gic ABCD ni tip ng trn (O) c cc cnh i khng song song v
cc ng cho ct nhau ti E. F l giao im ca AD vi BC. M, N ln lt l trung im
ca AB, CD. Chng minh rng EF l tip tuyn ca ng trn ngoi tip tam gic EMN.
Li gii
(i) Cch 1.
79
Q
K
S
M
L
I
E
K
F
N
A
C
D
B
Dng cc hnh bnh hnh AEBL, CEDK. Gi I l trung im ca EF. Khi , I, M, N thng
hng v chng nm trn ng thng Gauss ca t gic ton phn AEBF.
Php v t tm E t s 2 bin I F, M L, N K. Do , F, L, K thng hng.
Do FAB FCD suy ra :
FD
FB
=
CD
AB
=
EC
EB
=
DK
EB
Hn na, ta c

FBE =

FDK.
T hai ng thc trn suy ra FDK FBE. Do

FEB =

FKD.
Mc khc, EAB EDC m M, N l trung im AB, CD nn EBM ECN.
Suy ra

MEB =

NEC =

DKE. Kt hp vi

FEB =

FKD, ta c

FEM =

FKE.
Ch rng MN | LK nn

FEM =

ENM.
ng thc ny chng t FE l tip tuyn ca ng trn ngoi tip tam gic EMN.
(ii) Cch 2.
Gi K, Q l giao im ca AB, CD vi EF; S l giao im ca AB, CD.
Ta cn chng minh rng IE
2
= IM IN.
p dng h thc Maclaurin cho :
(DCQS) = 1 vi N l trung im CD : SQ SN = SC SD
(ABKS) = 1 vi M l trung im AB : SM SK = SA SB.
Do ABCD ni tip nn SA SB = SC SD, suy ra SQ SN = SM SK.
Suy ra MNQK ni tip hay IM IN = IQ IK.
Do , bi ton quy v chng minh IQ IK = IE
2
.
Php chiu xuyn tm B bin hng im iu ha (DCQS) thnh hng iu ha (EFQK). p
dng h thc Newton cho hng im ny vi I l trung im EF, ta c ngay IQ IK = IE
2
,
y l iu cn chng minh.
80
Bi 2.6 Cho tam gic ABC vi ng trn ni tip (I) v E, F l cc tip im ca (I) vi
CA, AB. Ly K bt k thuc on EF, gi H, L l giao im ca BK, CK vi AC, AB tng
ng. Chng minh rng HL tip xc vi (I).
Li gii
T
Q
H
V
F
E
P
I
A
B
C
K
Tip tuyn ca ng trn (I) i qua H v ct AB ti V . Bi ton quy v chng minh V L
hay tng ng vi BH, CV, EF ng quy. y l mt tnh cht quen thuc ca t gic ngoi
tip, xin c php chng minh li tnh cht ny.
Gi Q, P l tip im ca (I) ln V H, BC. V ng thng song song vi BV qua H v ct
EF ti T. EF ct BH, PQ ti K
1
, K
2
. Ta c

V FK =

HTE v

V FK =

HET. Do tam gic


HET cn ti H. Suy ra,
K
1
B
K
1
H
=
BF
HT
=
BF
HE
=
BP
QH
=
K
2
B
K
2
H
K
1
, K
2
cng chia trong on thng BH vi cng mt t s nn chng trng nhau. iu ny
chng t BH i qua giao im ca PQ, EF. Chng minh tng t, CV cng i qua giao im
ny. Do , BH, CV, EF, PQ ng quy ti K, y l iu cn chng minh. Bi ton c gii
quyt.
Bi 2.7 Gi BH, BD ln lt l ng cao v phn gic ca tam gic ABC. N, L, M ln
lt l trung im ca BH, BD, AC. Ly K l giao im ca MN v BD. Chng minh rng,
AL, AK l hai ng ng gic trong gc

BAC.
Li gii
81
K
M
L
N
D
H
A
B
C
chng minh AK, AL l hai ng ng gic trong gc

BAC, ta ch cn ch ra rng :
KD
KB

LD
LB
=
AD
2
AB
2
Nhng do L l trung im ca BD nn ng thc cn chng minh tng ng vi :
KD
KB
=
AD
2
AB
2
=
b
2
(a +c)
2
p dng nh l Menelaus cho tam gic BDH vi ct tuyt MNK, ta c
KD
KB

NB
NH

MH
MD
= 1
Suy ra
KD
KB
=
MD
MH
Chn hng dng trn ng thng AB theo chiu

AB, khi :
AM =
b
2
, AD =
bc
a +c
, AH = c cos A =
b
2
+c
2
a
2
2b
Suy ra
MD =
b(c a)
2(c +a)
, MH =
c
2
a
2
2b
V vy
MD
MH
=
b
2
(a +c)
2
hay
KD
KB
=
b
2
(a +c)
2
, y l iu cn chng minh.
Bi 2.8 Cho tam gic ABC vung ti A. Trn cc tia AB, AC ly E, F tng ng sao cho
BE = BC = CF. Chng minh rng vi mi im M nm trn ng trn ng knh BC,
ta u c
MA +MB +MC EF
Li gii
82
E
F C
O
B
A
M
t BC = a, CA = b v AB = c.
p dng nh l Ptolemy cho t gic ni tip MBAC :
aMA = bMB +cMC
Theo bt ng thc Cauchy-Schwarz :
(MA +MB +MC)
2
=
1
a
2
[MB(a +b) +MC(a +c)]
2

MB
2
+MC
2
a
2
_
(a +b)
2
+ (a +c)
2

= EF
2
Vi ch rng MA +MB +MC > 0 v EF > 0, khai cn hai v, ta c
MA +MB +MC EF
y l iu cn chng minh.
Bi 2.9 Cho tam gic ABC c BC = a, CA = b, AB = c v I l tm ng trn ni tip
tam gic ABC. Chng minh rng
IA +IB +IC

ab +bc +ca
Li gii
c
b
a
D
I
A
B C
83
Ta s s dng b :
IA =
_
bc(b +c a)
a +b +c
Chng minh b .
Gi D l chn ng phn gic t nh A. Theo cng thc ng phn gic :
IA
c
=
ID
BD
=
AD
c +BD
T cc ng thc
BD =
ac
b +c
AD
2
=
4bc
(b +c)
2
p(p a)
Ta suy ra :
IA =
_
bc(b +c a)
a +b +c
B c chng minh.
Theo b , ta cn chng minh rng :
_
bc(b +c a) +
_
ca(c +a b) +
_
ab(a +b c)
_
(a +b +c)(ab +bc +ca)
Bnh phng hai v, ta c

(b
2
c +bc
2
) 3abc + 2
_
abc
2
[c
2
(a b)
2
]

(b
2
c +bc
2
) + 3abc
Bt ng thc trn tng ng vi

_
(b +c a)(c +a b)
ab
3
n y ta c th s dng AM-GM nh sau :

_
(b +c a)(c +a b)
ab
=

_
b +c a
b

_
c +a b
a

1
2

_
b +c a
b
+
c +a b
a
_
= 3
Bt ng thc cui c chng minh nn suy ra
IA +IB +IC

ab +bc +ca
Chng minh hon tt ti y.
Bi 2.10 T im A nm ngoi ng trn (O), k hai tip tuyn AB, AC n (O). Gi
E, F l trung im ca AB, AC. Ly D l mt im bt k trn EF, v cc tip DP, DQ ti
ng trn. PQ ct BC, EF ln lt ti N, M. Chng minh rng, ON | AM.
Li gii
(i) Cch 1.
84
K
S
L
T
N
M
P
Q
F
E
A
O
B
C
D
Xt cc - i cc i vi ng trn (O, R) : Al cc ca BC, D l cc ca PQm BCPQ = N
nn N chnh l cc ca AD i vi O.
AD ON.
Mc khc, t ED OA suy ra,
DO
2
DA
2
= EO
2
EA
2
=
1
2
_
OA
2
+OB
2
_

1
4
AB
2

1
4
AB
2
= R
2
V vy
DA
2
= DO
2
R
2
= T
D/(O)
ng thc ny chng t D l tm ca ng trn ngoi tip tam gic APQ. Xt cc - i cc
i vi ng trn ny : O l cc ca PQ nn M, O lin hp. Hn na, DM OA nn M l
cc ca OA.
Do AM AD. T y suy ra ON | AM (iu cn chng minh).
(ii) Cch 2.
Theo chng minh cch 1, ta c c DA = DP = DQ = r. Hn na, do E, F l trung im
AB, AC nn EF chnh l trc ng phng ca (O; R) v (A; 0). T suy ra MA l tip
tuyn (APQ) hay AM DA.
OD ct BC, PQ T, L v OA ct BC, PQ K, S. Ta c OL OD = R
2
= OK OA nn
AKLD ni tip. D thy rng SKTL cng ni tip nn AD | ST.
Tam gic STO nhn N lm trc tm nn ST ON.
Do , AD ON. T y suy ra ON | AM.
85
Bi 2.11 Cho tam gic ABC cn ti A ni tip ng trn (O). Trn cnh y BC, ly
im M (M khc B, C). V ng trn tm D qua M tip xc vi AB ti B v ng trn
tm E qua M tip xc vi AC ti C. Gi N l giao im th hai ca hai ng trn ny.
(a) Chng minh rng tng bn knh ca hai ng trn (D), (E) l khng i khi M di
ng trn BC.
(b) Tm tp hp trung im I ca DE.
Li gii
F
K
H
I
N
E
D
P
Q
A
O
B C
M
(a) Gi K l giao im ca BD, CE. Ch rng cc tam gic DBM, EMC, BKC cn nn
DM | CK, EM | BK v BK = CK = k khng i. p dng nh l Thales, ta c
DM
CK
=
BM
BC
,
EM
BK
=
CM
BC
Suy ra
R
(D)
+R
(E)
k
=
BM +CM
BC
= 1
V vy R
(D)
+R
(E)
= k khng i.
(b) Gi P, Q, H, F ln lt l hnh chiu ca D, E, I, K ln BC.
DP
KF
+
EQ
KF
=
BD
BK
+
CE
CK
= 1
Suy ra DP + EQ = KF = khng i. T y IH =

2
cng khng i. Do , I di chuyn
trn ng thng song song v cch BC mt khong

2
khng i.
Bi 2.12 Cho M l im di ng trn ng trn (O, r) c hai ng knh c nh AB, CD
vung gc vi nhau. Gi I l hnh chiu ca M ln CD v P l giao im ca OM, AI. Tm
tp hp cc im P.
Li gii
86
y
x
P
I
B
C
A O
D
M
Chn h trc ta nhn O lm gc v A(r, 0), B(r, 0), C(0, r), D(0, r) v M(r cos , r sin ).
Khi ta c :
Phng trnh ng thng CD : x = 0.
Phng trnh ng thng IM : y = r sin .
T suy ra ta im I l I(0, r sin ).
Phng trnh ng thng OM :
x
r cos
=
y
r sin
Phng trnh ng thng AI :
x +r
r
=
y
r sin
Suy ra P c ta tha mn h phng trnh :
_

_
x
r cos
=
y
r sin
x +r
r
=
y
r sin

_
_
_
tan =
y
x
sin =
y
x +r
Ta li c tan
2
=
sin
2

1 sin
2

, suy ra :
y
2
x
2
=
y
2
(x +r)
2
1
y
2
(x +r)
2
ng thc ny tng ng vi y
2
= 2xr +r
2
.
Vy tp hp cc im P l parabol c phng trnh y
2
= 2xr +r
2
.
87
Bi 2.13 Cho tam gic u ABCv mt im M bt k trong mt phng tam gic. Gi
x, y, z l khong cch t M n cc nh A, B, C v p, q, r l khong cch t M n cc cnh
AB, BC, CA. Chng minh rng :
p
2
+q
2
+r
2

1
4
(x
2
+y
2
+z
2
)
Li gii
C'
B'
A'
B
A
C
M
Nu M trng vi mt trong cc nh A, B, C th d thy bt ng thc cn chng minh l
ng.
Xt trng hp M khng trng vi nh no ca tam gic ABC. Gi A

, B

, C

ln lt l hnh
chiu vung gc ca M ln cc ng thng BC, CA, AB theo th t v G l trng tm tam
gic A

.
Theo nh l Leibniz, ta c
MA
2
+MB
2
+MC
2
= 3MG
2
+
1
3
_
B

C
2
+C

A
2
+A

B
2
_

1
3
_
B

C
2
+C

A
2
+A

B
2
_
Mt khc, tam gic AB

ni tip ng trn ng knh AM, do

AC

= 60

hoc

AC

= 120

. V vy (theo nh l sin)
B

= MAsin 60

(= MAsin 120

) =
x

3
2
Suy ra B

C
2
=
3x
2
4
. Tng t, ta c C

A
2
=
3y
2
4
, A

B
2
=
3z
2
4
.
Do
B

C
2
+C

A
2
+A

B
2
=
3
4
_
x
2
+y
2
+z
2
_
V vy
p
2
+q
2
+r
2

1
4
_
x
2
+y
2
+z
2
_
y chnh l bt ng thc cn chng minh.
88
Bi 2.14 Cho a gic u A
1
A
2
A
3
A
4
A
5
A
6
A
7
v im M bt k trong mt phng. Chng
minh rng
MA
1
+MA
3
+MA
5
+M
7
MA
2
+MA
4
+MA
6
Li gii
A
1
O
M
A
2
A
3
A
4
A
5
A
6
A
7
t A
1
A
2
= a, A
1
A
3
= b, A
1
A
4
= c.
p dng nh l Ptolemy :
i vi t gic A
1
A
2
A
3
M :
a(MA
1
+MA
3
) bMA
2
(1)
i vi t gic A
5
A
6
A
7
M :
a(MA
5
+MA
7
) bMA
6
(2)
i vi t gic A
2
A
4
A
6
M :
b(MA
2
+MA
6
) cMA
4
(3)
T (1) v (2) suy ra :
a(MA
1
+MA
3
+MA
5
+MA
7
) b(MA
2
+MA
6
) (4)
T (3) v (4) suy ra :
a(MA
1
+MA
3
+MA
5
+MA
7
) cMA
4
(5)
T (4) v (5) suy ra :
a(MA
1
+MA
3
+MA
5
+MA
7
)
_
1
b
+
1
c
_
MA
2
+MA
4
+MA
6
(6)
p dng nh l Ptolemy cho t gic ni tip A
1
A
3
A
4
A
5
, ta c :
ab +ac = bc a
_
1
b
+
1
c
_
= 1
89
Thay vo (6) ta c :
MA
1
+MA
3
+MA
5
+MA
7
MA
2
+MA
4
+MA
6
Ta c iu cn chng minh.
Bi 2.15 Tam gic ABC khng cn ni tip (O) c A
1
, B
1
, C
1
l trung im ca
BC, CA, AB. Gi A
2
l mt im trn tia OA
1
sao cho 2 tam gic OAA
1
v OA
2
A ng
dng. Cc im B
2
, C
2
nh ngha tng t. Chng minh rng AA
2
, BB
2
, CC
2
ng quy.
Li gii
V
T
A
2
B
2
C
2
A
1
B
1
C
1
O
A
B
C
(i) Cch 1.
T hai tam gic OAA
1
v OA
2
A ng dng suy ra OA
1
OA
2
= OA
2
= R
2
. Do , A
2
chnh
l giao im cc tip tuyn ti B, C ca (O).
ng thng qua A
2
song song vi tip tuyn ca (O) ti A ct AB, AC ti T, V . Do

A
2
BT =

A
2
TB nn A
2
B = A
2
T. Mt cch tng t, A
2
T = A
2
B = A
2
C = A
2
V . V th, BCV T ni
tip (A
2
) hay ABC AV T.
Li c A
1
, A
2
ln lt l trung im BC, TV nn AA
1
C AA
2
T. Suy ra

CAA
1
=

TAA
2
.
ng thc ny chng t AA
2
l ng i trung ca tam gic ABC. Do , cc ng thng
AA
2
, BB
2
, CC
2
s ng quy ti im Lemoine ca tam gic ABC.
(ii) Cch 2.
Theo chng minh cch 1 th (O) chnh l ng trn ni tip ca tam gic A
2
B
2
C
2
. Do
BA
2
= CA
2
, CB
2
= AB
2
, BC
2
= AC
2
nn :
CA
2
CB
2

AB
2
AC
2

BC
2
BA
2
= 1
90
Theo nh l Ceva, ta c ngay AA
2
, BB
2
, CC
2
ng quy.
Bi 2.16 Cho tam gic ABC vi M l trung im BC. V ng trn (O) ty qua A v
ct cc on AB, AC, AM ln lt ti B
1
, C
1
, M
1
. Chng minh rng
AB
1
AB +AC
1
AC = 2AM
1
AM
Li gii
C
1
M
1
B
1
M
A
B C
O
Ta c
_

_
AB
1
AB = AB
2
BB
1
AB = AB
2
T
B/(O)
AC
1
AC = AC
2
CC
1
AC = AC
2
T
C/(O)
2AM
1
AM = 2AM
2
2T
M/(O)
= AB
2
+AC
2

BC
2
2
2T
M/(O)
Do , ch cn kim tra ng thc sau l :
T
B/(O)
+T
C/(O)
2T
M/(O)
=
BC
2
2
ng thc ny tng ng vi :
OB
2
+OC
2
2OM
2
=
BC
2
2
(ng theo cng thc trung tuyn cho tam gic OBC).
V vy, bi ton c chng minh hon tt.
Bi 2.17 Cho tam gic ABC ni tip ng trn bn knh R.Gi q l chu vi tam gic c
cc nh l tm cc ng trn bng tip tam gic ABC. Chng minh rng :
q 6

3R
Li gii
91
O
I
b
I
I
a
I
c
A
B
C
Ni dung ca bi ton thc cht l s kt hp trc tip ca hai b sau :
B 1 : Cho tam gic XY Z ni tip ng trn (O, R).
Khi XY +Y Z +ZX 3

3R.
Chng minh.
Gi G l trng tm tam gic XY Z, khi theo nh l Leibniz, ta c
9R
2
(XY
2
+Y Z
2
+ZX
2
) = 9OG
2
0
Kt hp vi bt ng thc Cauchy-Schwarz, ta c
27R
2
3(XY
2
+Y Z
2
+ZX
2
)
(XY +Y Z +ZX)
2
Tng ng vi
3

3R XY +Y Z +ZX
B 1 c chng minh.
B 2 : Cho tam gic ABC ni tip (O, R). I
a
, I
b
, I
c
theo th t l tm ng trn bng
tip cc gc A, B, C. Khi ng trn ngoi tip tam gic I
a
I
b
I
c
c bn knh bng 2R.
Chng minh.
V AI
a
v I
b
I
c
l cc ng phn gic trong v ngoi ca gc

BAC nn I
a
AI
b
I
c
.
Do A, B, C l chn cc ng cao trong tam gic ABC nn (ABC) l ng trn Euler ca
tam gic I
a
I
b
I
c
. V vy bn knh ng trn (I
a
I
b
I
c
) bng 2R. B 2 c chng minh.
Bi 2.18 Cho tam gic ABC c : BC = a; CA = b; AB = c; v r v R theo th t l bn
knh ng trn ni tip v ngoi tip tam gic ABC. Chng minh rng
r
R
+
(a b)
2
+ (b c)
2
+ (c a)
2
16R
2

1
2
Li gii
92
M D
I
O
A
B C
Gi I l tm ng trn ni tip tam gic ABC v D l tip im ca ng trn ni tip (I)
trn cnh BC, M l trung im ca BC.
Khng mt tnh tng qut, gi s b a c. Khi :
R
2
2Rr = OI
2
DM
2
=
(b c)
2
4
Tng ng vi
r
R
+
(b c)
2
8R
2

1
2
Mt khc :
(b c)
2
= (a b)
2
+ (c a)
2
+ 2(a b)(c a)
(a b)
2
+ (c a)
2
Suy ra :
r
R
+
(a b)
2
+ (b c)
2
+ (c a)
2
16R
2

1
2
Chng minh hon tt.
Bi 2.19 Cho tam gic ABC. Cc ng phn gic BE, CF ct nhau ti I. AI ct EF ti
M. ng thng qua M song song vi BC theo th t ct AB, AC ti N, P. Chng minh
rng
MB +MC < 3NP
Li gii
93
S R H
K
L
P
F
M
E
N
I
O
A
B C
u tin, ta chng minh b sau y :
B : Cho tam gic ABC, c phn gic BD, CE. Ly im M bt k thuc DE. K
MH BC, MK AC, ML AB. Khi ta c MH = ML +MK.
Chng minh b .
Gi T l giao im DF v MH.
T E, D v EF, DO BC; DN AB; EP AC. Suy ra : EF = EP; DN = DO.
Theo nh l Thales, ta c
MK
EP
=
MD
DE
=
MT
EF
Do EF = EP nn MT = MK (1)
Cng theo nh l Thales, ta c
ML
DN
=
EM
ED
=
FH
FO
=
HT
DO
M DO = DN nn TH = ML (2)
T (1), (2) suy ra
MT +TH = MH = ML +MK
B c chng minh.
Tr li vi bi ton. Gi H, K, L theo th t l hnh chiu ca M ln BC, CA, AB
Qua M k MR | AB v MS | AC. p dng b ta c
MH = ML +MK = 2ML = 2MK
Ch rng : MRH MNL v MSH MPK.
Suy ra MR = 2MN v MS = 2MP.
p dng bt ng thc tam gic, ta c
MB +MC < (MR +BR) + (MS +SC)
= 3(MN +MP)
= 3NP
94
Ta c iu cn chng minh.
Bi 2.20 Cho tam gic ABC nhn vi ng cao CF v CB > CA. Gi O, H ln lt l
tm ngoi tip v trc tm ca tam gic ABC. ng thng qua F vung gc vi OF ct
AC ti P. Chng minh rng

FHP =

BAC.
Li gii
Q
M
N
P
H
K
F
O
A
B
C
Gi K l im i xng ca H qua AB, khi K (O). ng thng PF ct (O) v BK, AC
ln lt ti M, M, Q, P, trong P, N thuc cng mt na mt phng b CK khng cha B.
Xt dy cung MN c OF MN nn F l trung im ca MN. Do , p dng nh l con
bm cho dy cung MN, ta thy rng F cng l trung im ca PQ. Mc khc, F l trung
im HK nn PHQK l hnh bnh hnh.
Vy

PHF =

BKC =

BAC, ta c iu cn chng minh.


Bi 2.21 Cho ng trn (O; R) v mt im P c nh bn trong ng trn. AB, CD l
2 dy cung di ng ca (O) nhng lun i qua P v lun vung gc vi nhau.
(a) Chng minh rng PA
2
+PB
2
+PC
2
+PD
2
khng i.
(b) Gi I l trung im BC. Hi I di ng trn ng no?
Li gii
M
I
K
A
B
O
C
D
P
95
(a) p dng nh l Pythagore, ta thy rng :
PA
2
+PB
2
+PC
2
+PD
2
= AC
2
+BD
2
= BC
2
+AD
2
V ng knh AK ca ng trn (O). Khi , BK AB m AB CD nn BK | CD.
Hnh thang BCDK ni tip nn l hnh thang cn. T y suy ra CK = BD.
p dng nh l Pythagore cho tam gic ACK vung ti C : AC
2
+CK
2
= AK
2
Suy ra AC
2
+BD
2
= 4R
2
hay PA
2
+PB
2
+PC
2
+PD
2
= 4R
2
khng i.
(b)Trc tin, ta s chng minh rng :
IO
2
+IP
2
= R
2
Tht vy, p dng nh l Pythagore cho tam gic OIB vung ti I, ta thu c :
OB
2
= OI
2
+IB
2
Tam gic PBC vung ti P c I l trung im BC nn PI = IB. Do :
R
2
= OI
2
+IP
2
Gi M l trung im OP. Theo cng thc ng trung tuyn (c th chng minh da vo kin
thc lp 9) :
IM
2
=
2(IP
2
+IO
2
) OP
2
4
=
2R
2
OP
2
4
Do I di chuyn trn
_
M;
2R
2
OP
2
4
_
c nh.
Bi 2.22 Cho tam gic ABC v im M bt k nm trong tam gic . Chng minh rng :
MA +MB +MC + minMA, MB, MC < AB +BC +CA
Li gii
Trc ht, ta chng minh b sau y :
B : Cho t gic ABCD v im M bt k nm trong t gic . Chng minh rng :
MD +MC < DA +AB +BC
Chng minh b .
L
A
B
C
D
M
96
Xt M nm trong tam gic DBC. Gi L l giao im ca DM v BC.
p dng bt ng thc tam gic, ta c
DA +AB +BC DB +BC = DB +BL +LC
DL +LC = DM +ML +LC
DM +MC
Tng t xt M nm trong tam gic ABD, ta chng minh c :
AD +AB +BC DM +MC
Suy ra iu cn chng minh.
Tr li vi bi ton.
D
F
E
A
B C
M
Gi D, E, F theo th t l trung im ca BC, CA, AB.
D thy vi mi im M thuc tam gic ABC th tn ti t nht hai trong ba hnh thang
BCEF, CAFD, ABDE cha n. Khng mt tnh tng qut, gi s M nm trong hnh thang
BCEF v ABDE.
p dng b , ta c
_

_
MA +MB <
1
2
(AB +BC +CA)
MB +MC <
1
2
(AB +BC +CA)
Do
MA +MB +MC + minMA, MB, MC MA + 2MB +MC < AB +BC +CA
Ta c iu cn chng minh.
Bi 2.23 Tam gic cn ABC ni tip (O) c AB = AC v AQ l ng knh ca (O). Ly
M, N, P ln lt trn cnh AB, BC, CA sao cho AMNP l hnh bnh hnh. Chng minh rng
NQ MP.
Li gii
97
K
P
N
Q
A
B
C
M
(i) Cch 1.
Ly K l im i xng ca N qua MP.
Ta c

MKP =

MNP =

MAP, suy ra t gic AMPK ni tip.


Li c

MPK =

MPN =

AMP nn AP = MK. Do

MAK =

PKA (1)
Mt khc, PC = PN = PK nn tam gic PKC cn ti P hay

PKC =

PCK (2)
Ta c

ABC +

AKC +

BAK +

BCK = 360

Tng ng vi

ABC +

AKP +

PKC +

BAK +

BCK = 360

T , kt hp vi (1), (2) v tam gic ABC cn, ta suy ra

ACB +

PCK +

MAK +

BAK +

BCK = 360

Do
2
_

BCK +

BAK
_
= 360

V vy

BCK +

BAK = 180

hay K (O).
Cng t (1), ta c AK | MP hay AKNK.
Vy N, K, Q thng hng hay MPNQ.
(ii) Cch 2. S dng kin thc v vector, ta cn chng minh

QN

MP = 0.
thc hin iu ny, ta t k =
NC
BC
v ch rng QB MA, QC MN. Bin i nh
sau :

QN

MP =
_
(1 k)

QC +k

OB
_
(

MA +

MN)
= (1 k)

QC

MA +k

QB

MN
= (1 k)

BC

MA +k

CB

MN
= (1 k)BC MAcos B kBC MN cos C
Vi cos B = cos C, ta ch cn chng minh rng :
(1 k) MA = k MN
Tng ng vi
MA
MN
=
k
1 k
98
ng thc ny ng v ta c
k
1 k
=
NC
BC
NB
BC
=
NC
NB
=
MA
MB
=
MA
MN
Bi ton c chng minh.
Bi 2.24 Cho t gic ABCD c M, N ln lt l trung im AB, CD v O l giao im ca
2 ng cho. Gi H, K l trc tm ca tam gic OAB, OCD. Hy chng minh MN HK.
Li gii
K
H
O
N
M
A
D
B
C
Ta s dng vector chng minh rng :

HK

MN = 0
Trc ht, xin pht biu m khng chng minh chng minh mt b quen thuc :
2

MN =

AC +

BD
Tr li bi ton, theo b ta c c :
2

MN

HK =

AC

HK +

BD

HK
= x AC y BD
Trong , x, y ln lt l di tuyt i hnh chiu ca HK ln AC, BD. Khi , khng kh
thy rng :
x = BDsin

OBK, y = AC sin

OCK
Nhng r rng

OBK =

OCK nn x AC y BD = 0. T y ta c iu cn chng minh.


Bi 2.25 Cho t gic ABCD ni tip (O) c hai ng cho ct nhau ti I. Gi M, N ln
lt l trung im ca AB, CD. P, Q l chn ng cao k t I ca tam gic IAD, IBC.
Chng minh rng, PQ MN.
Li gii
(i) Cch 1.
99
B
K
N
M
Q
P
I
A
D
C
Ly K l trung im ca AC, khi y KM, KN l ng trung bnh ca cc tam gic ABC, ACD
nn KM =
BC
2
, KN =
AD
2
.
Hai tam gic IAD, IBC ng dng c IP, IQ l ng cao tng ng nn :
IP
IQ
=
AD
BC
=
KN
KM
Hn na, MK | BC, NK | AD nn

MKN b vi gc to bi AD, BC, nn gc ny cng


bng vi

PIQ. Do , KMN IQP.


Suy ra

KMN =

IQP, m KM IQ MN PQ, ta c iu cn chng minh.


(ii) Cch 2.
Ta c th s dng vector, tc quy v chng minh :

PQ

MN = 0
Tng t bi 2.24, ta cn chng minh rng
x AC = y BD
Trong x l hnh chiu ca PQ ln AC, y l hnh chiu ca PQ ln BD.
S dng h thc lng trong tam gic vung th :
x =
IP
2
IA
+
IQ
2
IC
, y =
IP
2
ID
+
IQ
2
IB
Ta cn chng minh ng thc sau :
AC
_
IP
2
IA
+
IQ
2
IC
_
= BD
_
IP
2
ID
+
IQ
2
IB
_
Ch do 2 tam gic IAD, IBC ng dng nn
IP
IQ
=
IA
IB
=
ID
IC
v IA IC = IB ID = T
I/(O)
.
Khi , sau khi chia 2 v ng thc trn cho IP
2
, ta c dy cc ng thc tng ng :
(IA ID
2
+IC IA
2
)(IA +IC) = (IA
2
ID +IB ID
2
)(IB +ID)
100
IA
3
IC +IA IC ID
2
= IB ID
3
+IA
2
T
I/(O)
(IA
2
ID
2
) = T
I/(O)
(IA
2
ID
2
)
ng thc cui cng hin nhin ng, bi ton c chng minh.
Bi 2.26 Cho tam gic ABC v tam gic DBC c tm ni tip ln lt l H, K. Chng
minh rng AD HK.
Li gii
H
K
B C
A
D
Trc ht, xin pht biu v khng chng minh hai b sau y :
B 1 : Cho tam gic ABC, im M nm trong tam gic . Khi
MB +MC < AB +AC
B 2 : Cho tam gic ABC ngoi tip ng trn (I). Khi ta c
BC

IA +CA

IB +AB

IC =

0
Tr li vi bi ton, tin bin i, ta k hiu BC = a, AB = c, CA = b, DB = c
1
, DC = b
1
.
T b 2 ta c
a

HA +b

HB +c

HC =

0
a

KD +b
1

KB +c
1

KC =

0
Suy ra
a

HA +b

HB +c

HC =

0
a

HD +b
1

HB +c
1

HC = (a +b
1
+c
1
)

HK
Do
a(

HD

HA) + (b
1
b)

HB + (c
1
c)

HC = (a +b
1
+c
1
)

HK
T rt ra
a

AD + (b
1
b)

HB + (c
1
c)

HC = (a +b
1
+c
1
)

HK (1)
T bt ng thc tam gic v b 1, ta c
[b b
1
[ AD, [c c
1
[ AD, HB +HC < b +c (2)
101
T (1), (2) suy ra
(a +b
1
+c
1
)HK aAD +HB.AD +HC.AD
= aAD + (BH +HC)AD
aAD + (b +c)AD
Ta suy ra
(a +b +c)AD (a +b
1
+c
1
)HK (3)
Chng minh tng t, ta c :
(a +b
1
+c
1
)AD (a +b +c)HK (4)
Cng theo v (3) v (4) ri thu gn ta c AD HK.
Bi 2.27 Cho K l im nm trong tam gic ABC. Mt ng thng qua K ct hai cnh
AB, AC theo th t M, N. Chng minh rng :
S
ABC
8
_
S
BMK
S
CNK
Li gii
A
B C
M
N
K
Ta s chng minh bt ng thc mnh hn :
3
_
S
BMK
+
3
_
S
CNK

3
_
S
ABC
Ta k hiu : KM = x
1
, KN = x
2
, MB = y
1
, MA = y
2
, NC = z
1
, NA = z
2
.
Ta c
S
BMK
S
ABC
=
S
MBK
S
MAN

S
AMN
S
ABC
=
x
1
y
1
y
2
(x
1
+x
2
)

y
2
z
2
(y
1
+y
2
)(z
1
+z
2
)
=
x
1
y
1
z
2
(x
1
+x
2
)(y
1
+y
2
)(z
1
+z
2
)
Suy ra
3
_
S
BMK
S
ABC
=
3
_
x
1
x
1
+x
2

y
1
y
1
+y
2

z
2
z
1
+z
2
p dng bt ng thc AM - GM, ta c
3
_
S
BMK
S
ABC

1
3
_
x
1
x
1
+x
2
+
y
1
y
1
+y
2
+
z
2
z
1
+z
2
_
(1)
102
Chng minh tng t ta c :
3
_
S
CNK
S
ABC

1
3
_
x
2
x
1
+x
2
+
y
2
y
1
+y
2
+
z
1
z
1
+z
2
_
(2)
Cng theo v (1), (2) ta c
3
_
S
BMK
S
ABC
+
3
_
S
CNK
S
ABC
1
Suy ra
3
_
S
BMK
+
3
_
S
CNK

3
_
S
ABC
n y, p dng AM - GM, ta c
2
6
_
S
BMK
S
CNK

3
_
S
ABC
Suy ra
S
ABC
8
_
S
BMK
S
CNK
Chng minh hon tt.
Bi 2.28 Cho tam gic ABC nhn v M l mt im thuc min trong tam gic. Gi
A
1
, B
1
, C
1
ln lt l giao im ca MA, MB, MC vi cc cnh tam gic ABC. Ly A
2
, B
2
, C
2
l cc im i xng vi M qua trung im ca B
1
C
1
, C
1
A
1
, A
1
B
1
. Chng minh rng
AA
2
, BB
2
, CC
2
ng quy.
Li gii
(i) Cch 1.
G
P
A
2
C
1
M
A
B C
A
1
B
1
S
Gi S l im i xng ca M qua trung im P ca BC. Do trung im ca MA, B
1
C
1
, BC
thng hng (v chng nm trn ng thng Gauss ca t gic ton phn AB
1
MC
1
BC) nn
A, A
2
, S cng thng hng.
Gi G l trng tm tam gic ABC, khi GA =
2
3
GP. Xt tam gic AMS c AP l trung
103
tuyn m GA =
2
3
GP nn G cng l trng tm ca tam gic ny. iu ny chng t AA
2
ct
MG ti im Q chia on MG theo t s 3 : 1.
L lun tng t, ta thy BB
2
, CC
2
cng i qua Q. Ta c iu cn chng minh.
(ii) Cch 2.
p dng nh l hm s sin cho hai tam gic AA
2
C
1
, AA
2
B
1
v ch MB
1
A
2
C
1
l hnh bnh
hnh suy ra :
sin

BAA
2
= sin

AC
1
A
2

C
1
A
2
AA
2
= sin

ABM
MB
1
AA
2
sin

CAA
2
= sin

AB
1
A
2

B
1
A
2
AA
2
= sin

ACM
MC
1
AA
2
Do
sin

BAA
2
sin

CAA
2
=
sin

ABM
sin

ACM

MB
1
MC
1
V vy

sin

BAA
2
sin

CAA
2
=

sin

ABM
sin

ACM

MB
1
MC
1
Theo nh l Ceva cho tam gic ABC th :

sin

ABM
sin

ACM
= 1
Do ,

sin

BAA
2
sin

CAA
2
= 1
Cng theo nh l Ceva cho tam gic ABC, ta suy ra AA
2
, BB
2
, CC
2
ng quy.
Bi 2.29 Cho tam gic ABC ni tip (O; R) c M thuc cung BC khng cha A. Tm v
tr ca M P = 2010 MB + 2011 MC t gi tr ln nht.
Li gii
Gi T l im trn cung BC cha A sao cho 2010 TB = 2011 TC. Suy ra T c nh.
p dng nh l Ptolemy cho t gic TBMC ni tip (O) ta c
TB CM +TC BM = BC TM
Do
2011 TC
2010
CM +TC BM = BC TM
V vy
P = 2011 CM + 2010 BM =
2010 BC TM
TC
V T, B, C c nh nn P ln nht khi v ch khi TM ln nht, tc l TM phi l ng knh
ca (O).
104
Bi 2.30 Cho tam gic ABC. Cc im D, E, F nm trn cc cnh BC, CA, AB sao cho
AD, BE, CF ng quy ti O. Qua O k ng thng song song vi BC ct DE, DF theo th
t ti H v K. Chng minh O l trung im HK.
Li gii
(i) Cch 1.
Q H
K
P
F
O
A
B C
D
E
Gi P, Q l giao im ca ng thng HK vi AB, AC.
p dng nh l Thales, ta c :
PO
PQ
=
BD
BC
v
KO
PO
=
CD
BC
Suy ra
KO
PQ
=
PO
PQ

KO
PO
=
BD CD
BC
2
(1)
Tng t, ta c
HO
PQ
=
BD CD
BC
2
(2)
T (1) v (2), ta c iu cn chng minh.
(ii) Cch 2.
K
H
M N
F O
A
B C
D
E
105
Qua A k ng thng song song vi BC ct cc tia DE, DF ti M, N.
p dng nh l Ceva v nh l Thales, ta c dy cc ng thc tng ng sau :
AF
FB

BD
DC

CE
EA
= 1
AN
BD

BD
DC

CD
AM
= 1
AN
AM
= 1
AM = AN
p dng nh l Thales mt ln na, ta suy ra OK = OH.
Bi 2.31 Cho tam gic ABC. M l mt im bt k trn mt phng v khng nm trn
tam gic ABC. Cc ng thng AM, BM, CM ln lt ct cc ng thng BC, CA, AB
ti D, E, F. Gi H, K ln lt l giao im ca cc cp ng thng BM vi FD; CM vi
ED. Chng minh cc ng thng AD, BK, CH ng quy.
Li gii
H
K
F
M
A
B C
D
E
Ta c
MH
BH
=
S
FMD
S
FBD
=
S
FMD
S
AFD
S
FBD
S
AFD
=
DM
DA
BF
FA
=
MD FA
AD FB
(1)
Tng t :
CK
KM
=
CE AD
EA MD
(2)
T (1) v (2) suy ra
MH
BH

CK
KM
=
AF
BF

CE
EA
MH
BH

CK
KM

BD
DC
=
AF
BF

CE
EA

BD
DC
Theo nh l Ceva, ta c
AF
BF

CE
EA

BD
DC
= 1
106
Do ,
MH
BH

CK
KM

BD
DC
= 1
Theo nh l Ceva o, ta c iu cn chng minh.
Bi 2.32 Cho t gic li ABCD. Chng minh :
minAB, BC, CD, DA

AC
2
+BD
2
2
maxAB, BC, CD, DA
Li gii
E
J
I
D
A
C
B
t m = minAB, BC, CD, DA.
Ta c hai nhn xt sau
Nhn xt 1. Trong tam gic ABC ta c :

BAC 90

BC
2
AB
2
+AC
2
Nhn xt 2. Trong t gic ABCD, gi I, J theo th t l trung im ca AC, BD; ta c
AB
2
+BC
2
+CD
2
+DA
2
= AC
2
+BD
2
+ 4IJ
2
Tr li bi ton :
Bt ng thc

AC
2
+BD
2
2
maxAB, BC, CD, DA l h qu trc tip ca nhn xt 2.
Ta chng minh bt ng thc bn tri :

BAD +

ADC +

ABC +

BCD = 360

Khng mt tnh tng qut, ta c th gi s :

BAD +

ADC 180

BAD 90

.
Dng hnh bnh hnh ABED. Khi , DE nm gia DB, DC.
Gi I, J theo th t l trung im ca AC, BD.
Trong tam gic ACE c IJ l ng trung bnh nn CE = 2IJ.
C 2 trng hp xy ra :
Trng hp 1 : E nm trong t gic ABCD. Trong 2 gc

AEB,

AED c t nht mt gc
nhn.
107
Khng mt tnh tng qut, ta gi s

AEB 90

.
Ta suy ra

CEB 90

. Theo nhn xt 1, ta c
BC
2
BE
2
+CE
2
BC
2
4IJ
2
AD
2
S dng nhn xt 2, ta c
AC
2
+BD
2
= AB
2
+AD
2
+CD
2
+ (BC
2
4IJ
2
)
AB
2
+AD
2
+CD
2
+AD
2
4m
2
Suy ra m

AC
2
+BD
2
2
.
Trng hp 2 : E nm ngoi t gic ABCD.
Khi CB nm gia CD, CE. Do :

BEC

BED =

BAD 90

.
Chng minh hon ton tng t trng hp 1. (Bn c t chng minh)
Vy bi ton c chng minh.
Bi 2.33 Cho ng trn (O; R) v hai im A, B c nh i xng vi nhau qua O. Gi
M l im chy trn (O). ng thng MA, MB ct (O) ti P, Q tng ng. Chng minh
rng gi tr biu thc
MA
AP
+
MB
BQ
khng i khi M di chuyn trn (O).
Li gii
T gi thit suy ra phng tch ca im A v B vi (O) l nh nhau. Do :
MA
AP
+
MB
BQ
=
MA
2
T
A/(O)
+
MB
2
T
B/(O)
=
MA
2
+MB
2
T
=
2MO
2
+
AB
2
2
T
V A, B c nh v MO = R khng i nn
2MO
2
+
AB
2
2
T
khng i, ta c iu cn chng
minh.
Bi 2.34 Cho (O) v dy AB. im M di chuyn trn cung ln AB. Cc ng cao AE, BF
ca ABM ct nhau ti H. K (H; HM) ct MA, MB C v D. Chng minh ng thng
k t H vung gc vi CD lun i qua mt im c nh khi M di chuyn trn cung ln AB.
Li gii
108
I
O'
D
C
H
E
F
O
B A
M
T gi thit ta c E, F tng ng l cc trung im ca cc on thng MD, MC. Suy ra
EF | CD.
Theo mt kt qu quen thuc th OM EF. Do OM CD.
Gi O

l im i xng vi O qua AB th OO

v AB vung gc vi nhau ti trung im I


ca mi ng
Theo mt tnh cht quen thuc ca trc tm tam gic th ta c MH = 2OI MH = OO

M MH | OO

(cng vung gc vi BC) nn MHO

O l hnh bnh hnh. Suy ra HO

| MO.
T ta c HO

CD.
Vy khi M di chuyn trn cung ln AB th ng thng qua H vung gc vi CD lun i qua
im O

c nh.
Bi 2.35 Cho tam gic ABC ni tip ng trn (O). G l trng tm tam gic. AG, BG, CG
ln lt ct (O) ti A
1
, B
1
, C
1
. Chng minh rng :
GA
1
+GB
1
+GC
1
GA +GB +GC
Li gii
C
1
B
1
A
1
G
O
A
B
C
109
K hiu l phng tch ca G i vi (O). Ta c
GA
1
+GA
2
+GA
3
=
GA
1
GA
GA
+
GB
2
GB
GB
+
GC
2
GC
GC
=
_
1
GA
+
1
GB
+
1
GC
_

9
GA +GB +GC
S dng h thc Jacobi, ta c
=
GA
2
+GB
2
+GC
2
3

(GA +GB +GC)
2
9
Thay nh gi ny vo bt ng thc trn, ta c iu cn chng minh.
Nhn xt.
Nu thay trng tm G bng tm ng trn ni tip tam gic bi I v A
1
, B
1
, C
1
l giao im
ca cc tia AI, BI, CI vi (O) th bi ton vn ng, tc l
IA
1
+IB
1
+IC
1
IA +IB +IC
Bi 2.36 Cho ABC v D, E, F ln lt l hnh chiu ca A, B, C xung ba cnh tng
ng. ng thng qua D song song vi EF ct AB, AC ti P, Q. Bit EF BC = R. Chng
minh rng ng trn ngoi tip PQR i qua trung im BC.
Li gii
M
Q
P
R
H
F
E
D
A
B
C
V D, E, F, M ng vin (ng trn Euler ca tam gic ABC), ta c
RD RM = RE RF
V B, E, F, C ng vin nn ta c
RB RC = RE RF
Suy ra
RB RC = RD RM (1)
110
Mt khc, ta c
(PQ, PA) (FE, FA) (CA, CB) (mod )
Do B, C, P, Q ng vin. Suy ra
DB DC = DP DQ
chng minh P, Q, R, M ng vin th ta cn chng minh
DP DQ = DR DM
Tng ng vi
DB DC = DR DM
Bin i t (1) ta c ngay iu cn chng minh.
Bi 2.37 Cho t gic li ABCD ni tip ng trn (O). Cho AB = a, CD = b,

AIB = ,
trong I l giao im ca hai ng cho AC v BD. Tnh bn knh ng trn (O) theo
a, b v .
Li gii
b
a

I
O
A
D C
B
Ta c

AIB =

AOB
2
+

COD
2
.
Suy ra
cos = cos

AOB
2
cos

COD
2
sin

AOB
2
sin

COD
2
=

_
1
a
2
4R
2
__
1
b
2
4R
2
_

ab
4R
2
Nh vy, t ng thc trn suy ra
_
4R
2
cos +ab
_
2
=
_
4R
2
a
2
_ _
4R
2
b
2
_
Tng ng vi
4R
2
_
a
2
+b
2
+ 2ab cos
_
= 16R
4
sin
2

111
V vy
R =

a
2
+b
2
+ 2ab cos
2 sin
Ta c c p s ca bi ton.
Bi 2.38 Cho ABC c trc tm H. ng trn qua B, C ct AB, AC ti D, E. Gi F l
trc tm ADE v I l giao im ca BE v CD. Chng minh rng I, H, F thng hng.
Li gii
I
F
E
H
A
B C
D
Gi F
1
, F
2
l hnh chiu vung gc ca F ln AB, AC; H
1
, H
2
l hnh chiu vung gc ca H
ln AB, AC.
Theo mt kt qu quen thuc v trc tm tam gic, ta c
FF
1
FE = FF
2
FD v HH
1
HC = HH
2
HB
Mt khc, ta c
IB IE = IC ID
Suy ra F, H, I cng thuc trc ng phng ca ng trn ng knh BD v ng trn
ng knh CE. Do ta c iu cn chng minh.
Bi 2.39 Cho ABC khng cn, ngoi tip ng trn (I). Tip im ca (I) trn
BC, CA, AB ln lt l D, E, F. DE ct AB P. Mt ng thng qua C ct AB, FE
ln lt N, M. PM ct AC Q. Chng minh rng IN vung gc vi FQ.
Li gii
112
T
Q
N
P
E
D
F I
A
B
C
M
Gi giao im ca FQ vi (I) l T ,= F. Gi s TD EF = M

.
p dng nh l Pascal cho 6 im E, E, D, F, T, F ta c Q, M

, P thng hng, suy ra M M

.
T N k tip tuyn tip xc vi (I) ti T

v ct AC ti S.
Theo mt tnh cht quen thuc, do t gic NSCB ngoi tip nn EF, T

D, NC, SB ng quy,
t ta c T T

. Suy ra FQ l ng i cc ca N i vi (I). T ta c iu cn chng


minh.
Bi 2.40 Cho t gic ABCD. Gi I, J theo th t l trung im ca AC, BD. Chng minh
rng :
AC +BD + 2IJ < AB +BC +CD +DA
Li gii
C
J
I
E B
A D
Trc ht, xin pht biu m khng chng minh li b sau :
B : Trong mt t gic li, tng di hai ng cho nh hn chu vi v ln hn tng
di hai cnh i ca t gic.
Tr li vi bi ton, c 2 trng hp xy ra.
113
Trng hp 1 : T gic ABCD c t nht mt cp cnh i song song.
Khng mt tnh tng qut, gi s AB | CD v AB < CD.
Khi d thy : 2IJ = CD AB. Ta c :
AB +BC +CD +DA = (AB +BC) + (BA +AD) +CD AB
> AC +BD + 2IJ
T y ta c iu cn chng minh.
Trng hp 2 : T gic ABCD c cc ng thng cha cp cnh i ct nhau.
Khng mt tnh tng qut, gi s

ABC l gc ln nht ca t gic ABCD.


Khi ta c :

ABC +

BCD 180

hoc

ABC +

BAD 180

Tht vy, nu

ABC +

BCD < 180

ABC +

BAD < 180

.
Suy ra

ABC +

BCD +

CDA +

DAB (

ABC +

BCD) + (

ABC +

BAD)
< 360

iu ny v l. Do ta c th gi s

ABC +

BCD 180

.
Dng hnh bnh hnh ABED. Khi BE nm gia BA v BC. (1)
Li c :

ADE =

ABC

ADC DC nm gia DB v DE. (2)


T (1), (2) suy ra BCED l t gic li.
D thy rng CE = 2IJ. (3)
p dng b , ta c
CE +BD < CD +BE = CD +AD (4)
Theo bt ng thc tam gic, ta c AC < AB +BC. (5)
T (3), (4), (5) ta c
AC +BD + 2IJ < AB +BC +CD +DA
Chng minh hon tt.
Bi 2.41 Cho ABC ni tip ng trn (O). E thuc cung BC khng cha A v khng
trng B, C. AE ct tip tuyn ti B, C ca (O) ti M, N. Gi giao im ca CM v BN l
F. Chng minh rng EF lun i qua mt im c nh khi E di chuyn trn cung BC khng
cha A.
Li gii
(i) Cch 1.
114
J
L
P
K
Q
F
N
M
A
B
C
E
Gi K l giao im ca tip tuyn ti B v C ca (O). AK ct (O) ti J; AE, AK ln lt ct
BC ti P, Q; FK ct BC, AM ln lt ti L, I.
Ta c (LPBC) = 1, suy ra (EL, EP, EB, EC) = 1.
Li c (EJ, EA, EB, EC) = 1 nn L, E, J thng hng.
Mt khc (EL, EI, EF, EK) = (EJ, EA, EQ, EK) = 1 nn theo php chiu xuyn tm E ta
suy ra c F, E, Q thng hng.
Vy EF i qua Q c nh.
(ii) Cch 2.
G
T
Q
L
F
M
P
J
N
K
C
A
B
E
115
Cng gi K l giao im ca hai tip tuyn; P, Q l giao im ca AE, AK vi BC; L l giao
im ca FK vi BC.
Xt cc - i cc vi (O). Gi T l cc ca AB, v J l cc ca CE. Ta c G = AB CE l
cc ca TJ. M P = AE BC nn T, P, J thng hng.
Ta c F v K l hai im lin hp vi P, suy ra P l cc ca FK.
Do FK, CE, AB ng qui ti G.
p dng nh l Pappus cho hai b 3 im (G, B, A) v (N, K, C), ta suy ra E, F, Q thng hng
hay EF i qua Q c nh.
Bi 2.42 Cho t gic ABCD ni tip tha mn AB CD = AD BC. ng trn (C) qua
A, B v tip xc vi BC, ng trn (C

) qua A, D v tip xc CD. Chng minh rng giao


im khc A ca (C) v (C

) l trung im BD.
Li gii
E
B
A
C
D
Gi E l trung im BD.Ta s chng minh rng (AEB) tip xc vi BC v (AED) tip xc
vi CD
Tht vy, p dng nh l Ptolemy, ta c
2BE AC = BD AC = AB CD +AD BC = 2AB CD
Suy ra
BE AC = AB CD
Tng ng vi
BA
BE
=
CA
CD
Mt khc, ta c

ABD =

ACD. Do AEB ADC.


Suy ra

CBD =

CAD =

BAE. V vy (AEB) tip xc vi BC ti B.


Chng minh tng t nh trn ta cng c (ADE) tip xc CD. Vy trung im E ca BD l
im chung khc A ca (C) v (C

).
116
Bi 2.43 Cho tam gic nhn ABC, gi H l trc tm ca tam gic. Tm iu kin cn v
i vi cc gc ca tam gic 9 im : chn cc ng cao ca tam gic, trung im cc
cnh ca tam gic, trung im cc on thng HA, HB, HC l nh ca mt a gic u.
Li gii
N
M
P
X
Z
Y
F
E
D
A
B
C
Gi M, N, P ln lt l trung im cc cnh BC, CA, AB; D, E, F ln lt l chn ng cao
h t A, B, C; X, Y, Z ln lt l trung im HA, HB, HC. Ta c ba trng hp sau :
Trng hp 1. C t nht hai trong 3 b (M, D); (N, E); (P, F) c hai im trong b trng
nhau.
T suy ra ABC u v M D, N E, P F.
Khng kh chng minh MZNXPY l lc gic u.
Trng hp 2. C ng mt trong 3 b (M, D); (N, E); (P, F) c hai im trong b trng
nhau.
Gi s l (M, D), khi ABC cn ti A.
Ta c MZENXPFY l bt gic u v do ABC nhn nn
_
_
_

M =

Z =

E =

N =

X =

P =

F =

Y = 135

MZ = ZE = EN = NX = XP = PF = FY = Y M
hay tng ng vi :
_

A = 45

;

B =

C = 67, 5

AB

AB
2
= AB cot(67, 5


A = 45

;

B =

C = 67, 5

Trng hp 3. Khng c b no trong ba b (M, D); (N, E); (P, F) c hai im trong
b trng nhau.
Khng mt tnh tng qut, gi s on EF khng ct on NP.
iu kin cn tha mn iu kin bi ton l

EXF = 140



A = 70

.
Cng khng mt tng qut, gi s on DF khng ct on MP.
Do , thm iu kin cn na l

MY P = 140



B = 40

,

C = 70

.
Do ta thy mu thun.
117
Vy iu kin cn v 9 im D, E, F, M, N, P, X, Y, Z l cc nh ca mt a gic u l

A = 45

;

B =

C = 67, 5

hoc ABC u.
Bi 2.44 Cho tam gic ABC. ng trn (I) ni tip tam gic ABC v tip xc vi
BC, AC, AB ln lt ti D, E, F. Chng minh rng ID, EF v trung tuyn AM (M BC)
ng quy.
Li gii
V
U
N
M
E
D
F
I
A
B C
Gi N = ID EF v M

= AN BC.
Ta s chng minh M M

. Tht vy, qua N dng ng thng vung gc vi ID ct AB, AC


ln lt ti U, V .
Khi cc b 4 im (I, F, U, N) v (I, V, E, N) ng vin. Suy ra

IUN =

IFN =

IEN =

IV N
Nh vy ta c IUV cn ti I. Do NU = NV . p dng nh l Thales, ta suy ra
M

B = M

C hay M

l trung im ca BC.
Bi 2.45 Cho hai on thng AB v A

bng nhau. Php quay tm M bin A thnh A

,
bin B thnh B

. Php quay tm N bin A thnh B

, bin B thnh A

. Gi S l trung im
ca AB. Chng minh rng SM vung gc vi SN.
Li gii
118
M
N
S'
S
A
B
A'
B'
Gi S, S

ln lt l trung im AB, A

. Gi f l php quay tm M bin A A

, B B

;
f

l php quay tm N bin A B

, B A

.
Theo gi thit ta c : f(S) = S

; f

(S) = S

. Do ta c
_
(SB, SN) (S

, S

N) (mod )
(SM, SA) (S

M, S

) (mod )
Suy ra
(SN, SM) (S

M, S

N) (mod )
V vy S, S

, M, N ng vin.
Li c MS = MS

, NS = NS

nn suy ra

MSN =

MS

N = 90

.
Bi 2.46 Cho tam gic ABC, M l im nm trong tam gic. AM, BM, CM ct
BC, CA, AB theo th t D, E, F. Gi H, I, K theo th t l hnh chiu ca M trn
BC, CA, AB . K hiu P(HIK) l chu vi tam gic HIK. Hy chng minh :
P(DEF) P(HIK)
Li gii
U
Y
Z V
X
T
K
I
H
F
M
A
B
C
D
E
119
Ta c hai b sau y :
B 1 : Cho im M nm trong gc

xOy. A, B theo th t l cc im khc O thuc tia
Ox, Oy; H, K theo th t l hnh chiu ca M trn Ox, Oy. Khi , ta c
P(MAB) 2HK
Chng minh b 1.
Gi M
1
, M
2
theo th t l im i xng ca M qua Ox, Oy. Ta c : M
1
M
2
= 2HK. C hai
trng hp xy ra :
Trng hp 1 :

xOy < 90

.
Ta c P(MAB) = MA +MB +AB = M
1
A +M
1
B +AB M
1
M
2
= 2HK.
Trng hp 2 :

xOy 90

.
Khi , M
1
M
2
i qua O hoc M
1
M
2
khng ng thi ct tia Ox, Oy.
Ta c bt ng thc thc s : MA +MB +AB = M
1
A +M
1
B +AB > 2HK.
Do , ta lun c P(MAB) > 2HK.
ng thc xy ra khi v ch khi O l tm bng tip gc M ca MAB.
B 2 : Cho tam gic ABC, M l im nm trong tam gic. AM, BM, CM ct BC, CA, AB
theo th t D, E, F. Ta c
AM
AD
+
BM
BE
+
CM
CF
= 2
Vic chng minh b 2 kh n gin, xin dnh cho bn c.
Tr li bi ton :
Qua M k ng thng song song vi EF, ct AB, AC ti X, Y ; song song vi FD ct BC, BA
ti Z, T; song song vi DE ct CA, CB ti U, V
Cc tam gic MUT, V MX, ZY M ng dng vi tam gic DEF theo cc t s tng ng l :
AM
AD
,
BM
BE
,
CM
CF
T b 1, ta c P(MUT) +P(V MX) +P(ZY M) 2IK + 2KH + 2HI.
T suy ra P(MUT) +P(V MX) +P(ZY M) 2P(HIK). (1)
T b 2, ta c :
AM
AD
+
BM
BE
+
CM
CF
= 2
Suy ra
2P(DEF) =
AM
AD
P(DEF) +
BM
BE
P(DEF) +
CM
CF
P(DEF)
Tng ng vi
2P(DEF) = P(MUT) +P(V MX) +P(ZY M) (2)
T (1) v (2) ta suy ra iu cn chng minh.
120
Bi 2.47 Tam gic ABC nhn ni tip (O), ng cao AH ct (O) ti A

. OA

ct BC ti
A

. Xc nh tng t cho B

, C

. Chng minh AA

, BB

, CC

ng quy.
Li gii
B''
C''
A''
C'
B'
A'
O
A
B
C
Ta c

OBC = 90


A v

CBA

CAA

= 90


C, suy ra

OBA

=

B.
Li c OB = OA

nn

OA

B =

B. Suy ra

BOA

= 180

B.
Tng t ta c

COA

= 180

C.
p dng nh l sin trong tam gic ta c
BA

sin

BOA

=
OA

sin

OBA

v
CA

sin

COA

=
OA

sin

OCA

Suy ra
BA

sin

BOA

=
CA

sin

COA

Do
BA

CA

=
sin(180

2B)
sin(180

2C)
=
sin 2B
sin 2C
Tng t cho hai im E, F v p dng nh l Ceva, ta c iu cn chng minh.
Bi 2.48 Cho ng trn (O) v mt ng thng d c nh. Gi H l hnh chiu ca ca
O trn d. Ly M c nh thuc ng trn. A, B thay i trn d sao cho H l trung im
AB. Gi s AM, BM ct (O) ti P, Q. Chng minh PQ lun i qua mt im c nh.
Li gii
121
R
N
K
Q
T
S
P
B H
O
M
A
Nu M, O, H thng hng, khi ta c PQ lun song song vi (d). Do ta ch xt trng hp
M, O, H khng thng hng.
Gi s (d) khng ct (O) (cc trng hp khc chng minh tng t).
Khng mt tnh tng qut, gi s M v B cng pha so vi OH. T P k ng thng d

song
song vi d ct MH, MB tng ng ti S, T. Gi N l trung im ca PQ v R l giao im
khc M ca MH vi (O).
Ta c NS | QT, suy ra
(NP, NS) = (QP, QT) = (RP, RS) (mod )
Do P, N, R, S ng vin. V vy
(RN, RH) = (PN, PS) = (KN, KH) (mod )
hay N, R, H, K ng vin, m

ONK =

OHK =

2
nn O, H, N, K ng vin. Nh vy ta suy
ra K l giao im ca (d) vi (OHR) nn K l im c nh.
Vy PQ lun i qua K c nh.
Bi 2.49 Cho ng trn tm I ni tip tam gic ABC tip xc vi BC, AB, AC ti D, E, F.
Qua E v ng song song vi BC ct AD, DF M, N. Chng minh rng M l trung im
ca EN.
Li gii
P
N
M
E
D
F
I
A
B C
122
Qua A dng ng thng (d) song song vi BC v ct DF ti P.
T cch dng trn suy ra MN | AP. Do theo nh l Thales ta c
MN
AP
=
DM
AD
Mt khc, cng theo nh l Thales, ta c
EM
AE
=
CD
CA
=
CE
CA
=
DM
AD
T hai ng thc trn ta suy ra
EM
AE
=
MN
AP
Mt khc, d thy AP = AF = AE nn suy ra EM = MN.
Vy M l trung im EN.
Bi 2.50 Cho tam gic ABC c AB = c, BC = a, AC = b v I l tm ng trrn ni
tip. Hai im B

, C

ln lt nm trn hai cnh AB, AC sao cho B

, C

, I thng hng. Chng


minh rng
S
ABC

a +b +c
2

bc

_
S
AB

C
S
ABC

Li gii
(i) Cch 1.
C'
D
I
A
B C
B'
Gi D l chn ng phn gic trong gc A.
Trc tin ta c cc kt qu quen thuc sau :
BD =
ac
b +c

AD =
b
b +c

AB +
c
b +c

AC
V I l chn ng phn gic trong ca tam gic ABD nn :
AI
DI
=
BA
BD
=
b +c
a

AI
AD
=
b +c
a +b +c
123
Ta suy ra :

AI =
b +c
a +b +c

AD
=
b +c
a +b +c
_
b
b +c

AB +
c
b +c

AC
_
=
b
a +b +c

AB +
c
a +b +c

AC
=
bAB
(a +b +c)AB

AB

+
cAC
(a +b +c)AC

AC

Mt khc B

, I, C

thng hng nn
bAB
AB

(a +b +c)
+
cAC
AC

(a +b +c)
= 1. Tng ng vi :
a +b +c =
bAB
AB

+
cAC
AC

AMGM
2
_
bAB
AB


cAC
AC

= 2

bc
_
AB AC
AC

AC

AB AC
AC

AB
= 2

bc

S
2
ABC
S
AB

C
S
AC

B
Suy ra :
S
ABC

a +b +c
2

bc

_
S
AB

C
S
ABC

n y chng minh hon tt.


(ii) Cch 2.
Bnh phng v chuyn v, bt ng thc u bi tng ng vi :
4bc
(a +b +c)
2

S
AB

C
S
ABC

S
ABC

S
ABC
4bc
(a +b +c)
2

AB

AC

AB AC
4b
2
c
2
(a +b +c)
2
AB

AC

Ta c b sau :
AB

AC


IA
2
cos
2
A
2
Xin khng chng minh b ny, bn c c th xem nh bi tp.
Tip theo, ta c cc ng thc :
IA =
_
bc(b +c a)
a +b +c
,
124
cos
2
A
2
=
cos A + 1
2
=
b
2
+c
2
a
2
2bc
+ 1
2
=
(b +c a)(a +b +c)
4bc
T thy rng :
AB

AC


IA
2
cos
2
A
2
=
4b
2
c
2
(a +b +c)
2
Bt ng thc c chng minh.
Bi 2.51 Cho t gic ABCD ni tip. E, F, G, H ln lt l tm ng trn ni tip cc
tam gic ABC, BCD, CDA, DAB. Chng minh rng t gic EFGH ni tip.
Li gii
H
G F
E
A
B
D
C
Ta c

DGC = 90

DAC
2
= 90

+
DBC
2
=

DFC
Suy ra t gic DGCF ni tip. Tng t, cc t gic CFEB, AHEB, AHGD ni tip.
T suy ra

EFG = 360

EFB +

BFC +

CFD +

DFG
_
= 360

ECB +

BFC +

CFD +

DCG
_
= 360

ACB
2
+ 90

BAC
2
+ 90

DAC
2
+

ACD
_
= 90

125
Chng minh tng t cho cc gc cn li, ta suy ra EFGH l hnh ch nht.
Bi 2.52 Cho hnh vung ABCD. I ty thuc AB, DI ct BC ti E, CI ct AE ti F.
Chng minh rng BF DE.
Li gii
G
K
T
F
E
D C
A B
I
Cho BF, AC ln lt ct DE ti T, K. Suy ra (KTIE) = 1.
Gi giao im ca ng trn ngoi tip ABCD vi DE l N. AN ct BC ti G.
Ta c :

DNC =

CNB =

BNG =

CNE = 45

.
Suy ra NC l phn gic ngoi v NG l phn gic trong ca tam gic BNE.
Do (CGBE) = 1 hay (KNIE) = 1 (xt php chiu xuyn tm A)
V vy N T. Ta c iu cn chng minh.
Bi 2.53 Cho tam gic ABC khng vung ni tip ng trn (O), trc tm H. d l ng
thng bt k qua H. Gi d
a
,d
b
, d
c
ln lt l cc ng thng i xng vi d qua BC, CA, AB.
Chng minh rng d
a
, d
b
, d
c
ng quy ti mt im trn (O).
Li gii
M
H
3
H
A
B
C
126
Gi H
1
, H
2
, H
3
ln lt l cc giao im th hai ca AH, BH, CH vi (O).
Ta c S
AB
: H H
3
. Cho nn H
3
d
c
. Tng t H
1
d
a
, H
2
d
b
.
Mt khc S
AB
: d
c
d v S
BC
: d d
a
.
Do S
BC
S
AB
= R
[B,2(BA,BC)]
: d
c
d
a
Suy ra (d
c
, d
a
) 2(BA, BC) (mod )
Gi giao im ca d
a
v d
c
l M. Ta c :
(CH
3
, CH
1
) 2(CH, CB) 2
_

2
(BA, BC)
_
(mod )
Nh vy th MCH
3
H
1
ni tip suy ra M nm trn (ABC).
Mt khc, ta c
(d
c
, MH
2
) (CH
3
, CH
2
) 2(AB, AC) (mod )
Nhng d
b
li qua H
2
v to vi d
c
mt gc 2(AB, AC) (chng minh tng t trn). Nh vy,
MH
2
trng vi d
b
, ta c iu cn chng minh.
Bi 2.54 Cho hnh thang ABCD (AB | CD). AC ct CD ti O. Bit khong cch t O
n AD v BC bng nhau, hy chng minh rng ABCD l hnh thang cn.
Li gii
O
H
N
D
M
A
B
C
Gi H l giao im ca AD, BC. M, N ln lt l giao im ca cc cp ng thng
(HO, AB), (HO, CD).
Suy ra M, N ln lt l trung im ca AB, CD.
V cc khong cch t O n AD, CB bng nhau nn HO l phn gic

DHC. Suy ra tam gic


HDC cn ti H do c ng phn gic cng l ng trung tuyn.
Vy ABCD l hnh thang cn.
Bi 2.55 Cho tam gic ABC cn ti A. ng trn tip xc AB, AC, ct BC ti K. AK
ct ti im th hai l M. P, Q l im i xng ca K qua B, C. Chng minh rng ng
trn ngoi tip tam gic MPQ tip xc vi .
Li gii
127
E
M
Q P
K
D
C B
A
Gi D, E ln lt l tip im ca vi AB, BC; P

l giao im ca MD v BC.
Ta c DMEK l t gic iu ha nn D(DEKM) = 1 hay D(BEKP

) = 1. M DE | P

K
nn B l trung im P

K hay P

P. V vy m M, D, P thng hng.
Tng t ta cng c M, E, Q thng hng.
Li c DE | PQ nn tn ti mt php v t : bin DE thnh PQ.
Suy ra : : (MDE) (MPQ). Vy hai ng trn v (MPQ) tip xc vi nhau ti M.
Bi 2.56 Cho tam gic ABC vung ti A c

B = 20

, phn gic trong BI. im H nm


trn cnh AB sao cho

ACH = 30

. Hy tnh s o

CHI.
Li gii
(i) Cch 1.
L
K
H
I
A
B
C
K phn gic CK ca gc

HCB.
Gi L l hnh chiu ca K trn BC.
Hai tam gic BLK v BAC ln lt vung ti L, A v c gc B chung nn chng ng
dng, suy ra
LB
AB
=
KB
BC
=
KH
CH
128
Li c tam gic BKC cn ti K nn L l trung im BC. V vy m
BC
AB
=
2KH
CH
=
KH
AH
Hay
IC
IA
=
HK
HA
T , theo nh l Thales th ta c HI | CK. Vy

CHI =

HCK = 20

.
(ii) Cch 2.
t

CHI =

AHI = 60

.
p dng nh l sin cho tam gic CHI ta c
CI
sin

CHI
=
HI
sin

ACH
Suy ra
CI
sin
=
HI
1
2
(1)
Ta li c :
HI
AI
=
1
sin

AHI
=
1
sin(60

)
(2)
T (1) v (2) ta c :
CI
AI
=
CI
HI

HI
AI
=
2 sin
sin(60

)
=
2 sin
cos(30

+)
M BI l phn gic

ABC nn
AI
CI
= cos 20

.
Suy ra
cos(30

+)
2 sin
= cos 20

hay cos(30

+) = 2 cos 20

sin ()
M 0

60

nn v tri l hm nghch bin, v phi l hm ng bin


Do phng trnh () c nghim duy nht = 20

.
Vy

CHI = 20

.
Bi 2.57 Cho tam gic ABC ngoi tip (I). Gi D, E, F ln lt l im i xng vi I
qua BC, CA, AB. Chng minh rng AD, BE, CF ng quy.
Li gii
129
E
D
F
I
A
B C
Ta c :
sin

BAD
sin

CAD
=
sin

BAD
sin

ABD

sin

ACD
sin

CAD

sin

ABD
sin

ACD
=
BD
AD

AD
CD

sin
3B
2
sin
3C
2
=
IB
IC

sin
3B
2
sin
3C
2
Chng minh tng t cho
sin

ACF
sin

BCF
v
sin

CBE
sin

ABE
, ta suy ra :

sin

BAD
sin

CAD
=

IB
IC

sin
3B
2
sin
3C
2
= 1
Theo dng nh l Ceva dng sin, ta c iu cn chng minh.
Bi 2.58 Cho tam gic ABC cn ti A ni tip (O). im M l trung im ca AC. BM
ct li (O) ti im th hai l Q. Chng minh rng 2AQ BQ.
Li gii
Q
M
A
O
B C
130
t AB = AC = a, BC = b (2a > b). Ta c :
2BM =

a
2
+ 2b
2
Tam gic AMQ ng dng vi tam gic BMC nn :
AQ
BC
=
AM
BM
AQ =
BC AM
BM
=
ab

a
2
+ 2b
2
Theo h thc lng trong ng trn th :
MQ MB = MA MC MQ =
a
2
2

a
2
+ 2b
2
Vy ta cn chng minh
4ab

a
2
+ 2b
2

a
2
+ 2b
2
+
a
2

a
2
+ 2b
2
Tng ng vi
4ab a
2
+ 2b
2
+a
2
Hay (a b)
2
0. Bt ng thc ny hin nhin ng nn ta c iu cn chng minh.
Nhn xt. Mt kt qu rng hn hn l :
BQ maxAC, 2AQ
C hai bt ng thc u c chng minh t ng thc
BQ =
1
2
a
_
b
AQ
+
AQ
b
_
Trong , a = AB = AC, b = BC. Vic chng minh ng thc ny xin dnh cho bn c.
Bi 2.59 Cho ABC tha mn AB+BC = 3CA. ng trn ni tip (I) tip xc AB, BC
ti D, E. Gi K, L tng ng i xng vi D, E qua I. Chng minh rng t gic ACKL ni
tip.
Li gii
G
F
M
L
K
E
D
I
B
C
A
131
Gi G l giao im CK, AB; F l giao im AL, BC; M l giao im AL, CK.
t BC = a, CA = b, AB = c,
a +b +c
2
= p.
D thy BG = AD = p a.
Do AG = c (p a) = c +a p =
c +a b
2
= b (do a +c = 3b)
Suy ra AGC cn ti A. Tng t, ta c ACF cn ti C.
T ta c

KML =

AGC +

BAF
= 90

BAC
2
+

BAC 90

BAC
2
=

BAC +

ACB
2
=
1
2
_
180

ABC
_
=
1
2

KIL
Suy ra M (I). Do

MLK =

MDK =

DGK =

ACG
Vy ta c iu cn chng minh.
Bi 2.60 Cho tam gic ABC ngoi tip (I). (I) tip xc BC, CA, AB ln lt ti D, E, F.
Chng minh rng tm ng trn ngoi tip cc tam gic AID, BIE, CIH thng hng.
Li gii
A
C
1
B
1
A
1
D
E
F
I
B
C
Gi A
1
, B
1
, C
1
ln lt l trung im ca EF, FD, DE.
Do DA
1
, EB
1
, FC
1
ng quy ti trng tm G ca tam gic DEF.
Xt php nghch o tm I phng tch k = r
2
, bin DA
1
, EB
1
, FC
1
thnh ng trn ngoi
tip cc tam gic IAD, IBE, ICF.
M DA
1
, EB
1
, FC
1
ng quy nn cc ng trn cng cng i qua mt im khc I.
T ta c iu cn chng minh.
132
Bi 2.61 Cho tam gic ABC ni tip (O). M, N ln lt l im chnh gia cung AB khng
cha C v cung AC khng cha B. D l trung im MN. G l mt im bt k trn cung
BC khng cha A. Gi I, J, K ln lt l tm ni tip cc tam gic ABC, ABG, ACG. Ly
P l giao im th hai ca (GJK) vi (ABC). Chng minh rng P DI.
Li gii
Do G, J, M v G, K, N thng hng v t gic PJKG ni tip nn

PJM =

PKN. Li c

PMJ =

PNK nn PJM PKN, suy ra


PM
PN
=
JM
KN
.
M JM = AM, KN = AN nn
PM
PN
=
AM
AN
hay t gic AMPN iu ha.
Gi P

l giao im DI v (O) (P

thuc cung BC khng cha A). Ta c MA = MI v


NA = NI nn A i xng vi I qua MN. V vy MN l ng phn gic ca gc to bi hai
ng thng DA, DP

. Suy ra t gic AMP

N iu ha.
Do P P

v ta c iu cn chng minh.
Bi 2.62 Cho n gic u A
1
A
2
. . . A
n
(n 4) tha mn iu kin
1
A
1
A
2
=
1
A
1
A
3
+
1
A
1
A
4
Hy tm n.
Li gii
Gi R l bn knh ng trn ngoi tip a gic .
p dng nh l sin ta c
A
1
A
2
= 2Rsin

n
, A
1
A
3
= 2Rsin
2
n
, A
1
A
4
= 2Rsin
3
n
133
t

n
= x
_
0 < x

4
_
.
Ta c dy cc ng thc tng ng sau
1
sin x
=
1
sin 2x
+
1
sin 3x
1
sin x
=
sin 2x + sin 3x
2 sin 2x sin 3x
1
sin x
=
sin 2x + sin 3x
2 sin x cos x sin 3x
sin 2x + sin 3x = 2 sin 3x cos x
sin 2x + sin 3x = sin 2x + sin 4x
sin 3x = sin 4x
M 0 < x

4
nn x =

7
.
Do , n = 7 l gi tr duy nht cn tm.
Bi 2.63 Gi AA
1
, BB
1
, CC
1
tng ng l cc ng phn gic trong ca tam gic ABC.
AA
1
, BB
1
, CC
1
ct ng trn ngoi tip tam gic ti A
2
, B
2
, C
2
theo th t. Chng minh
rng :
AA
1
AA
2
+
BB
1
BB
2
+
CC
1
CC
2

9
4
Li gii
A
2
A
1
B
2
B
1
C
2
C
1
O
A
B C
p dng h thc lng trong ng trn, ta c
AA
1
A
1
A
2
= A
1
B A
1
C
M cc h thc quen thuc cho ta :
A
1
B =
ac
b +c
A
1
C =
ab
b +c
AA
1
= l
a
=
_
bc(a +b +c)(b +c a)
b +c
134
Suy ra :
A
1
A
2
=
a
2

bc
(b +c)
_
(a +b +c)(b +c a)
Khi :
AA
1
AA
2
=
AA
1
AA
1
+A
1
A
2
=
(a +b +c)(b +c a)
(b +c)
2
= 1
a
2
(b +c)
2
Do vy bt ng thc cn chng minh tng ng vi :

a
2
(b +c)
2

3
4
Theo bt ng thc Cauchy - Schwarz v Nesbitt, ta c :
3

a
2
(b +c)
2

_

a
b +c
_
2

9
4
T y ta suy ra iu cn chng minh.
Bi 2.64 Cho tam gic ABC, ng thng d ct cc ng thng BC, CA, AB ln lt ti
D, E, F. Gi O
1
, O
2
, O
3
ln lt l tm ng trn ngoi tip cc tam gic AEF, BDF, CDE.
Chng minh rng trc tm tam gic O
1
O
2
O
3
nm trn d.
Li gii
M
O
3
O
2
O
1
E
A
B
C
D
F
Gi M l im Miquel ca t gic ton phn BCEFAD.
Ta li c im i xng ca M qua O
1
O
2
, O
2
O
3
, O
3
O
1
ln lt l D, E, F.
Suy ra M thuc ng trn ngoi tip tam gic O
1
O
2
O
3
v d l ng thng Steiner ca M
i vi (O
1
O
2
O
3
).
Suy ra trc tm ca tam gic O
1
O
2
O
3
nm trn d.
Ta c iu cn chng minh.
135
Bi 2.65 Cho t gic ABCD, AC ct BD ti O. Gi M, N, P, Q ln lt l hnh chiu ca
O trn AB, BC, CD, DA. Bit rng OM = OP, ON = OQ. Chng minh rng ABCD l hnh
bnh hnh.
Li gii
Q
P
N
M
O
E
F
A
D
B
C
Trng hp AB | CD hoc BC | AD th hin nhin ta c iu cn chng minh.
Trng hp khng song song ta s chng minh bng phn chng.
Gi s ABCD khng phi l hnh thang.
Gi E l giao im ca AD, BC; F l giao im ca AB, CD.
T gi thit ta c EO, FO ln lt l phn gic trong

AEB,

CFB.
Ta li c E(ABOF) = 1 nn EF l phn gic ngoi ca

AEB, suy ra EF EO.


Tng t ta c EF FO.
Do FO | EO (v l)
T suy ra iu cn chng minh.
Bi 2.66 Cho tam gic ABC, phn gic trong AD(D BC). Gi M, N l cc im thuc
tia AB, AC sao cho

MDA =

ABC,

NDA =

ACB. Cc ng thng AD, MN ct nhau ti


P. Chng minh rng :
AD
3
= AB AC AP
Li gii
N
M
D
A
B C
136
Ta c :

MDN =

MDA +

NDA =

B +

C = 180


A. Do t gic ADMN ni tip.
Suy ra

AMP =

ADN =

C v

ANP =

ADM =

B.
V vy AMP ACD v ANP ABD. T ta c cc ng thc
AM AD = AP AC (1)
AN AD = AP AB (2)
Mt khc, ADN ACD. Suy ra
AD
2
= AN AC (3)
Tng t ta c :
AD
2
= AM AB (4)
Nhn v theo v cc ng thc trn ta c :
AM AN AD
6
= AM AN AB
2
AC
2
AP
2
Tng ng vi
AD
3
= AB AC AP
Bi ton c chng minh.
Bi 2.67 Trn mt phng cho 2000 ng thng phn bit, i mt ct nhau. Chng minh
rng tn ti t nht 2 ng thng m gc ca chng khng ln hn
180
2000
().
Li gii
Xt mt im O bt k, qua ta v 2000 ng thng tng ng song song vi cc ng
cho. Khi cc gc gia 2 ng thng bo ton.
2000 ng thng trn to thnh 4000 tia chung gc O. Mi cp tia lin tip tng ng vi
mt gc gia 2 ng thng nn c ng 4000 gc v 4000 gc c tng s o l 360

.
Theo nguyn l Dirichlet ta c iu cn chng minh.
Bi 2.68 Cho t gic ABCD ni tip (O) c AB = AD. M, N nm trn cc cnh BC, CD
sao cho MN = BM +DN. AM, AN ct (O) ti P, Q.
Chng minh rng trc tm tam gic APQ nm trn MN.
Li gii
137
J
C
N'
D
B
N
H
A
P Q
M
Gi H l im trn on MN sao cho MH = BM, NH = DN; N

l im trn tia i ca tia


BC sao cho BN

= DN.
Ta c

ABN

ADN v AB = AD nn ABN

= ADN. Suy ra AN

= AN.
Li c MN

= MB + BN

= MB + DN = MN. Do N

v N i xng vi nhau qua AP.


T suy ra H i xng vi B qua AP. Tng t, ta c H i xng vi D qua AQ.
Gi J l giao im ca AH vi (O) th P, Q l trung im ca cc cung BJ, DJ khng cha
A. Suy ra PJ = PB = PH, QJ = QD = QH hay H i xng vi J qua PQ. V vy AHPQ.
Suy ra

PQH =

PQJ =

PAJ =

PAB =

PQB
Do B, Q, H thng hng hay QHAP.
Vy H l trc tm tam gic APQ
Bi 2.69 Cho t gic ABCD. Hai ng cho AC, BD ct nhau ti O. Gi r
1
, r
2
, r
3
, r
4
ln
lt l bn knh cc ng trn ni tip cc tam gic AOB, BOC, COD, DOA.
Chng minh rng
1
r
1
+
1
r
3
=
1
r
2
+
1
r
4
l iu kin cn v t gic ABCD ngoi tip c mt ng trn.
Li gii
t AB = a, BC = b, CD = c, DA = d v OA = x, OB = y, OC = z, OD = t. Gi =

AOB.
Khi ta c dy cc ng thc tng ng sau :
1
r
1
+
1
r
3
=
1
r
2
+
1
r
4
p
AOB
S
AOB
+
p
COD
S
COD
=
p
BOC
S
BOC
+
p
AOD
S
AOD
x +y +a
xy sin
+
z +t +c
zt sin
=
y +z +b
yz sin
+
x +t +d
xt sin
a
xy
+
c
zt
=
b
yz
+
d
xt
138
azt +cxy = btx +dyz
a
2
z
2
t
2
+c
2
y
2
x
2
+ 2acxyzt = b
2
x
2
t
2
+d
2
y
2
z
2
+ 2bdxyzt
2zt cos 2xy cos + 2ca = 2xt cos + 2yz cos + 2bd
(c
2
z
2
t
2
) + (a
2
x
2
y
2
) + 2ca = (d
2
x
2
t
2
) + (b
2
y
2
z
2
) + 2bd
(a +c)
2
= (b +d)
2
a +c = b +d
ng thc cui cng chng t t gic ABCD ngoi tip v cc ng thc trn u tng ng
vi nhau nn ta c iu cn chng minh.
Bi 2.70 Cho tam gic ABC c M l trung im ca BC v H l trc tm tam gic. ng
thng vung gc vi HM ti H ct AB, AC ti D, E. Chng minh rng H l trung im ca
DE.
Li gii
D
E
H
M
A
B
C
P
Q
(i) Cch 1.
Ta c :

DAH =

MCH
_
= 90

ABC
_
v

MHC =

HDA
_
= 90

IHD
_
(I l giao im
ca CH v AB).
Suy ra ADH CHM. Do
DH
HM
=
AH
MC
. V vy ta c
DH =
HM AH
MC
(1)
Hon ton tng t, ta c
HE =
HM AH
MB
(2)
T (1) v (2), kt hp vi MB = MC, ta suy ra HE = HD (iu cn chng minh)
(ii) Cch 2.
Ly P l mt im bt k trn ng thng qua A song song vi DE (P , A), Q l mt im
bt k trn ng thng qua H song song vi BC (Q , H).
Ta c :
MB
MC
= H(BCMQ) v
HD
HE
= A(DEHP).
139
M HB AC, HC AD, HM AP, HQ AH nn H(CBQM) = A(DEHP). Suy ra
MB
MC
=
HD
HE
M M l trung im BC nn ta c iu cn chng minh.
Bi 2.71 Cho on thng AB = a c nh. im M di ng trn AB (M khc A, B). Trong
cng mt na mt phng b l ng thng AB dng hinh vung AMCD v MBEF. Hai
ng thng AF, BC ct nhau N.
Tm v tr im M sao cho on MN c di ln nht.
Li gii
N
A
B
M
D C
F E
(i) Cch 1.
Gi s cc hnh vung AMCD, BEFM c hng dng. Khi R
(M,90

)
: A C, F B.
Suy ra R
(M,90

)
(AF) = CB.
Do

ANB = 90

nn t gic ANCM ni tip.


V vy

ANM =

ACM = 45

hay NM l phn gic ca

ANB.
Mt khc, trong mt tam gic th ng phn gic lun c di nh hn ng trung tuyn
xut pht t cng mt nh. Suy ra
MN
AB
2
(ii) Cch 2.
T cch 1 ta c

DNM =

ENM = 90

. Suy ra D, N, E thng hng.


Do MN l ng cao ca tam gic vung DME. V vy
1
MN
2
=
1
DM
2
+
1
ME
2

1
2
_
1
DM
+
1
EM
_
2

1
2
_
4
DM +ME
_
2
=
4
AB
2
Suy ra
MN
AB
2
140
ng thc xy ra khi M l trung im AB.
Vy MN t gi tr nh nht bng
AB
2
khi M l trung im AB.
Bi 2.72 Cho tam gic ABC nhn khng cn, ni tip (O). Cc ng cao AA
0
, BB
0
, CC
0
ng quy ti H. Cc im A
1
, A
2
thuc (O) sao cho ng trn ngoi tip cc tam gic
A
1
B
0
C
0
, A
2
B
0
C
0
tip xc trong vi (O) ti A
1
, A
2
. B
1
, B
2
, C
1
, C
2
xc nh tng t.
Chng minh rng B
1
B
2
, C
1
C
2
, A
1
A
2
ng quy ti mt im trn OH.
Li gii
X
A
A
0
B
0
C
0
H
O
A
B
C
A
1
A
2
Gi X
A
l giao im ca B
0
C
0
vi BC.
Tng t cho cc im X
B
, X
C
.
D chng minh c X
A
, X
B
, X
C
thng hng v A
1
, A
2
l cc tip im ca hai tip tuyn k
t X
A
i vi (O).
Cc im X
B
, X
C
c nh ngha tng t. Gi l ng thng i qua X
A
, X
B
, X
C
. V
A
1
A
2
l ng i cc ca X
A
i vi (O) nn A
1
A
2
, B
1
B
2
, C
1
C
2
ng quy ti cc ca ng
thng i vi (O).
chng minh im ng quy nm trn OH, ta ch cn chng minh vung gc vi OH.
V B, C, B
0
, C
0
ng vin nn X
A
nm trn trc ng phng ca (O) v ng trn Euler ca
tam gic ABC.
Tng t, suy ra l trc ng phng ca (O) v ng trn Euler ca tam gic ABC.
Do ng thng vung gc vi OH. Ta c iu cn chng minh.
Bi 2.73 Cho ng trn (I) ni tip tam gic ABC tip xc BC, CA, AB ti A
1
, B
1
, C
1
.
Cc ng thng IA
1
, IB
1
, IC
1
tng ng ct cc on thng B
1
C
1
, C
1
A
1
, A
1
B
1
ti A
2
, B
2
, C
2
.
Chng minh cc ng thng AA
2
, BB
2
, CC
2
ng quy.
Li gii
141
V
S
M
B
2
C
2
A
2
A
1
B
1
C
1
I
A
B
C
(i) Cch 1. (S dng tnh cht hng im iu ha)
Ta s chng minh rng AA
2
i qua trung im M ca BC.
Nu AB = AC th iu ny l hin nhin.
Nu AB ,= AC, gi S l giao im ca B
1
C
1
v ng thng qua A song song BC. V l giao
im ca AS v A
1
A
2
.
Khi

AV I = 90

. Suy ra V B
1
IC
1
ni tip, m IB
1
= IC
1
nn V A
2
l phn gic trong ca

B
1
V C
1
V V S V A
2
nn V S l phn gic ngoi ca

B
1
V C
1
.
Do (B
1
C
1
A
2
S) = 1 hay A(BCM

S) = 1 vi M

l giao im ca AA
2
vi BC.
Suy ra M

l trung im BC hay M M

.
Do AA
2
, BB
2
, CC
2
l cc ng trung tuyn ca tam gic ABC nn chng ng quy ti
trng tm tam gic.
(ii) Cch 2. (S dng nh l Menelaus v Ceva)
Do A
1
A
2
, B
1
B
2
, C
1
C
2
ng quy nn theo nh l Menelaus ta c :
A
2
B
1
A
2
C
1

B
2
C
1
B
2
A
1

A
1
C
2
C
2
B
1
= 1
M AB
1
= AC
1
nn :
sin

BAA
2
sin

CAA
2
=
C
1
A
2
B
1
A
2
,
sin

ACC
2
sin

BCC
2
=
B
1
C
2
B
1
C
2
,
sin

CBB
2
sin

ABB
2
=
A
1
B
2
C
1
B
2
Suy ra
sin

BAA
2
sin

CAA
2

sin

ACC
2
sin

BCC
2

sin

CBB
2
sin

ABB
2
=
A
2
B
1
A
2
C
1

B
2
C
1
B
2
A
1

C
2
A
1
C
2
B
1
= 1
Theo nh l Ceva dng sin ta c iu cn chng minh.
(iii) Cch 3.
Tng t nh bi 2.44, ta suy ra AA
2
l trung tuyn ca tam gic ABC. Suy ra AA
2
, BB
2
, CC
2
ng quy ti trng tm tam gic ABC.
Ch . Cch 1 ca bi ton ny c th p dng cho bi 2.44 nh mt cch chng minh khc.
142
Bi 2.74 Cho tam gic ABC cn ti A. Trn tia i ca tia CA ly im E. Giao im ca
BE v phn gic gc

BAC l D. ng thng qua D song song AB ct BC F. AF ct


BE ti M. Chng minh rng M l trung im BE.
Li gii
M
N
F
D
H
B
C
A
E
(i) Cch 1.
Gi N l giao im DF v AC, d c NA = ND.
Ta c

DNC =

EAB v

DCN =

EBA nn DNC EAB. Suy ra


NC
ND
=
AB
AE
Do
FC
FB
=
NC
NA
=
AC
AE
T , p dng nh l Menelaus cho BCE vi cc im A, F, M ta c iu cn chng minh.
(ii) Cch 2.
Gi H l trung im ca BC.
Ta c ABDF l hnh thang nn MH i qua trung im ca AB.
M MH song song vi CE nn suy ra M l trung im ca BE (ng trung bnh trong tam
gic BCE).
Bi 2.75 Cho t gic li ABCD sao cho AB ko song song vi CD v im X bn trong t
gic tha

ADX =

BCX < 90

DAX =

CBX < 90

. Gi Y l giao im ng trung trc


ca AB v CD. Chng minh rng

AY B = 2

ADX.
Li gii
143
N
M
O'
O
Z
O
2
O
1
Y
B
A
D
C
X
B : Cho hai ng trn (O
1
) v (O
2
) ct nhau ti X, Z. Ly A l mt im bt k nm
trn (O
1
). Dng tia ZB i xng tia ZA qua ZX vi B thuc (O
2
). Gi O l tm ngoi tip
ABZ. Khi ta c OO
1
= OO
2
.
Chng minh b .
(OO
1
, O
1
O
2
) (OO
1
, AZ) + (AZ, ZX) + (ZX, O
1
O
2
)
(O
1
O
2
, ZX) + (ZX, ZB) + (ZB, OO
2
)
(O
1
O
2
, OO
2
) (mod )
Do tam gic OO
1
O
2
cn ti O nn OO
1
= OO
2
.
B c chng minh.
Tr li bi ton :
Gi (O
1
), (O
2
) ln lt l ng trn ngoi tip XAD v XBC.
Gi Z l giao im th hai ca (O
1
), (O
2
).
Gi (O), (O

) ln lt l ng trn ngoi tip ZAB v ZCD.


Gi Y

l giao im th hai ca (O), (O

).
Ta c : M = ZX (O)(M ,= Z), N = ZX (O

) (N ,= Z)
Ta c : (ZA, ZX) (DA, DX) (CX, CB) (ZX, ZB) (mod ) nn p dng b trn ta
c OO
1
= OO
2
. Tung t ta cng c O

O
1
= O

O
2
.
Suy ra OO

O
1
O
2
.
Mt khc XZ O
1
O
2
, ZY

OO

nn ZY

ZX.
144
Xt (O) c ZY

ZM v M l im chnh gia cung AB khng cha Y

, ta suy ra Y

A = Y

B.
Tng t : Y

C = Y

D nn Y

Y .
V vy

AY B =

AZB = 2

ADX. Ta c iu cn chng minh.


Bi 2.76 Cho t gic li ABCD ni tip trong (O). AD ct BC ti E, AC ct BD ti
F.M, N l trung im AB, CD. Chng minh rng :
2MN
EF
=

AB
CD

CD
AB

Li gii
V
U
P
N
M
G
E
F
B
A
C
D
Gi s AB < CD, BC < AD.
Gi P l trung im EF. Khi M, N, P thng hng (p dng nh l Gauss cho t gic ton
phn AEBFDC) v M nm trn on PN.
Trc ht, ta s chng minh
2PM
EF
=
AB
CD
(1)
Tng ng vi
PM
AB
=
PF
CD
Gi U l im i xng vi F qua N, V l trung im EU.
Ta c CFDU l hnh bnh hnh. Nn

ADU = 180

CAD = 180

CBD =

EBF.
Mt khc, ta c
FB
DU
=
FB
FC
=
AB
CD
=
EB
ED
Do EBF EDU. Suy ra
PB
AB
=
V D
DC
145
Tng t, ta suy ra PAB V CD.
T ta c
PM
AB
=
V N
CD
=
PF
CD
ng thc (1) c chng minh.
Hon ton tng t, ta chng minh c
2PN
EF
=
CD
AB
(2)
T (1) v (2) ta suy ra iu cn chng minh.
Bi 2.77 Cho t gic ABCD ni tip c mt ng trn. Chng minh rng :
AC
BD
=
DA AB +BC CD
AB BC +CD DA
Li gii
(i) Cch 1.
F
E
A
B
D
C
K dy DE, CF song song vi AC, BD tng ng.
Ta c : AE = DC, CE = AD.
p dng nh l Ptolemy ta c
EA BC +AB CE = AC BE
Tng ng vi
DC BC +AB DA = AC BE (1)
Tng t, ta c
AB BC +CD DA = AF BD (2)
Chia theo v hai ng thc (1) v (2), vi ch rng AF = BE, ta c iu cn chng minh.
146
(ii) Cch 2.
Ta c :
AC AB BC +AC CD DA = 4R(S
ABC
+S
CDA
)
= 4R(S
ABD
+S
BCD
)
= DA AB BD +BC CD BD
T suy ra iu cn chng minh.
Bi 2.78 Cho tam gic nhn ABC ni tip (O; R).Gi R
1
, R
2
, R
3
tng ng l bn knh
ng trn ngoi tip cc tam gic OBC, OCA, OAB. Chng minh rng :
R
1
+R
2
+R
3
3R
Li gii
O
A
B C
Gi O
1
l tm ngoi tip ca tam gic OBC.Ta c :
R
1
= O
1
B,

BOO
1
=

A
p dng nh l hm s sin cho tam gic BOO
1
ta c :
R
1
= O
1
B =
OBsin

BOO
1
sin

BO
1
O
=
Rsin A
sin 2A
=
R
2 cos A
Do
R
1
+R
2
+R
3
=
R
2
_
1
cos A
+
1
cos B
+
1
cos C
_

R
2

9
cos A + cos B + cos C
3R
n y chng minh hon tt.
Gia nhng b c thng minh ngang nhau v trong nhng iu kin tng t,
ai c tinh thn hnh hc th ngi s thng v thu c mt cng lc hon ton mi m.
Blaise Pascal
Hnh hc l khoa hc ca l lun chnh xc trn cc s liu khng chnh xc.
George Plya
Hnh hc l nn tng ca tt c cc bc tranh.
Albrecht D urer
Cm hng lun cn thit trong hnh hc, cng ging nh trong thi ca.
Alexander Pushkin

You might also like